SlideShare ist ein Scribd-Unternehmen logo
1 von 63
Downloaden Sie, um offline zu lesen
Hoàng Vi t Quỳnh
Toaën hoåc phöí thöng
Các phương pháp gi i nhanh thi
i h c
WWW.MATHVN.COM
Liên hệ: Ketnoitrithuc2013@gmail.com
http://toilaquantri.com - Chia sẻ nhiều tài liệu LTĐH bổ ích
https://www.facebook.com/phung.huynh.93.1102
1
Các phương pháp gi i toán i s và
gi i tích
L i nói u:
Sau 12 năm h c t p, gi ây ch còn m t kì thi duy nh t ang ch i các em ó là kì thi i
h c. ây s là kì thi khó khăn nh t trong su t 12 năm các em ng i trên gh nhà trư ng. Kì thi
i h c chính là m t bư c ngo t l n trong cu c i c a m i h c sinh vì th m i h c sinh c n
ph i chu n b ki n th c th t toàn di n vì n i dung c a thi mang tính liên t c. Có l trong các
môn, môn toán v n luôn chi m v trí quan tr ng và là v t c n l n nh t trên bư c ư ng ti n t i
gi ng ư ng i h c. Vì th tôi xin m o mu i góp chút ki n th c ã thu lư m ư c trong quá
trình h c t p vi t lên quy n sách này. Hy v ng ây s là tài li u b ích cho các em h c t p.
Quy n sách ư c chia thành sáu ơn v bài h c và hai ph l c. M i bài u là nh ng ph n
quan tr ng, xu t hi n thư ng xuyên trong thi i h c. m i bài u có nh ng c i m
sau:
• Ph n tóm t t ki n th c ã h c ư c trình bày ng n g n và t ng quát nh m khơi l i ph n
ki n th c ã quên c a các em.
• H th ng các bài làm ư c ch n l c kĩ lư ng, có tính i n hình và khai thác t i a các
góc c nh c a v n nêu ra, ng th i phương pháp gi i ng n g n, tr c quan cùng nhi u
kinh ngh m gi i giúp các em có th hi u ư c n i dung bài gi i và cách áp d ng cho các
d ng thi s g p sau này. ng th i, các ví d u ư c trình bày t cơ b n n nâng cao.
ây là nh ng bài trích ra t thi d tr c a các năm trư c và tham kh o t nh ng tài
li u c a các th y cô có nhi u năm kinh nghi m trong quá trình luy n thi nên m b o v
m c và gi i h n ki n th c. L i gi i trong các ví d ch là tư ng trưng nh m m c ích nêu
lên phương pháp gi i, các em và các th y cô khi tham kh o cu n t i li u này có th tìm ra và
trình bày cách gi i và cách trình bày h p lí hơn. Các em nên t p gi i các d ng bài trên m t
cách thu n th c và c l p. sau khi gi i xong m i xem ph n l i gi i. ó là i u mà tác gi kì
v ng nhi u nh t.
• Lí gi i các phương pháp, ưa ra thu t toán gi i chung, ưa ra b n ch t l i gi i, ó là
ph n l i bình, lưu ý cu i m i bài t p.
Ph n ph l c là 12 thi tiêu bi u theo c u trúc thi m i nh t do B GD& T công b . Các
thi có m c khó r t cao, òi h i ngư i làm ph i tư duy r t nhi u. V i m c khó ó, tôi
mong r ng khi các em gi i thu n th c các bài trong b thi này các em s có t tin và ki n
th c t i m cao khi làm bài môn toán. Ph l c 2 là m t s m o dùng máy tính oán
nghi m c nh, ph c v cho quá trình gi i các bài t p v phương trình tích như lư ng giác, h
phương trình, phương trình, cách gi i nhanh bài toán hình h c b ng máy tính… ng th i gi i
thi u thêm phương pháp chia Horner giúp các em làm nhanh bài toán có chia a th c, phân
tích thành tích…
V i d nh là s gi i thi u quy n sách cho các em trong tháng cu i cùng trư c khi thi i
h c nên sách ã gi n lư c m t s ph n không c n thi t và các ki n th c bên l , ch gi i thi u
nh ng tr ng tâm c a thi nên bài t p có th còn ít. Tôi cũng có l i khuyên cho các thì sinh là
hãy tìm thêm các thi trên m ng internet vì ây là kho ki n th c vô t n.
M c dù r t c g ng nhưng cu n sách r t có th còn nhi u thi u sót do th i gain biên so n
ng n ng th i kinh nghi m và s hi u bi t còn h n ch . R t mong ư c s góp ý c a b n c.
M i góp ý xin liên h v i tác gi qua a ch sau:
Hoàng Vi t Quỳnh
Khu 6a – Th tr n L c Th ng – B o Lâm – Lâm ng
Email: vquynh2971991@yahoo.com.vn
Blog: http://vn.myblog.yahoo.com/vquynh-qflower
Tel: 063-3960344 - 01676897717
WWW.MATHVN.COM
2
Bài I: ng d ng phương trình ư ng th ng
gi i phương trình căn th c.
VD1. Nh c l i ki n th c v ư ng th ng.
1) Phương trình t ng quát:
ư ng th ng i qua M(x0;y0) và có vetơ pháp tuy n n (A;B) thì ư ng th ng ó có phương trình:
(d): A(x-x0)+B(y-y0)=0
(d): Ax+By+C=0
VD1. ư ng th ng qua M(1;2) nh n n (2;1) làm vectơ pháp tuy n.
(d): 2(x-1)+1(y-2)=0
(d): 2x+y-4=0
2) Phương trình tham s :
ư ng th ng i qua M(x0;y0) và có vectơ ch phương a (a1;a2)
(d):



+=
+=
tayy
taxx
20
10
VD2. ư ng th ng qua M(3;4) nh n a (2;3) làm vtcp có phương trình:
(d):



+=
+=
ty
tx
34
23
VD3. Cho (d): x+y=4. Vi t phương trình tham s c a (d).
Gi i:
Vectơ pháp tuy n : n (1,1)
Vectơ ch phương : a (1,-1)
i m i qua M(2;2)
(d) :



−=
+=
ty
tx
2
2
VD2. ng d ng
VD1. Gi i phương trình : 101238 33
=−++ xx
Gi i:
t: 83
+x =1+3t và
3
12 x− =3-t k( -1/3 ≤t≤1/3)
x3
+8=(1+3t)2
(*) và 12-x3
= (3-t)2
(**)
L y (*)+(**) ta có 20=10t2
+10 t2
=1 t=1 ho c t=-1(lo i)
x3
=8 x=2
Tip:
Có ph i b n ang t h i: thu t toán nào ã giúp ta nhìn th y ư c cách t n t ???
WWW.MATHVN.COM
3
Không ph i ng u nhiên mà tôi l i trình bày l i v n ư ng th ng, m t v n tư ng ch ng như
ch ng liên quan gì n i s . Nhưng gi ây ta m i nh n ra ư c “ ư ng th ng” chính là “tuy t chiêu”
gi i phương trình d ng căn th c. M u ch t ó là:
B1: 101238 33
=−++
YX
xx
T ó ta có phương trình ư ng th ng : X+3Y=10
B2: ta vi t l i phương trình: X+3Y=10 theo tham s t



=
=
t-3Y
3t+1X
Lúc này phương trình ã quy v 1 n t và vi c gi i phương trình trên là không khó. (Vì ây là ki n th c
“l p nhí”)
hi u rõ hơn v phương pháp này các b n hãy cùng tôi n v i VD2.
VD2. Gi i phương trình :
X
x 3+ +
Y
x3
2+ =1
Gi i:
G i (d): X=1+t và Y=0+t
(1) t




=+
−=+
tx
tx
3
2
13
(t≤1)




=+
+−=+
3
2
2
213
tx
ttx
L y phương trình 2 tr pt1 ta có: -1=t3
-t2
+2t-1 t3
-t2
+2t=0
• T=0 x=-2
Lưu ý:
Trong khi gi i thi, các b n nên trình bày t bư c(1) tr i nh m m b o tính ng n g n cho bài toán.
Bư c g i phương trình ư ng th ng ch nên làm ngoài gi y nháp.
• Trong bài trên ta có th t




=+
=+
vx
ux
3
2
3
và quy v gi i h phương trình. Các b n có th xem
cách này như m t bài t p. các b n hãy làm và so sánh s ưu vi t gi a 2 phương pháp.
• Trong bài trên ta h n ch phương pháp lũy th a vì n u mu n kh 2 căn th c khác b c trên, ta ph i
^6 phương trình. Ta s g p khó khăn và s i m t v i 1 phương trình “kinh kh ng” và ta ph i gi i
“x t khói” m i có th ra nghi m.
VD3. Gi i h phương trình :
( )
( )



=+++
=−+
2411
13
yx
xyyx
( thi H năm 2005)
Gi i:
t:




−=+
+=+
ty
tx
21
21
(-2≤t≤2)




+−=+
++=+
441
441
2
2
tty
ttx




+−=
++=
34
34
2
2
tty
ttx
Phương trình(1) tr thành: 2t2
+6- )43)(43( 22
tttt −+++ =3
WWW.MATHVN.COM
4
910 24
+− tt =2t2
+3
ho c
t=0 x=y=3
VD4. nh m phương trình sau có nghi m:
Gi i:
phương trình có nghi m:
mxf =)(
Min f(x)≤m ≤Max f(x)
t




−=−
+=+
txm
tmx
33
312
(-1/3≤t≤3)




+−=−
++=+
2
2
693
9612
ttxm
ttmx
c ng v v i v => 5m=10+10t2
2t2
+2=m f(t)=m
V i f(t)= 2t2
+2 mi n xác nh: D=[-1/3;3]
F’(t)=4t =>f’(t)=0 t=0
t -∞ -1/3 0 3 +∞
F’(t) - 0 +
F(t)
20/9 20
2
M có nghi m 2≤m≤20
VD3. Bài t p t luy n
1) Gi i h phương trình:
2) Gi i h phương trình:
3) Gi i h phương trình:
2 1 1 1
3 2 4
x y x
x y
 + + − + =

+ =
( thi d b 1A – 2005)
4) Gi i phương trình: 1 sin( ) 1 cos( ) 1x x− + + = ( thi d b 2A – 2004)
WWW.MATHVN.COM
5
Bài II: Các cách gi i phương trình và b t phương trình
vô t .
1)Lũy Th a
Phương pháp lũy th a là phương pháp t ng quát nh t gi i phương trình có căn. Khi g p các phương
trình có d ng căn ph c t p nhưng khi chúng ta bi t “m o lũy th a” thì có th gi i bài toán m t cách d
dàng. ây là m t phương pháp cơ b n, các b n ph i th c t p nhu n nhuy n vì phương trình trong thi
i h c có lúc r t d nhưng ta l i không ý. các b n hãy theo dõi các ví d sau. Nhưng trư c h t hãy
lưu ý v n sau:
• t i u ki n
• Lũy th a ch n thì hai v không âm
• Các d ng cơ b n:
BA =



=
≥
2
0
BA
B
BA <



≤≤
≥
2
0
0
BA
B
BA >










>
≥



≥
<
2
0
0
0
BA
B
A
B
VD1.
Gi i:







=−+−+
≥−
≥−
≥
10)5(25
010
05
0
xxxx
x
x
x




−=−
≤≤
xxx
x
552
50
2



+−=−
≤≤
22
1025)5(4
50
xxxx
x



=+−
≤≤
056
50
2
xx
x
x=1 ∨ x=5
VD2. 132 −<+− xxx
Gi i:
2 x = 3−x + 1−x




−++−++<
≥
)1)(3(2134
1
xxxxx
x




−>−+
≥
132
1
2
xxx
x



+−>−+
≥
1232
1
22
xxxx
x



>
≥
1
1
x
x
x=1
WWW.MATHVN.COM
6
VD3.
Gi i:
k: 2x+1>0 x>1/2
Bpt (4x2
-4x+1)(x2
-x+2)≥36
t t = (x2
-x) bpt tr thành:
(4t+1)(t+2)≥36
4t2
+9t-34≥0
t≤-17/4 ho c t≥2
x2
-x≤-17/4 ho c x2
-x≥2
x≤1 ho c x≥2
VD4. Gi i b t phương trình :
Gi i:









≥−−
>−
=+−
02
0
0
2
2
2
xx
xx
xx
10 =∨=⇔ xx
Lưu ý:
b t phương trình trên các b n không nên lũy th a tính toán vì quá trình lũy th a và nhân phân ph i
r t m t th i gian. Hơn n a, khi quy v m t phương trình h qu , chúng ta gi i r t d sai vì khi giao các
t p nghi m s không có giá tr nào th a mãn.
Trong bài trên tôi s d ng cách ánh giá theo ki u như sau:
A B ≥0








≥
>
=
0
0
0
A
B
B
ó chính là m u ch t c a bài toán
VD5. Gi i phương trình :
Gi i:














 −
=−
≥−
≥




 −
−
2
2
4
53
8
053
0
4
53
2
x
x
x
x
x=3
WWW.MATHVN.COM
7
Lưu ý:
Trong phương trình trên các b n ph i “ ý” và “nhanh” m t chút vì n u như ta nguyên phương trình
cho lũy th a thì ó là m t i u “không còn gì d i b ng” ta s i m t v i chuy n lũy th a 2 l n =>
m t phương trình b c 4. Phương trình này ta không th b m máy tính. Nhưng n u gi i tay thì ph i gi i “x t
khói” m i ra trong khi th i gian không ch i ai. ng th i chúng ta không c n gi i i u ki n v i vì giám
kh o ch quan tâm n bài làm và k t qu . Chúng ta hãy ch vi t “cái sư n” c a i u ki n. sau khi gi i ra
nghi m ch vi c th vào i u ki n là xong.
2) Phương pháp t n ph :
CÁCH GI I:
( )
( )
( ) 0)();(
0)();(
0)();(
=
≤
≥
n
n
n
xuxuf
xuxuf
xuxuf
t= n xu )( Phương trình h u t ho c h phương trình
BÀI T P ÁP D NG:
VD1.
Gi i:
t t= => t>0 ; t2
+2= x2
+ x
3t=2(t2
-1)
t=-0.5 (lo i) ho c t=2
x2
+x=6 x=2 ho c x=3
VD2.
Gi i:
T= 1−x



=+
≥
xt
t
1
0
2
Phương trình tr thành:
t2
+1-(t+1)=2 t2
-t-2=0 t=2 ho c t=-1
x=5
VD3.
Gi i:
=>
WWW.MATHVN.COM
8
pt tr thành: t2
+t+2=8 t=2 ∨ t=-3
TH1: t=2
TH2: t=-3
LO I II: ( )nn xvxuf )()( + { ≥0; ≤0; =0 }
Phương pháp chung:




=
=
vxv
uxu
m
n
)(
)(
=> ưa v h phương trình.
VD1. 085632323
=−−+− xx ( tuy n sinh i h c 2009)
Gi i:




≥=−
=−
)0(56
233
vvx
ux




=−+
=+
0832
3
8
3
5 23
vu
vu






−
=
=+
3
28
3
8
3
5 23
u
v
vu







−
=
=




 −
+
3
28
3
8
3
28
3
5
2
3
u
v
u
u





−
=
=+−+
3
28
0)202615)(2( 2
u
v
uuu



=
−=
4
2
v
u
x=-2
LO I III: H PHƯƠNG TRÌNH A TH C
Nh ng h phương trình này ta r t thư ng hay g p trong thi i h c. l p 10, ta thư ng g p nh ng
phương trình có tên là h i x ng, ng c p… Nh ng h này ã có cách gi i “ăn li n”. nhưng trong thi
i h c, ta không h tìm th y nh ng d ng ó. Nhưng t t c các h trên u quy v m t m i ó là “Phân
tích thành nhân t ”.
WWW.MATHVN.COM
9
VD1. Gi i h phương trình:
( )
( )3
1 1
1
2 1 2
x y
x y
y x

− = −

 = +
( H A 2003)
Gi i:
K: xy≠0
Ta có ( ) ( )
1
1 1 0
1
x y
x y
xyxy
=  
⇔ − + = ⇔   = −  
TH1:
( )( )23 3
1
1 5
1 1 0 22 1 2 1
1 5
2
x y
x yx y x y
x y
x x xy x x x
x y

 = =
== =  − + ⇔ ⇔ ⇔ = =   − + − == + = +   
− − = =
TH2: 3
3 4
1
1
1
22 1
1 2 0
y
xy yx
x
y x
x x x
x

= −= − = −  
⇔ ⇔  
= +  − = + + + =
Mà
2 2
4 2 1 1 3
2 0,
2 2 2
x x x x x VN
   
+ + = − + + + > ∀ ⇒   
   
V y nghi m c a h là ( ) ( )
1 5 1 5 1 5 1 5
; 1;1 , ; , ;
1 1 1 1
x y
   − + − + − − − −
=       
   
VD2. Gi i h phương trình:
( )
( )
( )
2
2
x 1 y(y x) 4y 1
x, y R .
(x 1)(y x 2) y 2
 + + + =
∈
+ + − =
(D b A2006)
Gi i:
( ) ( ) ( )2
1 1 4 0 *x y x y⇔ + + + − =
t:
2
1 0; 4u x v x y= + > = + −
H
( )
( ) ( )
0 3
2 4
u yv
u v y
− =
⇔ 
+ =
Thay (4) vào (3) ta có: ( ) ( ) ( )3 2 . 0 1 2 0u u v v u v v⇔ + + = ⇔ + + =  
2
2 1 0v v⇔ + + = 2
( 1) 0 1 3v v x y⇔ + = ⇔ = − ⇔ + =
V y (*) ( )
2
2
1 21 0
1 3 0
2 53
x yx y
x x
x yx y
= ⇒ = − + − = 
⇔ ⇔ + − − = ⇔  = ⇒ == − 
VD3. Gi i h phương trình
( )
( )
3 3
2 2
x 8x y 2y
x, y R .
x 3 3(y 1) *
 − = +
∈
− = +
(D b 2A 2006)
Gi i:
H
( ) ( ) ( ) ( )
( )
3 33 3
2 2 2 2
3 6 4 2 12 4
3 6 3 6 2
x y x yx y x y
x y x y
 − = + − = + 
⇔ ⇔ 
− = − =  
L y (2) thay vào (1) ta có
( ) ( )( )3 3 2 2 3 2 2
3 3 4 12 0x y x y x y x y x x y⇔ − = − + ⇔ − + = ( )2 2
12 0x x xy y⇔ + − =
D th y x=0 thì y=0. Th vào (*) ta th y không th a mãn. V y ây không ph i là nghi m c a phương
trình:
WWW.MATHVN.COM
10
( )( )2 2
2 2 2 2
3 4 012 0
3 6 3 6
x y x yx xy y
x y x y
 − + = + − = 
⇒ ⇔ 
− = − =  
TH1: 2 2 2
3 0 3 1 3
1 33 6 6 6
x y x y y x
y xx y y
− = = = ⇒ =  
⇔ ⇔   = − ⇒ = −− = =  
TH2: 2 2 2
78 4 78
4 4 13 13
3 6 13 6 78 4 78
13 13
y x
x y x y
x y y
y x
 −
= ⇒ == − = − 
⇔ ⇔ 
− = = 
= − ⇒ =

V y nghi m c a phương trình là:
( ) ( ) ( )
78 4 78 78 4 78
; 1;3 , 1; 3 , ; , ;
13 13 13 13
x y
   − −
= − −       
   
VD4. Gi i h phương trình
( )( ) ( )
( )( ) ( )
2 2
2 2
13 1
25 2
x y x y
x y x y
 − + =

+ − =
(D b 2005)
Gi i:
Nhân c 2 v c a (1) cho 25. Nhân c 2 v c a (2) cho 13. Sau ó l y (1)-(2).
(1)-(2) ( ) ( )( ) ( ) ( ) ( )22 2 2 2 2
13( ) 25 0 13 25 0x y x y x y x y x y x y x y ⇔ + − − − + = ⇔ − + − + =
 
( )( ) ( )( )2 2 2 2
12 26 12 0 2 12 26 12 0x y x xy y x y x xy y⇔ − − + − = ⇔ − − − + − =
D th y x=y không th a mãn h .
( )( )
( )( ) ( ) ( )
2
2 2
2
2
3 2
3
25
3 2 . 25 2
3 2 2 3 0 9 3
2 3
2 325
25
325 1
. 25
24 2
x y
y
y
x y y x
x y x y
x y
x yx y x y
x y x y
x
y y
y
 =
= − ⇔−   = = = −  − − =    = ⇒ ⇔ ⇔ 
=+ − =   + − =  =   = ⇔   =  
L i bình:
Làm sao ta có th phân tích nhanh ( )2 2
12 26 12x xy y− + − thành nhân t ( )( )3 2 2 3x y x y− − ??
Lúc này, công c c a chúng ta chính là máy tính b túi! Các b n hãy làm như sau:
Coi như ta không th y n y. v y nên ta có phương trình b c 2 theo x:( )2
12 26 12 0x x− + − = Ch c
h n các b n u bi t gi i phương trình b c 2 này b ng máy CASIO. Ta b m ư c nghi m:
3 2
2 3
x x= ∨ = . Lúc này ta g i l i n y b ng cách thêm y vào sau các nghi m tìm ư c.
3 2
2 3
x y x y= ∨ = . Quy ng b m u vì m u là h ng s . ta có nhân t c n phân tích. Lưu ý là
( )2 2
12 26 12 0x xy y− + − = ⇔ ( )( )3 2 2 3 0x y x y− − = . N u gi i b t phương trình, b n nên chú ý n
d u khi phân tích (Trư ng h p này là d u - : ( ) ( )( )2 2
12 26 12 2 3 2 2 3 0x xy y x y x y− + − = − − − = )
Khi g p d ng phương trình a th c có h ng s phía v ph i (ho c có th ưa c 2 phương trình
v d ng có h ng s v ph i), Ta nhân c 2 v c a phương trình trên cho s v ph i c a phương
trình dư i và nhân c 2 v c a phương trình dư i cho s phương trình trên. Sau ó tr v theo
WWW.MATHVN.COM
11
v . M c ích c a phương pháp này là quy h v phương trình tích sau ó ti n hành phân tích. H u
h t các lo i phương trình a th c u gi i ư c theo cách này!
Bài t p t luy n
Bài 1.
4 3 2 2
3 2
1
1
x x y x y
x y x xy
 − + =

− + =
Bài 2.
( ) ( )
2 2
4
1 1 2
x y x y
x x y y y
 + + + =

+ + + + =
Bài 3.
( )
( )
2 2
22 2
3
7
x xy y x y
x xy y x y
 − + = −

+ + = −
Bài 4.
( )
( )
3 2
3 2
log 2 3 5 3
log 2 3 5 3
x
y
x x x y
y y y x
 + − − =

+ − − =
Bài 5.
( )
( )
2
2
1 3 0
5
1 0
x x y
x y
x
 + + − =


+ − + =

Bài 6.
9 9
25 25 16 16
1x y
x y x y
 + =

+ = +
Bài 7.
4 3 2 2
2
2 2 9
2 6 6
x x y x y x
x xy x
 + + = +

+ = +
Bài 8. 2 2 2
1 7
1 13
xy x y
x y xy y
+ + =

+ + =
Bài 9.
( )
3
4
1 8
1
x y x
x y
 + − = −

− =
Bài 10.
2
2
2
2
2
3
2
3
y
y
x
x
x
y
 +
=


+ =

Bài 11.
3
1 1
2 1
x y
x y
y x

− = −

 = +
WWW.MATHVN.COM
2
Bài III: Phương trình lư ng giác.
M t s công th c lư ng giác c n nh :
1.
2 2 2 2
2 2
1 1
sin x cos x 1;1 tan ;1 cot .
cos sin
x x
x x
+ = + = + =
2.
sin cos 1
tanx ;cot x ;tan
cos sin cot
x x
x
x x x
= = = .
3. Công th c c ng:
sin( ) sin cos cos
cos( ) cos cos sin sin
a b a b asinb
a b a b a b
± = ±
± =
4. Công th c nhân ôi: sin2x = 2sinxcosx
5. cos2x = cos2
x – sin2
x = 2 cos2
x – 1 = 1 - 2 sin2
x
6. Công th c h b c:
2 21 cos2 1 cos2
cos ;sin
2 2
x x
x x
+ −
= =
7. Công th c nhân ba: Sin3x = 3sinx – 4sin3
x; cos3x = 4cos3
x – 3cosx.
8. Công th c bi u di n theo tanx:
2
2 2 2
2tan 1 tan 2tan
sin 2 ;cos2 ;tan 2
1 tan 1 tan 1 tan
x x x
x x x
x x x
−
= = =
+ + −
9. Công th c bi n i tích thành t ng
( )
( )
( )
1
cos cos cos( ) cos( )
2
1
sin sin cos( ) cos( )
2
1
sin cos sin( ) sin( )
2
a b a b a b
a b a b a b
a b a b a b
= − + +
= − − +
= − + +
10.Công th c bi n i t ng thành tích
sin sin 2sin cos
2 2
sin sin 2cos sin
2 2
cos cos 2cos cos
2 2
cos cos 2sin sin
2 2
x y x y
x y
x y x y
x y
x y x y
x y
x y x y
x y
+ −
+ =
+ −
− =
+ −
+ =
+ −
− = −
WWW.MATHVN.COM
3
Cách gi i các phương trình lư ng giác trong thi i h c:
Lưu ý trư c khi gi i :
Các phương trình lư ng giác trong thi i h c nhìn qua m t h c sinh thư ng r t khó khăn ph c t p
nhưng chúng u quy v nh ng phương trình ơn gi n. thi i h c các năm u xoay quanh bi n
i v d ng phương trình tích, t n ph . Năm 2009, thi có bi n i hơn ó là phương trình cu i
bi n i v d ng công th c c ng. Nhìn chung phương pháp gi i d ng toán này là các em h c thu c các
công th c trên ây và rèn luy n kĩ năng phân tích a th c thành nhân t …
GI I M T S THI TIÊU BI U:
1. Gi i phương trình: 2sin 2 4sin 1 0
6
x x
π 
− + + = 
 
(1)
Gi i:
(1) 3sin 2 cos2 4sin 1 0x x x− + + = ( ) 2
2sin 3 cos2 2 2sin 0x x x+ − =
( )2sin 3 cos sin 2 0x x x− + =
sinx 0
1
3 cos sin 1 cos cos
2 6
x k
x x x x
π
π
= ⇔ =

  − = − ⇔ + =   
5
2
6
7
2
6
x k
x k
x k
π
π
π
π
π

 =

 = +

 −
 = +

2. Tìm nghi m trên kho ng (0; π ) c a phương trình :
Gi i:
Tìm nghi m ( )0,∈ π
Ta có
2 2x 3
4sin 3 cos2x 1 2cos x
2 4
π 
− = + − 
 
(1)
(1) ( )
3
2 1 cosx 3 cos2x 1 1 cos 2x
2
π 
⇔ − − = + + − 
 
(1) 2 2cosx 3 cos2x 2 sin2x⇔ − − = −
(1) 2cosx 3 cos2x sin2x⇔ − = − . Chia hai v cho 2:
(1) ⇔ − = −
3 1
cosx cos2x sin2x
2 2
( )cos 2x cos x
6
π 
⇔ + = π − 
 
( ) ( )
π π π
⇔ = + = − + π
5 2 7
x k a hay x h2 b
18 3 6
2 2 3
4sin 3 cos2 1 2cos ( )
2 4
x
x x
π
− = + −
WWW.MATHVN.COM
4
Do ( )x 0,∈ π nên h nghi m (a) ch ch n k=0, k=1, h nghi m (b) ch ch n h = 1. Do ó ta có ba
nghi m x thu c ( )0,π là 1 2 3
5 17 5
x ,x ,x
18 18 6
π π π
= = =
3. . Gi i phương trình : 3
2 2 cos ( ) 3cos sin 0
4
x x x
π
− − − = (2)
Gi i:
(2)
3
2 cos x 3cosx sinx 0
4
π  
⇔ − − − =  
  
( )⇔ + − − =
⇔ + + + − − =
3
3 3 2 2
cosx sinx 3cosx sinx 0
cos x sin x 3cos xsinx 3cosxsin x 3cosx sinx 0
=
⇔ 
− =
3
cosx 0
sin x sinx 0
≠

+ + + − − − − =
2 3 2 3
cosx 0
hay
1 3tgx 3tg x tg x 3 3tg x tgx tg x 0
⇔ =2
sin x 1 =haytgx 1 x k
2
π
⇔ = + π hay
π
= + πx k
4
4. . Gi i phương trình : 2
2
cos2 1
( ) 3
2 cos
x
tg x tg x
x
π −
+ − = ( d b kh i B 2005)
Gi i:
(2)
2
2
2
2sin x
cotgx 3tg x
cos x
−
⇔ − − =
π
⇔ − − = ⇔ = − ⇔ = − ⇔ = − + π ∈2 31
tg x 0 tg x 1 tgx 1 x k ,k Z
tgx 4
PHƯƠNG PHÁP T N PH TRONG PHƯƠNG TRÌNH LƯ NG GIÁC:
A. t t=sinx
Cos2
x= 1 – sin2
x = 1-t2
t∈[-1;1]
Tan2
x =
2
2
sin
cos
x
x
=
2
2
1
t
t−
Cos2x =
2
1 2sin x− = 1-2t2
Sin3x =
3 3
3sin 4sin 3 4x x t t− = −
B. t t = cosx
2 2 2
sin 1 cos 1x x t= − = − 2
cos2 2 1x t= +
2 2
2
2 2
sin 1
tan
cos
x t
x
x t
−
= = 3 3
cos3 4cos 3cos 4 3x x x t t= − = −
C. t t= tanx
WWW.MATHVN.COM
5
1
cot x
t
= 2
2
1
cos
1
x
t
=
+
2
2
2
sin
1
t
x
t
=
+
2
2
1
cos 2
1
t
x
t
−
=
+
2
1
sin2x=2t
1 t
 
 
+ 
2
2
t an2
1
t
x
t
=
+
sin cos tan
sin cos tan
a x b x a x b at b
c x d x c x d ct d
+ + +
= =
+ + +
D. t t=sinx ± cosx t∈ 2; 2 − 
sinxcosx
2
1
2
t −
=
±
sin2x= ( )2
1t± +
( )( )
2 3
3 3 2 2 1 3
sin cos sin cos sin cos sin cos 1
2 2
t t
x x x x x x x x t
 − −
+ = + + − = − = 
 
NGUYÊN T C CHUNG GI I PHƯƠNG TRÌNH LƯ NG GIÁC
Bi n i: t t
Phân tích thành tích
Nguyên t c :
Lũy th a H b c
Tích T ng
T ng Tích
Bi n i không ư c thì i bi n.
GI I M T S THI TIÊU BI U:
Bài 1.
2cos2 1
cot 1 sin sin 2
1 tan 2
x
x x x
x
− = + −
+
Gi i:
t t=tanx, pt tr thành:
( )
2
2 2
2 2
1
11 1 2
1 0; 1
1 1 2 1
t
t t t
t t
t t t t
 −
 
+ − = + − ≠ ≠ −
+ + +
3 2
2 3 2 1 0t t t⇔ − + − = 1t⇔ = tan 1
4
x x k
π
π⇔ = ⇔ = +
Bài 2. cos3 cos 2 cos 1 0x x x+ − − =
Gi i:
t t=cosx, pt tr thành:
3 2
4 3 2 1 1 0t t t t⇔ − + − − − =
WWW.MATHVN.COM
6
cos 11
21
cos cos
32
xt
xt
π
= ±= ± 
⇔ ⇔−  ==
 
2
2
3
x k
x k
π
π
π
=
⇔
 = ± +

Bài 3. Gi i phương trình: 1 sin 1 cos 1x x− + − = ( thi d b 2 A – 2004) (1)
Gi i:
(1) 1 sin cos 2 (1 sin )(1 cos ) 0x x x x− − + − − =
t t=sinx +cosx
⇔
2
1
sin
2
t
xcosx
−
=
Pt tr thành:
2
1
1 2 1 0
2
t
t t
−
− + + − = 2 2 2
2 1 4 2 2 4 ( 1) 0 1t t t t t t⇔ − + = + − − ⇔ − = ⇔ =
Sinx+cosx =1 2 sin 1
4
x
π 
+ = 
 
sin sin
4 4
x
π π   
+ =   
   
x kπ=
Bài 4. ( )
2
2cos
sin 6tan 1 sin 2
1 sin
x
x x x
x
+ + − =
+
Gi i:
t t=sinx [ 1;1]t ∈ −
pt tr thành:
( )
2 2
2
2
1
6 1 2 6 1 0
1 1
t t
t t t t
t t
−
+ + − = ⇔ − − =
+ −
2
1 6
1
sin 52
22
1 6
sin sin
3 1
arccos 2
3
x k
t
x
x k
xt
x k
π
π
π
π
α
π

= + =  = ⇔ ⇔ ⇔ = + −  ==  − = +

Bài 5.
6 6 1
sin cos cos8
4
x x x+ = (1)
Gi i:
(1)
23 1 3 1 cos4 1
1 sin 2 cos8 1 cos8
4 4 4 2 4
x
x x x
− 
− = ⇔ − = 
 
t t=cos4x [ 1;1]t ∈ − pt tr thành:
( )2
2
4
3 1 1 16 42 4
1 2 1
3 34 2 4 2 4
4 16 42
k
xt x k
t
t
k
x k xt
π ππ
π
π π π
π
 
= += = + −  − = − ⇔ ⇔ ⇔  
  −  = + = +=  
WWW.MATHVN.COM
7
Bài t p t luy n
1 1
sin 2x sin x 2cot g2x
2sin x sin 2x
+ − − =
2
x3
cos2
42
x
cos
42
x5
sin =




 π
−−




 π
−
2
2cos x 2 3sinx cosx 1 3(sinx 3 cosx)+ + = +
gxcottgx
xsin
x2cos
xcos
x2sin
−=+
( )( )
1
2cos 1 sin sin2 cos2
2
x x x x− + − =
( )( )2sin 1 2cos 1 1x x+ − =
( )3 3
sin cos 2 1 sin cosx x x x+ = −
2sin cos cos 1
2
x
x x− =
4 4 3
sin cos cos .sin 3 0
4 4 2
x x x x
π π   
+ + − − − =   
   
Cho phương trình:
2sin cos 1
sin 2cos 3
x x
a
x x
+ +
=
− +
(2) ( d b kh i a 2002)
1. gi i phương trình khi a=
1
3
2. tìm a phương trình (2) có nghi m.
2
tan cos cos sin 1 tan tan
2
x
x x x x x
 
+ − = + 
 
( )2
4
4
2 sin 2 sin3
tan 1
cos
x x
x
x
−
+ =
WWW.MATHVN.COM
8
Bài IV: Tích Phân
Lưu ý trư c khi gi i thi:
Tích phân là bài toán r t thư ng xu t hi n trong thi i h c. K t năm 2002, khi b t u ti n hành thi
“Ba chung” các d ng toán tích phân và ng d ng luôn xu t hi n và là câu 1 i m. Bài t p ph n này
không quá khó nhưng v n ph i òi h i kĩ năng phán oán, phân tích , và n m rõ ư c các cách làm bài
toán tích phân cơ b n như i bi n s và tính theo tích phân t ng ph n… các em cùng theo dõi các ví d
dư i ây.
NGUYÊN T C CHUNG GI I BÀI TOÁN TÍCH PHÂN:
G m có 2 phương pháp chính:
A. I BI N:
• i bi n lo i 1:
( )( ) ( ). 'f u x u x dx t t=u(x)
Chú ý: Các bi u th c có quan h o hàm
GI I CÁC VÍ D :
VD 1. Tính tích phân:
2
2
0
sin 2
3 cos
x
I
x
π
=
+∫
Gi i:
t
2
3 cost x= + ( )2cos sindt x x dx⇒ = − 2sin 2dt xdx⇒ = −
X
0
2
π
t 4 3
4
3
4 4
ln ln
3 3
dt
I t I
t
−
= = ⇒ =∫
VD2. Tính tích phân:
6
2
dx
I
2x 1 4x 1
=
+ + +
∫ ( DB 1A – 2006)
Gi i:
t t=
2 1
4 1 4 1
2
x t x tdt dx+ ⇒ = + ⇒ =
X 2 6
t 3 5
( )
( ) ( )
5 5 5
2 2
3 3 3
51 1 1 3 1
ln 1 ln
31 1 2 121 1
t dt dt dt
t
t tt t
+ −  
= − = + + = − + + + +
∫ ∫ ∫
VD3. Tính tích phân:
4
2
0 cos 1 tan
dx
I
x x
π
=
+
∫
Gi i:
WWW.MATHVN.COM
9
t t=
2
2
1 tan 1 tan 2
cos
dx
x t x tdt
x
+ ⇒ = + ⇒ =
X
0
4
π
t 1 2
2 2
1 1
2 2
2 2 2 2 2
1
tdt
I dt t
t
= = = = −∫ ∫
VD 4. Tính tích phân:
e
1
3 2ln x
I dx.
x 1 2ln x
−
=
+∫
Gi i:
t t=
2
1 2ln 1 2ln
dx
x t x tdt
x
+ ⇒ = + ⇒ =
X e 1
t 2 1
( )
( )
22 2
2
1 1
3 1 10 2 11
4
3
t
I tdt t dt
t
− − −
= = − =∫ ∫
1. i bi n lo i 2:
B c t l n hơn b c m u: chia a th c
B c t nh hơn b c m u:
Xét quan h o hàm ⇒ i bi n
M u có nghi m⇒ Tách phân th c
Hàm h u t (m u vô nghi m):
( )( )
2 2
du
u x a+
∫ t u(x)=atant
Hàm căn th c:
( )( )
22
a u x+ ⇒ t u(x)=atant
( )( )
22
u xa − ⇒ t u(x)=asint (ho c u(x)=asint)
VD 5. Tính tích phân: I=
3
2
0
9
dx
x +∫
Gi i:
t x=3tan(t)
( )2
3 tan 1dx t dt⇒ = +
X 0 3
t
0
4
π
WWW.MATHVN.COM
10
( )
( )
24
2
0
3 tan 1 1
4
3 129 tan 1 0
t dt
I t
t
π
π
π+
= = =
+∫
VD 6. Tính tích phân:
( )
5
2
2
1 9 1
dx
I
x
=
− −
∫
Gi i:
t x-1= 3sint
3cosdx tdt⇒ =
X
1
5
2
t
0
6
π
6 6 6
2 2
0 0 0
3cos cos cos
6
cos 69 9sin 1 sin 0
tdt tdt tdt
I t
tt t
π π π
π
π
= = = = =
− −
∫ ∫ ∫
VD 7. Tính tích phân:
3
2 2
1 3
dx
I
x x
=
+
∫
Gi i:
t x= 3 tant ( )2
3 tan 1dx x dx⇒ = +
X 1 3
t
6
π
3
π
( )2 3 32
222 2
2 26 6
1
3 tan 1 1 1 coscos
3 3 sinsin 13tan 3tan 3
cos cos
dtt tdttI dx
ttt
t t
π π
π π
+ −
= = =
+
∫ ∫ ∫
( )3
2
6
sin1 1 6 2 33
3 sin 3sin 9
6
d t
I
t t
π
π
π
π
−
= − = − =∫
WWW.MATHVN.COM
11
B. PHƯƠNG PHÁP TÍNH TÍCH PHÂN T NG PH N:
Công th c:
b b
a a
b
udv uv vdu
a
= −∫ ∫ (1)
Cách l y ph n các tích phân:
Kí hi u P(x) là a th c. Khi g p hai d ng nguyên hàm sau ây, ta thư ng dùng phương pháp tích phân
t ng ph n:
D ng 1: ( )lnP x xdx∫ ta t u= ln x (Do lnx không có nguyên hàm)
D ng 2: ( ). sin( )
cos( )
ax b
e
P x ax b dx
ax b
+
 
 
+ 
 + 
∫ ta t u=P(x)
V i cách y khi l y công th c 1 ta s ư c bài toán d n t i nguyên hàm ng d ng v i b c c a P(x)
th p hơn…
GI I CÁC VÍ D :
VD 1. Tính tích phân:
2
0
I (x 1)sin2xdx.
π
= +∫ ( d b kh i D 2005)
Gi i:
t:
( ) 2
0
1
1 1
cos2 cos2 121
2 2 4sin2 cos2
02
u x du dx
x
I x xdx
dv xdx v x
π
π
π
= + ⇒ =
− +
⇒ = + = + −
= ⇒ =
∫
VD 2. Tính tích phân:
2
1
I (x 2)lnx dx.= −∫ ( d b kh i D 2006)
Gi i:
t:
( ) 2
1
ln
2
2
2
du dx
u x x
dv x dx x
v x

== 
⇒ 
= −  = −

22
1
2 5
2 ln 2 ln 4
12 2 4
x x
I x x dx
   
⇒ = − − − = − +   
  
∫
VD 3. Tính tích phân:
2
4
0
sin xdx
π
∫
Gi i:
t t=
2
2x t x tdt dx⇒ = ⇒ =
X
0
2
4
π
t
0
2
π
WWW.MATHVN.COM
12
2
0
2 sinB t tdt
π
= ∫
Tính
2
0
sinI t tdt
π
= ∫
t:
sin cos
u t du dt
dv tdt v t
= = 
⇒ 
= = − 
2
0
cos cos cos 0cos0 sin 12 2
2 2
0 0
I t t tdt t
π
π π
π π
= − + = − + + =∫
B=2I=2
VD 4. Tính tích phân: A=
2
0
cosx
e xdx
π
∫
Gi i:
t:
sin cos
x x
u e du e dx
dv xdx v x
 = =
⇒ 
= − = − 
2 2 2
02
0 0 0
cos cos cos cos0 cos 1 cos2
2
0
x x x x
A e x e xdx e e e xdx e xdx
π π π
ππ
π
= − + = − + + = +∫ ∫ ∫ (1)
Tính
2
0
cosx
K e xdx
π
= ∫
t:
cos sin
x x
u e du e dx
dv xdx v x
 = =
⇒ 
= = 
2
2
0
sin sin2
0
x x
K e x e xdx e A
π
ππ
= − = −∫
Thay vào (1):
2
2 2
1
1 2 1
2
e
A e A A e A
π
π π
+
= + − ⇒ = + ⇒ =
VD 5. Tính tích phân: A=
2
0
sin cosx x xdx
π
∫
Gi i:
t: 22
sin cossin cos
du dxu x
v x xdxdv x xdx
== 
⇒ 
==  ∫
Tính:
2
sin cosv x xdx= ∫
t : cos sint x dt xdx= ⇒ = −
WWW.MATHVN.COM
13
V=
3 3
2 cos
3 3
t x
t dt C C
−
− = + = − +∫
Ch n C=0
3
cos
3
x
v⇒ = −
V y
3
3
0
cos 1 1
cos
03 3 3 3
x
A x xdx K
π
π π
= − + = +∫ (1)
Tính ( )3 2
0 0
cos 1 sin cosK xdx x xdx
π π
= = −∫ ∫
t t=sin(x) cosdt xdx⇒ =
X 0 π
t 0 0
( )
0
2
0
1 0K t dt= − =∫
Thay vào (1):
1
3 3 3
A K
π π
= + =
VD 6. Tính tích phân:
2
3
sin
1 cos
x x
D dx
x
π
π
+
=
+∫
Gi i:
2
2
3
sin
2cos
2
x x
D
x
π
π
+
= ∫ t:
( )
2
sin
1 cos
1
tan
2cos 2
2
u x x
du x dx
dv dx x
x v
= +
 = + 
⇒ =
= 

V y: ( ) ( )
2
3
3 32
sin tan 1 cos tan 1
2 2 2 3 2 3
3
x x
D x x x dx K
π
π
π
π π
π
  
= + − + = + − + −       
∫ (3)
V i: ( )
2 2 2
2
3 3 3
1 cos tan 2cos tan sin
2 2 2
x x x
K x dx dx xdx
π π π
π π π
= + = =∫ ∫ ∫
12
cos
2
3
x
π
π
= − =
Thay vào (3) ta có: D=
( )9 2 3
18
π+
L i bình: tích phân t ng ph n ta có cách nh t u như sau: nh t “log” – nhì “ a” ( a th c) – tam
“Lư ng” (Lư ng giác) – T “mũ”. Trong phép tính tích phân t ng ph n, g p phép nào ng trư c trong 4
phép trên, hãy t u b ng phép ó!
WWW.MATHVN.COM
14
Bài t p t luy n
Tính tích phân:
3
2
0
sin .I x tgxdx
π
= ∫
Tính tích phân:
7
3
0
2
1
x
I dx
x
+
=
+
∫
Tính tích phân:
2
0
ln
e
I x xdx= ∫
Tính tích phân:
4
sin
0
( cos )x
I tgx e x dx
π
= +∫
Tính tích phân:
0
cos sinI x xdx
π
= ∫
Tính tích phân:
3
2 2
6
tan cot 2I x x dx
π
π
= + −∫
Tính tích phân: ( )
2
2
2 1 cos2I x dx
π
π−
= +∫
Tính tích phân:
3
6
sin 4 sin3
tan cot 2
x x
I dx
x x
π
π
=
+∫
Tính tích phân:
10
5
dx
I
x 2 x 1
=
− −
∫
Tính tích phân:
e
1
3 2ln x
I dx.
x 1 2ln x
−
=
+∫
Tính tích phân: 2
0
sin
1 sin
x x
I
x
π
=
+∫
Tính tích phân:
36
0
sin sin
cos2
x x
I
x
π
+
= ∫
Tính di n tích hình ph ng gi i h n b i parabol ( ) 2
P : y x x 3= − + và ư ng th ng
d : y 2x 1.= +
Tính di n tích hình ph ng gi i h n b i các ư ng: ( ) ( ) ( )
2
2 27
1 ; 2 ; 3
27
x
C y x C y C y
x
= = =
WWW.MATHVN.COM
15
Bài V:Các bài toán liên quan n ng
d ng c a o hàm và th hàm s .
Lưu ý trư c khi gi i thi:
Các bài toán d ng này là câu chi m 1 i m, thư ng n m câu th 2 sau ph n kh o sát hàm s trong
thi i h c. Mu n gi i ư c d ng toán này ta c n n m v ng các lí thuy t v s tăng, gi m hàm s , các
v n v c c tr , s tương giao gi a hai th ( i u ki n ti p xúc c a hai ư ng cong)… Các ví d dư i
ây s trình bày m t cách có h th ng các v n nêu trên và cách gi i ơn gi n và d hi u nh t. Các
b n tham kh o các ví d sau ây:
I: S TĂNG GI M C A HÀM S :
Nh c l i ki n th c:
Cho hàm s ( )y f x= có o hàm trên mi n I
( ) 0;f x x I≥ ∀ ∈ Hàm s tăng
( ) 0;f x x I≤ ∀ ∈ Hàm s gi m
VD 1. Cho hàm s : ( ) ( )3 2 21
2
3
y f x x mx m m x= = − + + −
Tìm m hàm s :
a. Tăng trên R
b. Gi m trên (0;2)
c. Tăng trên ( )4;+∞
d. Gi m trên o n có dài b ng 2
e. Tăng trên 2 kho ng ( );4−∞ và ( )2;+∞
Gi i:
TX : D R=
2 2
' 2 2 ' 2y x mx m m m= − + + − ⇒ ∆ = − +
a. Ycbt ' 0 2 0 2m m∆ ≤ ⇔ − + ≤ ⇔ ≥
b. Ycbt
( )
( )
2
2
' 0 0 2 0
1
' 2 0 3 2 0
y m m
m
y m m
 ≤  + − ≤ 
⇔ ⇔ ≤ 
≤ − + ≤ 
Vì
c. Ycbt
TH1: ' 0 2 0 2m m∆ ≤ ⇔ − + ≤ ⇔ ≥
x -∞ 0 2 +∞
F’(x) + - +
F(x)
WWW.MATHVN.COM
16
TH2: ( ) 2
2' 0
' 4 0 9 14 0
4
4
2
m
y m m
mS

 <∆ > 
 
≥ ⇔ + + ≥ 
  < − <

V y ycbt
( ); 7
2
m
m
 ∈ −∞ −

≥
d. Ycbt 1 2
2
2 2 2 2 2 2 1 1x x m m m
a
∆
⇔ − = ⇔ = ⇔ − + = ⇔ − + = ⇔ =
Chú ý:
X1=
'b
a
− + ∆
; x2=
'b
a
− − ∆
1 2x x⇒ − =
2
a
∆
e. Ycbt
( )
( )
2
2
' 0
2
' 0
2 0
2' 4 0
9 14 0
2 1' 2 0
3 2 0
4 24 2
2
m
m
my
m m
my
m m
S
m
∆ ≤
≥
∆ >  − + >  ≥≥  ⇔ ⇔ ⇔+ + ≥   − ≤ ≤  − ≥
− + ≥ 
  − < <− < < 
VD 2. Cho hàm s ( )
2
2 2 21
3 3
m
y x mx m m x
−
= + + − + tìm m hàm s :
a. Gi m trên mi n xác nh.
b. Tăng trên (0;2)
c. Gi m trên ( )6;+∞
d. Tăng trên o n có dài b ng 2
e. Gi m trên 2 kho ng ( );0−∞ và ( )6;+∞
Gi i:
MX : D=R
2 2
' 2y x mx m m= − + + −
' m∆ =
a. Gi m trên mi n xác nh.
' 0 0m⇔ ∆ ≤ ⇔ ≤
b. Tăng trên (0;2)
( )
( )
2
2
' 0 0 0
1
' 2 0 5 4 0
y m m
m
y m m
 ≥ − + ≥ 
⇔ ⇔ ⇔ = 
≥ − + + ≥ 
c. Gi m trên ( )6;+∞
TH1: ' 0 0m∆ ≤ ⇒ ≤ (Rõ ràng vì gi m trên D cũng có nghĩa là gi m trên ( )6;+∞ )
WWW.MATHVN.COM
17
TH2: ( ) 2
0' 0
' 6 0 13 36 0
6
6
2
m
y m m
mS

 >∆ > 
 
≤ ⇔ − + − ≤ 
  < <

V y YCBT
[ ]
0
4
0;4
m
m
m
≤
⇔ ⇔ ≤
∈
d. Tăng trên o n có dài b ng 2
1 2
2 '
2 2 2 2 1x x m m
a
∆
⇔ − = ⇔ = ⇔ = ⇔ =
e. Gi m trên 2 kho ng ( );0−∞ và ( )6;+∞
TH1: (Gi m trên D):
' 0 0m∆ ≤ ⇔ ≤
TH2:
( )
( )
' 0
' 0 0
1 4' 6 0
0 6
2
y
my
S
∆ ≥

≤
⇔ ≤ ≤ ≤

 < <

Tóm l i: ycbt
0
1 4
m
m
≤
 ≤ ≤
II: C C TR C A HÀM S
Nh c l i ki n th c:
X
X0
Y’ + 0 -
Y
C c i
X
X0
Y’ - 0 +
Y
C c Ti u
Bài 1: Cho (Cm) ( )3 2 2 31
2 1
3
y x mx m x m m= − + − + − . Tìm m :
a. Tìm m C có i m c c i n m trên Oy
b. Hàm s t C và CT t i i m có hoành <1
WWW.MATHVN.COM
18
c. Hàm s t C và CT t i i m có hoành >-1
d. Hàm s t C và CT t i i m có hoành n m trong [-2;3]
e. Hàm s t C và CT t i i m có hoành dương
f. Hàm s t C và CT t i i m có hoành trái d u nhau
g. Hàm s t C và CT t i x1;x2 sao cho ( )3 3
1 2x x+ nh nh t
Gi i:
MX : D=R
2 2
' 2 2 1y x mx m= − + −
2
' 1m∆ = − +
' 0∆ > :
X
−∞ X1 X2 +∞
Y’ + 0 - 0 +
Y
C
CT
a. Ycbt Hàm s t c c i t i x=0
( ) 2' 0 0
2 1 0 2
200
2
y
m
mS
m
 =
 − =
⇔ ⇔ ⇔ = 
>< 

b. Ycbt :
( )
2
2
1
1 0' 0
0
' 1 0 2 2 0
1
1
1 1
2
m
m
m
y m m
m
mS
m
  <
 − + >∆ > 
 < 
⇔ > ⇔ − > ⇔   >  <
 < <
⇒ 1 0m− < <
c. Ycbt Hàm s t C và CT t i i m có hoành >-1
( )
2
2
1
1' 0
0
' 1 0 2 2 0
1
1
1 1
2
m
m
m
y m m
m
mS
m
  <
  <∆ > 
 > 
⇔ − > ⇔ + > ⇔ ⇔   < −  > −
 > − > −
0 1m< <
d. Hàm s t C và CT t i i m có hoành n m trong [-2;3]
Ycbt
( )
( )
( )
( )
2
2
' 0
1
' 2 0
2 4 3 0
1 1' 3 0
2 6 8 0
2 3 2 3
2
m
y
m m m
my
m m m
S
m
∆ >  < − ≥ + + ≥ ∀
⇔ ⇔ ⇔ − < < ≥
− + ≥ ∀ 
 − ≤ ≤ − ≤ ≤
e. Hàm s t C và CT t i i m có hoành dương
WWW.MATHVN.COM
19
Ycbt ( ) 2
1 1
21 1' 0
22' 0 0 2 1 0 1
22
0
0 2
2
0
m
m m
y m m
m mS
m
− < <
 
 − < <∆ >  ≤ −  ⇔ ≥ ⇔ − ≥ ⇔ ⇔ ≤ <  
  > ≥ < 
 
>
f. Hàm s t C và CT t i i m có hoành trái d u nhau
( ) 2
' 0 0 2 2
2 1 0
2 2' 0 1
y
m m
m
 < −
⇔ ⇔ − < ⇔ < <
∆ > ⇒ <
g. Hàm s t C và CT t i x1;x2 sao cho ( )3 3
1 2x x+ nh nh t
Ycbt
( ) ( )
3
1 2 1 2 1 2
' 0
3 minP x x x x x x
∆ >
⇔ 
= + − + →
(1)
V i
2
1 2
1 2
2 1
2
x x m
x x m
 = −

+ =
V y ta có (1)
( ) ( )
2
3 2
1 0
2 3 2 1 .2 min
m
P m m m
− + >
⇔ 
= − − →
3
1 1
4 6 min
m
P m m
− < <
⇔ 
= + →
2
2
2
' 12 6 ' 0
2
2
m
P m P
m

=
⇒ = − + ⇒ = ⇔

= −

B ng bi n thiên:
X
−∞ -1
2
2
−
2
2
1 +∞
Y’ - 0 + 0 -
Y
-2 2 2
- 2 2 2
min 2 2P = − khi
2
2
m
−
=
L i bình:
Có l các b n ang th c m c: “T i sao l i có nh ng l i gi i ng n g n và d dàng như v y?” Bí quy t n m
bi u th c y’ và d u c a nó. Lúc này, t t c yêu c u bài toán (ycbt) liên quan n c c tr u n m n dư i
nh ng d u + - c a y’. Và tr c quan hơn n a, ta th y ư c hư ng i c a mình qua b ng bi n thiên. Tôi s
minh h a kĩ câu d c a ví d trên ây:
Ycbt : Hàm s t C và CT t i i m có hoành n m trong [-2;3]
- có c c i và c c ti u y’=0 có hai nghi m ' 0⇒ ∆ >
- V b ng bi n thiên:
WWW.MATHVN.COM
20
X
−∞ -2 X1
2
S
X2 3 +∞
Y’ + 0 - 0 +
Y
C
CT
T ó ta có
( )
( )
' 2 0
' 3 0
y
y
 − ≥

≥
. V y là i u ki n th 2 ã ư c bi u hi n r t rõ ràng trên b ng bi n
thiên. ây th c ra là xét quan h v d u c a h s a: ( )af α nhưng ây khi ta ã bi t rõ d u
c a a thì ch c n t d u ó vào trư c ( )f α là ư c. ây cũng có th là bư c rút g n th i
gian mà các em nên làm, tránh khai tri n m t th i gian.
-
2
S
là t ng hai nghi m X1;X2 c a phương trình y’=0 hay b ng
2
b
a
−
. Rõ ràng n u X1;X2 n m
trong [-2;3] thì
2
S
cũng ph i n m trong o n này. Vì
2
b
a
−
là giá tr có th rút ra d dàng t
phương trình g c nên ta ch n giá tr trung bình này làm i u ki n. Nút th t th 3 ư c g
b .
- L i khuyên ó là: khi g p nh ng d ng toán như trên h c sinh hãy v b ng bi n thiên như
trên ra gi y nháp sau ó tùy theo câu h i mà i n các thông s thích h p vào b ng. t ó
m i hư ng gi i u ư c phơi bày!
Tôi có tham kh o qua m t vài tài li u c a các th y cô giáo thì th y ph n l n các sách u trình bày l i
gi i m t cách máy móc, không tr c quan, nhi u lúc có th coi là lu n qu n. . Ví d : tìm m hàm s
y=f(x) tăng trên (1;+ ∞), các th y cô trình bày trong sách cũng như trên l p theo phương pháp Min-
Max, xét nhi u trư ng h p… Nh ng cách gi i ó không ph i là sai tuy nhiên i u ó ôi khi làm khó các
em h c sinh trong quá trình tư duy tìm trư ng h p, nh t là các em h c sinh trung bình. Phương pháp
xét d u trình bày trên ây v a ng n g n rõ ràng l i không b sót trư ng h p. bài toán ư c ơn gi n
hóa.
Cách gi i trên cũng áp d ng ư c cho hàm s
2
2
' ' '
ax bx c
y
a x b x c
+ +
=
+ +
vì d ng o hàm
( )
2
22
2
' ' ' ' ' '
'
' '
a b a c b c
x x
a b a c b c
y
a x b x c
+ +
=
+ +
. Trong trư ng h p này, tùy bi u th c m u có nghi m hay
không ta t thêm trư ng h p. Vì m u th c ≥0 nên khi xét d u ta ch c n xét d u t s tương t
như các ví d trình bày trên.
D ng hàm s này ã không còn thông d ng ( ch gi i thi u sơ lư c trong sách giáo khoa) nên xu
hư ng ra ch xoay quanh 3 hàm là: b c 3, trùng phương và
' '
ax b
y
a x b
+
=
+
.
Bài 2: Cho (Cm): ( )3 2
3 3 1 4y x mx m x= − + − +
nh m :
a. C(m) có hai i m c c tr A;B sao cho AB th ng hàng v i C(1;-1)
b. C(m) có hai i m c c tr A;B sao cho AB = 2 5
c. C(m) có hai i m c c tr A;B sao cho AB cách u : 2y∆ =
Gi i:
WWW.MATHVN.COM
21
MX : D=R
T a 2 i m c c tr th a h :
' 0
( )
y
y f x
=

=
V y:
2
' 2 1 0y x x m= − − + =
( )3 2
3 3 1 4y x mx m x= − + − + ( )( )2
0
2 1y x x m cx d ax b ax b⇒ = − − + + + + = +
( )2
2 1 ( 1) 2 5y x x m x mx m⇔ = − − + − − − +
( )
( )
2
2 1 0 1
2 5 2
x x m
y mx m
 + − + =
⇔ 
= − − +
C(m) có hai c c tr (1) ph i có 2 nghi m phân bi t ' 0⇒ ∆ ≥ 0m⇒ >
a. C(m) có hai i m c c tr A;B sao cho AB th ng hàng v i C(1;-1)
(2) ⇒ phương trình ư ng th ng qua hai i m c c tr là 2 5y mx m= − − +
Vì AB th ng hàng v i C(1;-1) ⇒ C ∈ AB nên: -1=-2m.1-m+5 2m⇔ =
V y v i m=2 AB th ng hàng v i C(1;-1)
b. C(m) có hai i m c c tr A;B sao cho AB = 2 5
( ) 2 1
2 '
1 2x x m
a
∆
⇒ − = =
( ) ( )2 1 2 12 2 4y y m x x m m⇒ − = − − = − ( ) ( )
2 2
2 1 2 1 2 5AB x x y y⇒ = − + − =
2
1
16 4 20 5
4
m
m m
m
=
⇒ + = ⇔
 = −

So sánh k ⇒ 1m =
c. C(m) có hai i m c c tr A;B sao cho AB cách u : 2y∆ =
Ycbt ( ) ( ); ;d A d B⇔ ∆ = ∆ v i : 2y∆ =
( )
1 2 1 2
1 2
1 2 1 2
2 2
2 2
2 2 4
y y y y
y y
y y y y
− = − =
⇔ − = − ⇔ ⇔ − = − − + =
( ) ( ) ( )1 2 1 22 5 2 5 4 2 2 10 4mx m mx m m x x m⇔ − − + + − − + = ⇔ − + − + =
2 .2 2 10 4 1m m m⇔ − − + = ⇔ =
Bài 3: Cho (Cm): ( )3 2
3 3 1y x x m x= + − −
nh m :
a. C(m) có hai i m c c tr A;B sao cho OAB∆ vuông t i O
b. C(m) có hai i m c c tr A;B n m khác phía v i tr c Ox
c. C(m) có hai i m c c tr A;B cùng phía v i tr c Oy
d. C(m) có hai i m c c tr A;B n m cách u ư ng th ng y=5
e. Có ư ng th ng i qua hai i m c c tr cách g c t a m t kho ng b ng 1
f. Có ư ng th ng i qua hai i m c c tr ti p xúc v i ư ng tròn ( ) ( )
2 2
1 1 4x y− + − =
g. Có ư ng th ng i qua hai i m c c tr t o v i hai tr c t a m t tam giác cân
h. Có ư ng th ng i qua hai i m c c tr t o v i hai tr c t a m t tam giác có di n tích =8
Gi i:
WWW.MATHVN.COM
22
CT
CD
x
y
1x
1x
5y =
MX : D=R
T a 2 i m c c tr th a h :
' 0
( )
y
y f x
=

=
( ) ( )( )
2
3 2 2
'
2 1 0
3
3 3 1 2 1 1 2 1
y
x x m
y x x m x x x m x mx m

= + − + =
⇔ 
 = + − − = + − + + − + −
( )
( )
2
2 1 0 1
2 1
x x m
y mx m
 + − + =
⇔ 
= − + − ∆
C(m) có hai c c tr (1) ph i có 2 nghi m phân bi t ' 0⇒ ∆ ≥ 0m⇒ > (*)
a. C(m) có hai i m c c tr A;B sao cho OAB∆ vuông t i O
Ycbt OA OB⇔ ⊥
.OAOB⇔ v i
( )
( )
;
;
A A
B B
OA x y
OB x y
 =

=
( )( )1 2 1 2 1 2 1 20 2 1 2 1 0x x y y x x mx m mx m⇔ + = ⇔ + − + − − + − =
( )( ) ( )
22 2
1 2 1 2 1 24 2 2 1 0x x m x x m m x x m⇔ + + − + + + − =
⇔ ( ) ( ) ( ) ( )
22 2
1 4 1 2 2 . 2 1 0m m m m m m− + + − + + − + − + − =
3 2
4 9 7 2 0m m m⇔ − + − + = ( )( )2
vì 7
4 5 2 1 0
VN
m m m
∆=−
⇔ − + − − = 1m⇔ = (th a i u ki n(*))
b. C(m) có hai i m c c tr A;B n m khác phía v i tr c Ox
Ycbt 1 2. 0y y⇔ < ( )( )1 22 1 2 1 0mx m mx m⇔ − + − − + − <
( )( ) ( )
22 2
1 2 1 24 2 2 1 0m x x m m x x m⇔ + − + + + − <
( ) ( ) ( )
22 2
4 1 2 2 2 1 0m m m m m⇔ − + − − + + − <
( )( )
23 2
0
4 9 6 1 0 4 1 1 0m m m m m
≥
⇔ − + − + < ⇔ − + − <
1
4
1
m
m

>
⇔ 
 ≠
c. C(m) có hai i m c c tr A;B cùng phía v i tr c Oy
Ycbt 1 2 0x x⇔ > ( 1x cùng d u v i 2x ) 1 0 1m m⇔ − + > ⇔ <
d. C(m) có hai i m c c tr A;B n m cách u ư ng th ng y=5
Ycbt : y=5 c t (Cm) t i trung i m AB. M là trung i m AB có t a 1 2
; 2 1
2
x x
mx m
+ 
− + − 
 
( )1;3 1M m⇒ − − 5 3 1 2Ycbt m m⇔ = − ⇔ =
So sánh v i i u ki n (*) ta th y m=2 là k t qu c n tìm.
e. Có ư ng th ng i qua hai i m c c tr cách g c t a m t kho ng b ng 1
: 2 1 : 2 1 0y mx m mx y m∆ = − + − ⇔ ∆ + − + =
WWW.MATHVN.COM
23
Ycbt ( ); 1d O⇔ ∆ =
( )
2 2
2 .0 0 1
1
2 1
m m
m
+ − +
⇔ =
+
( ) ( )
2 2 2 2
1 2 1 3 2 0m m m m⇔ − + = + ⇔ + =
0
2
3
m
m
=
⇔ −
 =

So sánh v i i u ki n m>0 ta nh n th y không có giá tr m th a mãn yêu c u bài toán.
f. Có ư ng th ng i qua hai i m c c tr ti p xúc v i ư ng tròn ( ) ( )
2 2
1 1 4x y− + − =
Ycbt ( );d I R⇔ ∆ = v i tâm I(1;1) và R=2
: 2 1 0mx y m∆ + + − =
( )
2
2 .1 1 1
2
2 1
m m
m
+ − +
⇒ =
+
( )
2 2 2
2 16 4 15 4 0m m m m⇔ + = + ⇔ − + =
0
4
15
m
m
=
⇔
 =

So sánh v i (*) ta nh n
4
15
m =
g. Có ư ng th ng i qua hai i m c c tr t o v i hai tr c t a m t tam giác cân
G i M là giao i m c a ∆ và Ox:
2 1 0 1
;0
0 2
mx m m
M
y m
− + − = − 
⇒ ⇒  =  
G i N là giao i m c a ∆ và Oy: ( )
2 .0 1
0; 1
0
y m m
N m
x
= − + −
⇒ ⇒ −
=
Ycbt
1 1
1 1 . 1 0
2 2
M N
m
x y m m
m m
 −
⇔ = ⇔ = − ⇔ − − =  
 
1
1
2
1
2
m
m
m

 =

⇔ =

 −
 =

D th y v i m=1, ∆ i qua g c t a , v i m=
1
2
−
không th a (*) nên lo i. V y ta ch n
1
2
m =
h. Có ư ng th ng i qua hai i m c c tr t o v i hai tr c t a m t tam giác có di n tích =8
Ycbt:
1 1 1
.
2 8 2
OMN M NS OM ON x y∆⇔ = ⇔ =
( )
2
11 1 1
. 1
4 2 4 2
mm
m
m m
−−
⇔ = − ⇔ =
( )
2
2
2
2 1 1
2
2
2 1
2
m
m
m m
m
m
m m VN
 =
 − + = ⇔
  =
⇔  
 −
 − + =

So sánh (*) v y có hai giá tr m th a mãn: m=2 và m=0.5
WWW.MATHVN.COM
24
III: S TƯƠNG GIAO GI A HAI TH
Nh c l i ki n th c:
Cho: ( ) ( )1 2: ; :C y f x C y g x= =
S giao i m c a C1 và C2 là s nghi m c a phương trình hoành giao i m:
( ) ( )f x g x=
c bi t khi C1 ti p xúc C2:
( ) ( )
( ) ( )' '
f x g x
f x g x
=

=
Lưu ý: Không ư c s d ng i u ki n nghi m kép làm d ng toán ti p xúc c a hai th .
hi u rõ hơn, ta hãy n v i các ví d sau:
Bài 1: Cho hàm s ( ) ( )
2 3 2
: 2
1
m
mx m
C y m
x
− −
= ≠ −
−
và ( ): 1d y x= −
nh m (d) c t (Cm) t i hai i m phân bi t:
a)Có hoành l n hơn -1
b)Có hoành nh hơn 2
c) Có hoành n ng trong kho ng [ ]2;3−
d)Có hoành dương
e)Có hoành trái d u.
Gi i:
Phương trình hoành giao i m gi a (Cm) và d:
( ) ( )22 3 2
1 : 2 1 3 3 0
1
mx m
x g x x m x m
x
− −
= − ⇔ − + + + =
−
x
−∞ 1x
2
S
2x +∞
( )g x + 0 - 0 +
(d) c t (Cm) t i hai i m phân bi t g(x)=0 có hai nghi m phân bi t
( )
2
' 0
1
1 0 2
m
m
g m
 >
∆ > ⇔  < −
 ≠ ⇔ ≠ −
(*)
a)Có hoành l n hơn -1
Ycbt:
( )1 0
1
2
g
S
− >

⇔ 
− <

( ) 6
1 2 1 3 3 0
5
1 1
2
m m m
m
m
−
+ + + + > > 
⇔ 
+ > −  > −
So sánh v i (*) ta k t lu n:
6
1
5
2
m
m
−
< <

>
b)Có hoành nh hơn 2
( ) ( )
2 0
4 4 1 3 3 0 3 0 3
1 11 22
2
g
m m m m
S m mm
>
− + + + > − + < <  
⇔ ⇔ ⇔   
< <+ <<   

WWW.MATHVN.COM
25
So sánh v i (*) ta k t lu n:
2
2 1
m
m
< −
− < < −
c) Có hoành n ng trong kho ng [ ]2;3−
Ycbt:
( )
( )
( )
( )
11
2 0 4 4 1 3 3 0 7
3 0 9 6 1 3 3 0 2
3 22 1 3
2 3
2
mg m m
g m m m
mS m
 
≥ − − ≥ + + + + ≥ 
  
≥ ⇔ − + + + ≥ ⇔ ≤  
  − ≤ ≤− ≤ + ≤ − ≤ ≤

So sánh i u ki n (*) ta suy ra:
11
1
7
m
−
≤ ≤ −
d)Có hoành dương
Ycbt:
( ) 0
3 3 0 1
1 0 10
2
g o
m m
S m m
>
+ > ⇔ > −
⇔ ⇔ 
+ ≥ ⇔ ≥ −≤ 

So sánh v i (*) ta suy ra: m>2
e)Có hoành trái d u.
Ycbt: ( )0 0 3 3 0 1g m m< ⇔ + < ⇔ < −
So sánh i u ki n (*) ( ) ( ); 2 2; 1m⇒ ∈ −∞ − ∨ − −
Bài 2: Cho hàm s ( )
1
:
1
x
C y
x
+
=
−
và ( ): 1d y mx= +
Tìm m d c t (C):
a)T i 2 i m phân bi t n m trên 2 nhánh c a th .
b)T i 2 i m phân bi t n m trên cùng 1 nhánh c a th
Gi i:
Phương trình hoành giao i m c a (C) và d:
( )
1
1 1
1
x
mx x
x
+
= + ≠
−
( ) ( )2
2 0 1g x mx mx⇔ = − − =
a)T i 2 i m phân bi t n m trên 2 nhánh c a th . (Hình 1)
Ycbt: phương trình (1) có hai nghi m phân bi t th a 1 21x x< <
x
−∞ 1x 1
tiem can dung
2x +∞
( )g x Cùng d u m 0 Trái d u m 0 Cùng d u m
( ) ( ). 1 0 2 0 2 0 0m g m m m m m⇔ < ⇔ − − < ⇔ − < ⇔ >
WWW.MATHVN.COM
26
ình1H
ình 2H
ình3H
Lưu ý: Trư ng h p này không c n ph i xét bi t th c ∆ vì khi d c t
C v 2 phía c a ti m c n ng x=1 thì m c nhiên phương trình ã
có 2 nghi m, không c n thi t ph i xét ∆
b)T i 2 i m phân bi t n m trên cùng 1 nhánh c a th
(Hình 2)
Phương trình (1) có hai nghi m phân bi t th a:
1 2
1 2
1
1
x x
x x
< <
 < <
( )
2
0 8 0
. 1 0 2 0
m m
m g m
∆ >  + <
⇔ ⇔ 
> − > 
0
0
8
m
m
m
<

⇔ >
 < −
8m⇔ < −
Bài 3: Vi t phương trình ư ng th ng c t th :
( ) 3
: 3 2C y x x= − + t i 3 i m phân bi t A,B,C sao cho xA=2 và
BC= 2 2 .
Gi i: (hình 3)
2 4A Ax y= ⇒ =
Phương trình ư ng th ng qua A(2;4) là
( ): ( ) : 2 4A Ay k x x y y k x∆ = − + ⇒ ∆ = − +
L p phương trình hoành giao i m c a (C) và ∆ :
( ) ( )3 3
3 2 2 4 3 2 2x x k x x x k x− + = − + ⇔ − − = −
( )3
3 2 2 0x k x k⇔ − + + − = ( )( )2
2 2 1 0x x x k⇔ − + − + =
( ) 2
2
2 1
x
g x x x k
=
⇔ 
= + − +
i u ki n có BC:
Khi ó t a
( ) ( )1 1 2 2; ; ;B x y C x y th a
h :
( )
( )
2
2 1 0 1
2 4 2
x x k
y kx k
 + − + =

= − +
(1) 2 1
2 '
2x x k
a
∆
⇔ − = =
(2) ( )2 1 2 1 2y y k x x k k⇔ − = − =
( ) ( )
2 2
2 1 2 1 2 2BC x x y y= − + − =
3 3
4 4 2 2 4 4 8 0 1k k k k k⇔ + = ⇔ + − = ⇔ =
x
y
x
y
2 2
x
y
( )
' 0 0 0
2 0 4 4 1 0 9
k k
g k k
∆ > > > 
⇔ ⇔  
≠ + − + ≠ ≠  
WWW.MATHVN.COM
27
V y ( ): 1 2 4y x∆ = − +
Bài 3: Cho (C) ( ) 3 2
3 2y f x x x= = − + . Tìm trên ư ng th ng (d):y=-2 nh ng i m mà t ó có th v
ư c n (C) :
a. Ba ti p tuy n phân bi t
b. Ba ti p tuy n phân bi t trong ó có 2 ti p tuy n vuông góc v i nhau
Gi i:
a. Ba ti p tuy n phân bi t
Xét ( ; 2) : 2A a d y− ∈ = − .
Phương trình ư ng th ng ∆ qua ( ; 2)A a − và có h s góc :
( ) ( )2y k x a= − − ∆ .
∆ ti p xúc v i (C) H phương trình sau có nghi m:
( ) ( )
( )
3 2
2
3 2 2 1
3 6 2
x x k x a
x x k
 − + = − −

− =
Thay k t (2) vào 1 ta ư c:
( )( ) ( )3 2 2
3 2 3 6 2 3x x x x x a− + = − − −
( )3 2
2 3 1 6 4 0x a x ax⇔ − + + − =
( ) ( )3
2 2 3 1 2 0x x a x ⇔ − − − + = 
( ) ( ) ( )2
2
2 3 1 2 0 4
x
g x x a x
=
⇔ 
= − − + =
T A k ư c ba ti p tuy n phân bi t n (C)
phương trình (3) có 3 nghi m phân bi t
phương trình (4) có 2 nghi m phân bi t khác 2
( )
( )
( )
( )
2
2
5
0 3 1 16 0 1
*3
2 0 2.2 3 1 .2 2 0 2
g a a a
g a a
∆ > − − > < − ∨ >  
⇔ ⇔ ⇔  
≠ − − + ≠   ≠
b. Ba ti p tuy n phân bi t trong ó có 2 ti p tuy n vuông góc v i nhau
Khi ó phương trình (3) có 3 nghi m phân bi t:
0 1 22; ;x x x= ( v i x1;x2 là hai nghi m c a phương trình g(x)=0) và 3 ti p tuy n ng v i h s góc là:
( ) ( ) ( )2 2
0 1 1 1 1 2 2 2 2' 2 0; ' 3 6 ; ' 3 6k f k f x x x k f x x x= = = = − = = −
Vì 0 0k = nên : Ycbt k1.k2=-1.
( )( ) ( ) ( )2 2 2 2
1 1 2 2 1 2 1 2 1 2 1 23 6 3 6 1 9 2 4 1 **x x x x x x x x x x x x ⇔ − − = − ⇔ − + + = − 
Áp d ng nh lí Viet cho phương trình (4) ta có:
1 2
3 1a
x x
x
−
+ = và 1 2 1x x =
Do ó (**)
3 1
9 1 2 4 1
2
a −  
⇔ − + = −  
  
55
27
a⇔ = (th a i u ki n (*)).
V y i m c n tìm là
55
; 2
27
A
 
− 
 
.
WWW.MATHVN.COM
28
D NG TOÁN: H Ư NG CONG TI P XÚC V I M T Ư NG C NH
Phương pháp:
D ng 1: Cho h ư ng cong ( )mC :y=f(x;m). ch ng minh ( )mC luôn ti p xúc v i m t ư ng (C) c nh .
◊ TH1:
( )mC :y=f(x;m). là hàm a th c.
ưa : ( );y f x m= v d ng: ( ) ( ) ( ): ê 2
n
y ax bm g x n nguy n= ± + + ≥ .
Xét ư ng cong ( ) ( ):C y g x= và ch ng minh h :
( ) ( ) ( )
( ) ( ) ( )
1
' '
n
n
ax bm g x g x
na ax bm g x g x
−
± + + =

± + + =
Có nghi m m∀
◊ TH2:
( )mC :y=f(x;m). là hàm h u t : (D ng t ng quát)
( ∆ ) ti p xúc v i (C) h sau có nghi m
( ) ( )
( )
( ) ( )
0 0
2
1
2
c
ax b k x x y
x d
c
a k x a
x d

+ + = − + +

 − = ≠
+
Gi i hê trên qua 3 bư c:
B1: nhân 2 v c a phương trình (2) cho: x+d
( ) ( )3
c
ax ad k x d
x d
+ − = +
+
B2: (1)-(3):
( )0 0
2c
b ad k x d y
x d
− + = − − +
+
( ) ( )0 0
2
4
c
k x d y ad b
x d
⇔ = − − + + +
+
B3: Thay (4) vào (2) s có 1 phương trình theo k. gi i phương trình này và tìm m sao cho phương trình
úng m∀ .
Lưu ý: cách gi i trên có th áp d ng i v i hàm s
ax b
cx d
+
+
D ng 2: Tìm i u ki n h ư ng cong ti p xúc v i 1 ư ng c nh:
Dùng i u ki n ti p xúc.
II/ M t s ví d :
Bài 1: Cho ( ) ( )3 2 2
: 2 2 1 2mC y x x m x m= + + + + + . Ch ng minh r ng (Cm) luôn ti p xúc v i m t ư ng
cong c nh.
Gi i:
Ta có: ( ) ( )3 2 2
: 2 2 1 2mC y x x m x m= + + + + + ( )
2 3 2
2x m x x x⇔ + + + + +
Xét ư ng cong ( ) 3 2
: 2C y x x x= + + +
( )mC luôn ti p xúc v i (C): h sau có nghi m:
( )
( )
( )
2 3 2 3 2
2 2
2 2
1
2 3 2 1 3 2 1
x m x x x x x x
x m x x x x
 + + + + + = + + +

+ + + + = + +
WWW.MATHVN.COM
29
Ta có: ( )
( )
( )
2
0
1
2 0
x m
x m
 + =
⇔ 
+ =
Rõ ràng v i m i m , h (1) luôn có nghi m x=-m
Vây m∀ , (Cm) luôn ti p xúc v i 1 ư ng cong c nh: ( ) 3 2
: 2C y x x x= + + + .
Bài 2:
Cho ( )
( ) ( )2
2 2 4
:m
m x m m
C y
x m
− − − +
=
−
. Ch ng minh (Cm) luôn ti p xúc v i hai ư ng th ng c nh.
Gi i:
( )
( ) ( )2
2 2 4
:m
m x m m
C y
x m
− − − +
=
−
( )
4
2y m
x m
⇔ = − −
−
(Cm) luôn ti p xúc v i ư ng th ng ( ): y ax b∆ = +
⇔ H phương trình sau có nghi m m∀ :
( ) ( )
( )
( )
( )
2
4
2 1
4
2
m ax b
x m
I
a
x m

− − = + −

 =
−
◊ Nhân 2 v c a phương trình (2) cho: x-m
( ) ( )
4
3a x m
x m
⇒ = −
−
◊ L y (1)-(3):
( ) ( ) ( )
8 8
2 1 2 4m b am a m b
x m x m
−
⇔ − − = + ⇔ = − + +
− −
◊ Thay (4) vào (2):
( ) ( )
2
1 2 16a m b a⇔ − + + =  
( ) ( )( ) ( ) ( )
2 22
1 2 1 2 2 16 0 *a m a b m b a⇔ − + − + + − − =
H (1) có nghi m m∀ ( )*⇔ úng m∀ :
( )
( )( )
( )
2
2
1 0
1
2 1 2 0
2 6
2 16 0
a
a
a b
b b
b a
 − =
 =
⇔ − + = ⇔ 
= ∨ = −
+ − =
V y (Cm) luôn ti p xúc v i 2 ư ng th ng c nh y=x+2 và y=x-6
WWW.MATHVN.COM
30
Bài t p t luy n
1. Cho hàm s ( ) ( )3 2 21 1
1 2
3 3
y x m x m m x= − + + + − . nh m hàm s :
a) Tăng trên R
b) Gi m trên (0;1)
c) Tăng trên (-∞;2)
d) Gi m trên o n có dài b ng 3
e) Tăng trên 2 kho ng (-∞;0) và (2; +∞)
2. Cho hàm s ( ) ( )3 2 2 3
: 3 3 1 1mC y x mx m m x m= + + − + + + + . Tìm m :
a) (Cm) có i m c c i n m trên x=5
b) Hàm s t c c i và c c ti u t i nh ng i m có hoành >1
c) Hàm s t c c i và c c ti u t i x1 và x2 sao cho: 1 2
2 1
14
5
x x
x x
−
+ =
3. Cho hàm s ( ) 3
: 3 2mC y x x= − + .
a) Vi t phương trình ti p tuy n có h s góc nh nh t
b) Vi t phương trình ti p tuy n i qua M(1;0)
c) Tìm trên Ox nh ng i m mà t ó k ư c trên C úng:
◊ m t ti p tuy n ◊ hai ti p tuy n
◊ Ba ti p tuy n ◊ hai ti p tuy n vuông góc v i nhau
d) Tìm trên ư ng th ng x=1 nh ng i m mà t ó k ư c trên C úng:
◊ m t ti p tuy n ◊ hai ti p tuy n
◊ Ba ti p tuy n
e) Tìm trên (C) nh ng i m mà t ó k ư c trên C úng 1 ti p tuy n.
4. Cho hàm s ( ) 4 2
: 2 2 1mC y x mx m= − + − . Tìm m (Cm) c t Ox t i b n di m phân bi t có hoàn l p
thành c p s c ng.
5. Xác nh m phương trình có nghi m duy nh t:
3 2
1 0x mx+ − =
6. Cho hàm s ( ) ( )3 2 2 3
: 3 3 1mC y x mx m x m= − + − − . Tìm m (Cm) c t Ox t i 3 i m phân bi t trong ó
có úng 2 i m có hoành âm.
7. Cho hàm s ( ) ( )3
: 1 1mC y x k x= + + + . Tìm k (Ck) ti p xúc v i ư ng th ng ( ): 1y x∆ = +
8. Cho hàm s ( ) 3 2 3
: 3 4mC y x mx m= − + . Tìm m (Cm) c t ư ng th ng ( ):d y x= t i A,B,C sao cho
AB=BC.
9. Cho hàm s ( )
2 1
:
2
m
x
C y
x
+
=
+
. Ch ng t r ng ư ng th ng y=-x+m luôn luôn c t th t i hai i m
phân bi t AB. Tìm m o n AB ng n nh t.
10. Cho hàm s ( )
( )
( )
2
3 1
: 1m
m x m m
C y
x m
+ − +
=
+
. Trong ó m là tham s khác 0:
a) Tìm nh ng i m mà th không i qua m∀ .
b) Ch ng minh r ng th c a (1) luôn ti p xúc v i 2 ư ng th ng c nh.
11. Cho hàm s ( ) ( ) ( ) ( ) ( )3 2
: 3 3 1 6 1 1 1mC y m x m x m x m= + − + − + + + . Ch ng minh r ng h th (Cm)
luôn luôn i qua 3 i m c nh th ng hàng.
WWW.MATHVN.COM
31
Bài VI: M t s d ng toán khác c n lưu ý.
I/ Gi i h n:
D ng toán này ã t ng xu t hi n trong thi i h c t r t lâu (năm 2002 – 2003) Tuy nhiên ã r t lâu
không th y xu t hi n trong thi i h c. Tuy nhiên ta cũng nên chú ý n d ng toán này.
âu tôi xin trình bày phương pháp t ng quát làm bài d ng này là “ G i s h ng v ng b ng h s b t
nh”.
Bài 1. Tìm
33 2
21
5 7
lim
1x
x x
x→
− − +
−
Gi i:
Ta có: ( )
3 33 2 3 2
2 2 21 1
5 7 5 2 7 2
lim lim 1
1 1 1x x
x x x x
x x x→ →
 − − + − − + −
= − 
 − − − 
( )( )
3 3
21 1 2 3
5 2 1
lim lim
1 1 5 2x x
x x
x x x→ →
− − −
=
− − − +
=
( )
( )( )
( )
2
1 3
1 3
lim 2
81 5 2x
x x
x x→
− + + −
=
+ − +
( ) ( )
3 2 2
21 1 2 32 2 23
7 2 1
lim lim
1
1 7 2 7 4
x x
x x
x
x x x
→ →
+ − −
=
−  
− + + + + 
 
=
( )
( )21 32 23
1 1
lim 3
127 2 7 4
x
x x
→
=
+ + + +
Thay (2),(3) vào (1) có:
3 1 11
8 12 24
A
−
= − =
Lưu ý:
Trong l i gi i ta ã thêm s 2 vào t th c f(x). Có l b n ang t h i:
● T i sao ph i thêm s 2 ?
● Làm cách nào nh n ra s 2 ?
S 2 là h ng t ã b xóa! Mu n làm d ng bài này, ta ph i khôi ph c nó. Mu n khôi ph c s 2 này ta
làm như sau:
B1: c R∀ ∈ luôn có: ( )
33 2
2 2
5 7
1 1
x c x c
f x
x x
 − − + −
= − 
 − − 
B2: Trong các s c ó. Ta tìm s c sao cho x2
-1 có cùng nhân t chung v i ( ) 3
1 5f x x c= − − và
( ) 3 2
2 7f x x c= + − . i u ó x y ra khi và ch khi c là nghi m c a tuy n:
( )
( )
( )
( )
1
2
1
2
1 0
2
1 0
26
1 0
2
1 0
f
c
f
cc
f
c
f
 =
 =
= 
⇔ ⇔ == 
− =   =  − =
ó chính là lí do t i sao 2 xu t hi n trong bài gi i.
ây là vi c nên làm trong gi y nháp. Không nh t thi t trình bày trong bài làm.
Qua ví d trên ta nêu lên thu t toán sau:
Gi s ( )
( )
( )
f x
F x
g x
= có gi i h n
0
0
WWW.MATHVN.COM
32
B1: Phân tích ( )
( )
( )
( )
( )
1 2f x c f x c
f x
g x g x
+ −
= + .
B2: (Tìm c): G i ( )1;2;...i iα = là nghi m c a h g(x)=0
Khi ó c là nghi m c a h :
( )
( )
( )1
1
0
1;2;...
0
i
i
f c
i
f c
α
α
+ =
=
− =
V i c tìm ư c thì
( )
( )
1
lim
ix
f x c
g xα→
+
và
( )
( )
2
lim
ix
f x c
g xα→
−
s ho c là d ng xác nh ho c là d ng quen thu c.
Sau khi tìm c, vi c trình bày l i gi i như ã làm.
BÀI T P ÁP D NG:
A=
3 2 2
0
3 1 2 1
lim
1 cosx
x x
x→
− + +
−
( d b 2002)
B=
3
20
1 2 1 3
lim
x
x x
x→
+ − +
II/Phương trình và b t phương trình mũ và logarit:
ây là d ng toán cũng r t thư ng xuyên xu t hi n trong thi. Nhìn chung, d ng toán này không
khó. T t c các phép bi n i ch xoay quanh các công th c ã nêu trong sách giáo khoa. ph n
này, tôi không nêu l i các công th c trên. Xin trình bày cách gi i c a 1 s thi g n ây.
Bài làm qua 2 bư c:
B1: t i u ki n. (N u i u ki n quá ph c t p thì có th n bư c 2 r i th nghi m vào i u ki n)
B2: Bi n i phương trình hay b t phương trình v d ng ơn gi n cùng cơ s c 2 v :
• Mũ: Chia
• Logarit:
log
log
log
b
a
b
x
x
a
=
log logn
n
aa
m
x x
n
=
• t n ph : ( )logat f x= phương trình h u t ho c phương trình mũ
( )f x
t a= phương trình h u t .
• Phương pháp hàm s
Bài 1. ( )
22 2 2 3 1
2 3 1 2 3 1
81.4 78.6 16.9 0 1
x x
x x x x − +
− + − +
− + ≤
Gi i:
( )
2 2
2 3 1 2 3 1
6 9
1 81 78 16 0
4 4
x x x x− + − +
   
⇔ − + ≤   
   
( )2 2
2 3 1 2. 2 3 1
3 3
81 78 16 0
2 2
x x x x− + − +
   
⇔ − + ≤   
   
t
2
2 3 1
3
2
x x
t
− +
 
=  
 
k: t>0
Phương trình tr thành:
2
16 78 81 0t t− + ≤
3 27
;
2 8
t
 
⇔ ∈  
2
2 3 1
23 3 27
1 2 3 1 3
2 2 8
x x
x x
− +
 
⇔ ≤ ≤ ⇔ ≤ − + ≤ 
 
WWW.MATHVN.COM
33
2 2
2 2
3
2
02 3 1 1 2 3 0
12 3 1 3 2 3 2 0
2
2
x
xx x x x
x x x x
x
x

≥

≤ − + ≥ − ≥   
⇔ ⇔  
− + ≤ − − ≤     ≤
 ≥
2
1
2
x
x
≥
⇔
 ≤

Bài 2. Gi i b t phương trình:
1 1 1
1x x x
e e x+ − + −
− ≤ −
Gi i:
t:
1
1
1 1
u x x
u v x
v x
 = + −
⇔ − = −
= + −
Phương trình tr thành:
u v
e e u v− = −
( ) ( )f u f v⇔ ≤
V i ( ) ; 1x
f x e x x= − ≥
( )' 1 0x
f x e⇒ = + > ⇒ ( )f x tăng.
Do ó u v≤ 1 1 1 1x x x x⇔ + − ≤ + − ⇔ ≤ −
Bài 3. Gi i phương trình: ( )2 3log 1 logx x+ =
Gi i:
t 3log 3t
x t x= ⇔ =
Do ó: ( ) ( )2log 1 1 2 1 3 2
t
t t
x t x+ = ⇔ + = ⇔ + =
22
1 3 1 3 1 3
1
2 2 2 2 2 2
t tt t
          
+ = ⇔ + = +                         
( ) ( )2f t f⇔ = 2t⇔ = (Vì ( )
1 3
2 2
tt
f x
  
= +        
là hàm gi m)
2 9t x⇔ = ⇔ =
Bài 4. Gi i b t phương trình: ( ) ( )1
2log log 4 8 1 1x
x
+
 − ≥ 
Gi i:
K:
( )
( )2 11 3 5
4 8 0 2 2 2 1 3
2
xx
x x
−−
− > ⇔ > ⇔ − > ⇔ >
( ) ( )1
21 log log 4 8 logx
x x x+
 ⇔ − ≥  ( ) ( )1 1
2 2 2log 4 8 log 4 8 log 2x x x
x− −
⇔ − ≥ ⇔ − ≥
( )1
2 04
4 8 2 2 8 0 3
4 2 8
xx
x x x
x
loai
x−
 ≤
⇔ − ≥ ⇔ − − ≥ ⇔ ⇔ ≥
≥
Bài 5. Gi i h phương trình:
( )
( ) ( )2 3
9 3
1 2 1 1
3log 9 log 3 2
x y
x y
 − + − =

− =
( H A 2005)
WWW.MATHVN.COM
34
Gi i:
k:
1
0 2
x
y
≥

< ≤
( ) ( )3 3 3 32 3 1 log 3log 3 log logx y x y x y⇔ + − = ⇔ = ⇔ =
Thay x=y vào (1) ta có:
( )( )1 2 1 1 2 2 1 2 1x x x x x x− + − = ⇔ − + − + − − =
( )( )1 2 0 1, 2x x x x⇔ − − = ⇔ = =
V y h có hai nghi m là (x;y)=(1;1) và (x;y)=(2;2)
Bài 6. Gi i phương trình: ( ) ( )4 2
2 1
1 1
log 1 log 2 1
log 4 2x
x x
+
− + = + + (D b 1A – 2007)
Gi i:
K: x>1
( ) ( ) ( ) ( )4 4 4
1
1 log 1 log 2 1 log 2
2
x x x⇔ − + + − + =
( )( )
4
1 2 1 1
log à 1
2 2
x x
v x
x
− + 
⇔ = > 
+ 
2
2 1
2 à 1
2
x x
v x
x
− −
⇔ = >
+
2 5
2 3 5 0 à 1
2
x x v x x⇔ − − = > ⇔ =
BÀI T P ÁP D NG:
1)
( )2
2
2
4
log 6 7
2
1
1 log
4
x x
x x
+ −
≥
 
+ − + 
 
2) 2 3 2 3log log log logx x x x+ ≥
3) 2 2log 3 log 52
x x x+ =
4) ( ) ( )2 2
3 5log 15 log 45 2x x x x+ − − − =
5) ( ) ( )0.2 3 5log 2 log log 2x x x− + ≥ +
6) ( ) ( ) ( ) ( )2
2 33 log 2 4 2 log 2 16x x x x+ + + + + =
7) ( ) ( )2 2
3
1
log 3 1 2 log 1
log 2x
x x
+
− + = + +
8) CMR: v i m i a>0, h phương trình sau có nghi m duy nh t:
( ) ( )ln 1 ln 1x y
e e x y
x y a
 − = + − +

− =
9) Gi i h phương trình:
( ) ( )
( )2 2
2 2
2 2log 1 log
,
3 81x xy y
x y xy
x y R
− +
 + = +
∈
=
( H A 2009)
WWW.MATHVN.COM
2
10) Tìm m phương trình sau có úng 1
nghi m:
( ) ( )5 1 2 5 1 2
x x
m x+ + − =
11)
3 2 2 3
7 9.5 5 9.7x x x x
+ = +
12) ( ) ( )
7 5
5 7
x x
=
13) ( ) ( )
10
5 10
3 3 84 0
x x−
+ − =
14) ( )3 3
16 6 4 8 2 0x x
x x− −
+ − + − =
15) Tìm m phương trình sau có úng 1
nghi m:
2 2
sin cos
9 9x x
m+ =
16) ( )2
3
log 3 1x x
x−
− >
17) ( )3 3
16 6 4 8 2 0x x
x x− −
+ − + − =
18) Cho b t phương trình:
( ) ( ) ( )2
2 2log 1 log 1x ax a+ < +
a) Gi i b t phương trình khi a=2
b) Tìm t t c giá tr c a a b t phương
trình có nghi m
19)
2
2
3 9 6
x x
x x x
−
− = − +
20) ( )2 2
3.25 3 10 .5 3 0x x
x x− −
+ − + − =
21) Tìm m phương trình có 2 nghi m trái
d u:
( ) ( )3 16 2 1 4 1 0x x
m m m+ + − + + =
22) Tìm m phương trình có nghi m:
9 .3 2 1 0x x
m m− + + =
23) ( )2 5 4 5 3 5 3x x x
+ − − ≤ +
24) Tìm m h có nghi m:
( ) ( )2
2 2
log log 1mx y x y
x y m
 + + − =

− =
25) Gi i b t phương trình:
2 0.5
15
log log 2 2
16
x  
− ≤  
  
26) Gi i b t phương trình
3 4 1 1
3 4
3 1 1
log log log log
1 3 1
x x
x x
  −  + 
≤    + −    
WWW.MATHVN.COM
2
PH L C: M T S THI C N THAM KH O (Theo c u trúc thi c a B GD& T 2010)
1:
A. PH N CHUNG:
Câu 1: Cho hàm s (C) ( )( )2 21
1
4
y x m x= − + , m là tham s .
1. Kh o sát và v th (C) khi m =3
2. nh m bi t th hàm s (C) c t Ox t i A và B sao cho 2 ti p tuy n t i A và B vuông góc.
Câu 2:
1. Gi i phương trình:
3 27
cos 2 sin 2sin
2
x x x+ =
2. Gi i phương trình: ( ) ( )4 4 4 2x x x x x+ − − = −
Câu 3: Tính gi i h n:
( )2
2
sin 20
log cos
lim
2 1x xx
x x
x→
+
− +
Câu 4: Cho hình nón nh S có thi t di n qua tr c SO=a là m t tam giác vuông. M t ph ng qua S và c t
ư ng tròn áy t i A và B sao cho ∆ SAB u. Tình th tích hình c u ngo i ti p hình chóp SOAB.
Câu 5: Cho x,y,z [ ]0;1∈ . Tìm giá tr l n nh t: ( ) ( ) ( )
2 2 2
A x y y z z x= − + − + −
B. PH N T CH N: (Thí sinh ch ư c ch n Câu 6 ho c Câu 7
Câu 6: (Chương trình chu n)
a. Trong Oxy cho ∆ ABC có A(0;2), B(2;6), và : 3 1 0C d x y∈ − + = sao cho phân giác k t A song song
v i d. Tìm t a C.
b. Trong Oxyz vi t phương trình ư ng th ng ∆ qua A(0;1;2) c t 1
1 1
:
1 1 1
x y z
d
− −
= =
−
và h p v i
2
1 2 4
2 1 1
x y z
d
+ − −
= = =
−
m t góc 600
c. Cho ( ) ( ) ( )
1
1 1 01 1 ... 1 ,
n n n
n na x a x a x a x x R
−
−− + − + + − + = ∀ ∈ . Tìm n bi t 2 3 1 231a a a+ + =
Câu 7: (Chương trình nâng cao)
a. Trong Oxy tìm ( )
2 2
: 1
6 3
x y
M E∈ + = bi t kho ng cách t M n d: x+y=0 là l n nh t
b. Trong Oxyz vi t phương trình m t ph ng qua M(1;2;2) và c t Ox, Oy, Oz t i A,B,C sao cho:
2 2 2 2
1 1 1 1
OA OB OC OM
+ + =
c. B ng cách khai tri n: ( )
2
1
n
i+ hãy ch ng minh: ( )0 2 4 2
2 2 2 2... 1 2 cos
2
n n n
n n n n
n
C C C C
π
− + − + − = ,
( ), 0n N n∈ > .
WWW.MATHVN.COM
3
2:
A. PH N CHUNG:
Câu 1: Cho hàm s (C)
4 22
9
y x x= − +
1. Kh o sát và v th (C)
2. Tìm trên th (C) các i m A bi t ti p tuy n t i A c t (C) t i B và C sao cho AB=AC ( B,C khác A)
Câu 2:
1. Gi i phương trình: ( ) ( )1 3 cos sin 3 cos cos 1x x x x− + − =
2. Gi i h phương trình:
2 23
2 2 2
3 4 5
x y x y
x x y
 + − − =

+ + − =
Câu 3: Tính tích phân:
2
1 1 ln
e
dx
x x x+ −
∫
Câu 4: Cho lăng tr ng ABC.A’B’C’ có AB’=a; BC’=b và ∆ ABC vuông cân t i A. Tính th tích lăng
tr . ( )2a b a< <
Câu 5: Cho [ ], 1;2 .x y∈ Tính giá tr l n nh t và nh nh t:
( ) ( )2 2
2 2
1 1 1 1
4A x y x y
x y x y
   
= + + + − −   
   
B. PH N T CH N: (Thí sinh ch ư c ch n Câu 6 ho c Câu 7
Câu 6: (Chương trình chu n)
a. Trong Oxy tìm ( )
2 2
: 1
6 3
x y
M E∈ + = bi t góc F1MF2 b ng 600
.
b. Trong Oxyz vi t phương trình tham s ư ng th ng ∆ song song v i (P): 2x+2y-z-3=0 và c t hai
ư ng th ng 1
2 1
:
2 1 1
x y z
d
− −
= =
−
và 2
1 1
:
1 2 1
x y z
d
− +
= =
−
t i A và B sao cho AB=3
c. Gieo ng th i 3 con xúc x c, tính xác su t tích 3 s n t xu t hi n là 1 s ch n.
Câu 7: (Chương trình nâng cao)
a. Trong Oxy vi t phương trình chính t c hypebol qua M(2;1) th a góc F1MF2 b ng 600
b. Trong Oxyz vi t phương trình m t ph ng h p v i (Oxy) m t góc 450
, song song v i Ox và cách Ox m t
kho ng b ng 2
c. Cho z= 3 i+ . Tìm s t nhiên n>0 sao cho
n
z là s nguyên dương bé nh t.
WWW.MATHVN.COM
4
3:
A. PH N CHUNG:
Câu 1: Cho hàm s (C)
2mx
y
x m
+
=
+
1. Kh o sát và v th (C) khi m =-1
2. Tìm trên th (C) c t Ox t i A, C t Oy t i B sao cho 2 ti p tuy n t i A và B song song
Câu 2:
3. Gi i phương trình:
1
cos2 cos 3sin
2
x x x+ + =
4. Gi i phương trình: ( ) ( )2 2
2 3log 12 .log 12 2x x x x+ − − − =
Câu 3: Tính tích phân:
( )
2
4
0
sin3
1 cos
xdx
x
π
+
∫
Câu 4: Tính th tích hình chóp S.ABCD có áy là hình ch nh t, chi u cao SA=a h p v i (SBC) và
(SBD) các góc 450
và 300
Câu 5: nh m h sau có nghi m:
2
2
2
1
2 4
y
x xy
x x y m

− + =

 + − =
B. PH N T CH N: (Thí sinh ch ư c ch n Câu 6 ho c Câu 7)
Câu 6: (Chương trình chu n)
a. Vi t phương trình ư ng tròn i qua g c t a và c t Ox, Oy t i A,B sao cho AB= 4 2 . Bi t r ng
tâm ư ng tròn thu c d:x+y-4=0
b. Trong Oxyz vi t phương trình m t ph ng (P) qua M(1;1;0), song song v i
3
:
4 5 3
x y z
d
−
= =
−
và cách
g c t a m t kho ng b ng 1.
c. Tìm ,a b R∈ bi t phương trình 3
1 5
a b
z z
+ =
+ −
có 1 nghi m 1
5
1 2
i
z
i
=
+
. Tìm nghi m còn l i.
Câu 7: (Chương trình nâng cao)
a. Tìm t a 3 nh ∆ ABC vuông cân t i A có tr c i x ng là x-2y+1=0; ;A Ox B Oy∉ ∈ và
: 1 0C d x y∈ + − = .
b. Vi t phương trình tham s c a ư ng th ng d qua M(1;2;0), song song v i (P):2x-y+z-1=0 và h p v i
(Q): x+y+2z-1=0 m t góc 600
c. Trong h p ng 15 viên bi g m 4 bi , 5 bi xanh và 6 bi vàng. Tính xác su t ch n ư c 4 viên bi
c 3 màu.
WWW.MATHVN.COM
5
4:
A. PH N CHUNG:
Câu 1: Cho hàm s
3
2
3
x
y x= − + có th (C)
1. Kh o sát và v th (C)
2. Vi t Phương trình ư ng th ng d qua g c t a O và c t (C) t i A và B (khác O) saocho 2 ti p
tuy n c a (C) t i A và B vuông góc.
Câu 2:
5. Gi i phương trình:
tan tan sin 2 1 2sin2
4 2 2x x x x+ +
+ =
6. Gi i b t phương trình:
2 2 3
2 2 5
x x
x
x x
+ −
≥
− −
Câu 3: Tính tích phân:
44
4 4
0
sin
sin cos
x
dx
x x
π
+∫
Câu 4: Tính th tích hình chóp S.ABCD có áy là hình vuông chi u cao SA. Bi t SC=2a h p v i (SAB)
m t góc 300
.
Câu 5: Cho a,b,c>0 và a+b+c=1. Tìm giá tr nh nh t:
2 2 2
3 3 3
3
a b c
A a b c
+ +
= + + −
B. PH N T CH N: (Thí sinh ch ư c ch n Câu 6 ho c Câu 7)
Câu 6: (Chương trình chu n)
I/ Trong Oxyz cho A(2;3;-1), B(5;-3;2) và (P): x+y+z-3=0:
a. Vi t phương trình tham s ư ng th ng d vuông góc v i (P) và c t ư ng th ng AB t i I sao cho
2 0AI BI+ =
b. Tìm ( )M P∈ sao cho AM2
+2BM2
nh nh t
II/ Hãy phân ph i 2010 i m lên 2 ư ng th ng song song sao cho t ng s tam giác thu ư c là l n
nh t.
Câu 7: (Chương trình nâng cao)
I/
a Vi t phương trình ư ng tròn trong Oxy i qua A(2;1), Tâm thu c Oy và c t Ox t i B và C sao cho góc
BAC b ng 600
b. Trong Oxyz cho A(0;1;2), B(1;-1;1), C(-1;3;0). Vi t phương trình tham s ư ng th ng d vuông góc
v i (ABC) và c t (ABC) t i tr c tâm H c a ∆ ABC.
II/ nh m bi t th hàm s
( )2
1 2 1x m x m
y
x m
− + + −
=
−
ti p xúc v i Ox.
WWW.MATHVN.COM
6
5:
A. PH N CHUNG:
Câu 1: Cho hàm s
3
1
x
y
x
−
=
+
có th (C)
1. Kh o sát và v th (C)
2. Cho A(0;2). Tìm trên (C) i m M sao cho AM ng n nh t.
Câu 2:
1. Gi i phương trình:
2 2 3
cos cos cos3 cos 3
4
x x x x− + =
2. Gi i h phương trình:
2 2
2 2
1 1
3
1 1
1
x y
x y
x y xy

+ + + =


 + =
 +
Câu 3: Tính tích phân:
4
3
2
3
4
ln
1
x x
dx
x+
∫
Câu 4: Cho hình chóp S.ABCD có (SAB) ⊥ (ABC), ∆ ABC u và ∆ ABC vuông cân t i A. Tính th tích
m t c u ngo i ti p hình chóp Bi t SC= 2a
Câu 5: Cho a,b,>0 và
1 1
1
a b
+ = . Tìm giá tr nh nh t:
( )2 2
25
1 1 4
a b ab
A
a b a b
= + +
− − +
B. PH N T CH N: (Thí sinh ch ư c ch n Câu 6 ho c Câu 7)
Câu 6: (Chương trình chu n)
I/ Trong Oxyz cho A(2;-1;2), B(3;-3;3); C(1;-2;4) và (P): 2x-3y+z+1=0:
a. Vi t phương trình tham s ư ng th ng d i qua tâm ư ng tròn ngo i ti p ∆ ABC và vuông góc
v i (P)
b. Tìm ( )M P∈ sao cho AM2
+2BM2
+CM2
nh nh t
II/ Tìm ,a b R∈ bi t
2 3 4 2009
...Z i i i i i= − + − + + là nghi m c a phương trình 1
1 1
a b
z z
+ =
+ −
. Tìm
nghi m còn l i.
Câu 7: (Chương trình nâng cao)
I/ Trong Oxyz cho 1 : 1 2
2
x t
d y t
t
=

= +
 +
; 2
1
:
1 1 1
x y z
d
−
= =
−
a Tìm 1A d∈ bi t kho ng cách t A n d2 b ng 6
b. Vi t phương trình m t ph ng (P) ch a d2 và h p v i d1 m t góc 300
II/ Gi i h phương trình:
3 3
3
log log 2
2 6
log log 1
x
x y
x
y
y x
 + =

 + =

WWW.MATHVN.COM
7
6:
A. PH N CHUNG:
Câu 1: Cho hàm s (C)
4
2
1
4
x
y mx m= − + + , m là tham s .
1. Kh o sát và v th (C) khi m =1
2. nh m bi t th hàm s (C) có 3 i m c c tr t o thành tam giác có tr c tâm là g c t a
Câu 2:
1. Gi i phương trình: sin 2 cos 2 tan
6 3 4
x x x
π π π     
+ + + = +     
     
2. Gi i h phương trình:
( ) ( )3 3
3 3
2 3
1 1 1 1
8
log log 1
2 3
x y x y
x y x y
x y
    
+ + + + + =    
    

 =
Câu 3: Tính tích phân: 2
3
1
0
x
xdx
I
e +
= ∫
Câu 4: Tính th tích hình lăng tr u ABCD.A’B’C’D’ bi t AC’=a và góc gi a BD và CD’ b ng 600
.
Câu 5: Cho a,b,c>0 và
1 1 1
1
a b c
+ + = . Tìm giá tr l n nh t: 3 3 3 3 3 3
b c c a a b
A
b c c a a b
+ + +
= + +
+ + +
B. PH N T CH N: (Thí sinh ch ư c ch n Câu 6 ho c Câu 7
Câu 6: (Chương trình chu n)
a. Trong Oxy cho ∆ ABC vuông cân t i A có di n tích b ng 2, bi t 1 2 1 0A d x y∈ = − + = và
2, : 2 0B C d x y∈ + − = . Tìm t a A,B,C v i xA, xB>0.
b. Trong Oxyz vi t phương m t ph ng (P) qua A(0;1;2), B(1;3;3) và h p v i ( ): 2 0Q x y z− − = m t góc
nh nh t.
c. Tìm s t nhiên n th a:
3 2 3
1 1
1
7
n n nC C A+ +− =
Câu 7: (Chương trình nâng cao)
a. Trong Oxy cho hai ư ng tròn ( ) 2 2
: 2 2 0mC x y mx my m+ − − + − = và ( ) 2 2
: 3 1 0C x y x+ − + = . nh m
bi t s ti p tuy n chung c a hai ư ng tròn là m t s l .
b. Trong Oxyz vi t phương trình ư ng th ng d song song v i ( ): 2 1 0P x y z+ + − = và c t 2 ư ng th ng
Ox và
2 1
:
2 1 1
x y z− +
∆ = =
−
t i 2 i m A,B sao cho AB ng n nh t.
c. Gi i phương trình:
4 2
1 0z z+ + = , z C∈ .
WWW.MATHVN.COM
8
7:
A. PH N CHUNG:
Câu 1: Cho hàm s (C)
3 2
3y x ax b= − + , (1) ( ), 0a b >
1. Kh o sát và v th (C) khi a=1 b=4
2. nh a,b bi t th hàm s (C) có 2 i m c c tr A và B sao cho ∆ OAB vuông cân.
Câu 2:
3. Gi i phương trình:
2
tan2 1 tan .tan
2 sin3
x
x x
x
 
+ = 
 
4. Gi i h phương trình:
2 2 2 2
1 1 1
2
5 2 1
2
x y xy
x y x y

+ = +

 − =
 +
Câu 3: Tính gi i h n:
( )0
1
lim
ln 1 sin
x
x
e x
x→
− +
+
Câu 4: Cho hình chóp S.ABCD chi u cao SA=2a, áy là hình thang vuông t i A và B có AB=BC=a,
AD=2a. M t ph ng qua trung i m M c a SA ch a CD, c t SB t i N. Tính di n tích t giác CDMN.
Câu 5: nh m b t phương trình có nghi m:
( )2
1
ln 2 1
2
x x m x m
mx x
+ + − + − ≤
−
. Tìm nghi m tương ng
B. PH N T CH N: (Thí sinh ch ư c ch n Câu 6 ho c Câu 7
Câu 6: (Chương trình chu n)
a. Trong Oxy cho ( ) ( ) ( )7;1 , 3; 4 , 1;4A B C− − . Vi t phương trình ư ng tròn n i ti p ∆ ABC.
b. Trong Oxyz vi t phương trình m t ph ng (P) qua g c t a , song song v i
1 1 2
:
1 2 1
x y z
d
− + −
= =
−
và
h p v i
1 2
:
2 1 1
x y z+ −
∆ = = m t góc 600
c. Tìm h s c a
3
x trong khai tri n thành a th c c a bi u th c: ( )
62
1x x+ − .
Câu 7: (Chương trình nâng cao)
a. Trong Oxy cho ư ng tròn ( ) 2 2
: 6 5 0C x y x+ − + = . Tìm M thu c tr c tung sao cho qua M k ư c hai
ti p tuy n c a (C) mà góc gi a hai ti p tuy n b ng 600
b. Trong Oxyz Cho ( )2;1;0M và ư ng th ng d có phương trình
1 1
2 1 1
x y z− +
= =
− −
. Vi t phương trình
chính t c c a ư ng th ng i qua i m M, c t và vuông góc v i ư ng th ng d.
c. Tìm h s c a
3
x trong khai tri n thành a th c c a bi u th c: ( )
52
1x x+ − .
WWW.MATHVN.COM
9
8:
A. PH N CHUNG:
Câu 1: Cho hàm s (C)
1
1
mx
y
x
+
=
+
1. Kh o sát và v th (C) khi m =-1
2. nh m bi t ti p tuy n t i i m c nh c a h th (C) cách I(1;0) m t kho ng l n nh t
Câu 2:
1. Gi i phương trình:
2 2
sin sin 2 .sin 4 cos 2x x x x+ =
2. Gi i b t phương trình : ( )2 3 2 3
2 2 7 2 2 15x x x x+ − −
+ − + ≤
Câu 3: Tính th tích v t th tròn xoay sinh ra b i hình ph ng t o b i ( )
1 1
: 1 1 ,C y
x x
= + + − tr c Ox
và 2 ư ng th ng x=1; x=2 quay quanh Ox.
Câu 4: Cho hình vuông ABCD c nh a và hai ư ng th ng 1 2;d d l n lư t qua A và C và vuông góc v i
m t ph ng (ABCD). L y 1 2, NM d d∈ ∈ sao cho ,AM CN cùng chi u và có t ng dài b ng 6a. Tính
th tích t di n MNBD
Câu 5: Gi i h phương trình:
2
2
1 1
1 ln
1 1
1 ln
xy x
x y y
xy y
y x x

+ = + +

 + = +
 +
B. PH N T CH N: (Thí sinh ch ư c ch n Câu 6 ho c Câu 7)
Câu 6: (Chương trình chu n)
a. Trong Oxy cho A,B là hai i m trên ( ) 2
:P y x= sao cho ∆ OAB vuông t i A. Tìm t a A,B
( )0Ay < bi t OB ng n nh t.
b. Trong Oxyz vi t phương trình m t ph ng (P) qua g c t a và song song v i
1 1 2
:
2 2 1
x y z
d
− − −
= =
và cách d m t kho ng b ng 1.
c. Cho a giác l i n nh, bi t r ng s tam giác có nh và c nh chung v i a giác là 70. Tìm s tam
giác có nh chung và không có c nh chung v i a giác.
Câu 7: (Chương trình nâng cao)
a. Trong Oxy vi t phương trình chính t c elip (E) qua M(2;1) sao cho 1 2.MF MF nh nh t.
b. Trong Oxyz vi t phương trình m t ph ng (P) qua g c t a và l n lư t h p v i 2 m t ph ng
( ) ( ): 1 0 và : 2 1 0Q x z R x y z+ − = + − + = các góc 300
và 600
c. Tính giá tr : ( )( )2 2008 2 3 2008
1 2 3 ... 2009 1 2 3 4 ... 2009Z i i i i i i i= + + + + − + − + + .
WWW.MATHVN.COM
10
9:
A. PH N CHUNG:
Câu 1: Cho hàm s (C) ( )( )2
1y x m x x= − − +
1. Kh o sát và v th (C) khi m =3
2. nh m bi t (Cm) c t Ox t i A, c t Oy t i B sao cho hai ti p tuy n c a (Cm) t i A và B vuông góc.
Câu 2:
1. Gi i phương trình:
1 sin cos
tan
1 sin cos
x x
x
x x
− +
=
+ +
2. Gi i b t phương trình : ( ) ( )2 2 2
2
log log log 0.25
7 5 2 3 2 2
xx
x
+
+ = −
Câu 3: Tính di n tích hình ph ng gi i h n b i: ( ) 2
: 2 3C y x x= − − và : 1d y x= +
Câu 4: Cho hình chóp S.ABCD chi u cao SA=a, áy là hình vuông c nh a. ch ng minh AI ⊥ (SBD) av2
tính th tích t di n SIBD, bi t I là trung i m SC.
Câu 5: Tìm giá tr nh nh t tham s m h :
2
2
1 1
3
2
x y
x y m

+ =

 + =
có nghi m x,y>0. Tìm nghi m tương
ng.
B. PH N T CH N: (Thí sinh ch ư c ch n Câu 6 ho c Câu 7)
Câu 6: (Chương trình chu n)
a. Trong Oxy cho ∆ ABC có ư ng cao và trung tuy n k t A là 2 4 0Ah x y= + + = , 2 0Am y= − = và
ư ng trung tuy n k t B là :3 11 21 0Bm x y+ + = . Tính góc C
b. Trong Oxyz cho 1 2
2 1 2
: ,d : 2
1 2 1
1
x t
x y z
d y t
z t
=
− − − 
= = =
 = +
Ch ng minh r ng có vô s m t ph ng (P) ch a
d2 và song song v i d1. Vi t phương trình (P) sao cho d2 là hình chi u vuông góc c a d1 lên (P)
c. Tìm ,x y R∈ th a:
( ) ( )
2
1 1 1
2 2 1x y i y xi i
− =
+ − + + +
Câu 7: (Chương trình nâng cao)
a. Trong Oxy cho ( ) ( )
2 2
2 2
: 1 , 0
x y
H a b
a b
− = > có hai tiêu i m là 1F 2; F . ư ng th ng d qua 2; F vuông góc
Ox và c t (H) t i M và N sao cho 1F MN∆ u. Tìm tâm sai c a (H) và vi t phương trình (H) n u bi t di n
tích 1 4 3F MN∆ =
b. Trong Oxyz cho A(-1;2;2), B(0;3;0). Hãy tìm trong (P) sao cho ∆ ABC u.
c. M t ư ng th ng ti p xúc v i th hàm s
3 3
4
x
y
x
= + và c t 2 ư ng ti m c n t i A và B. Tính di n
tích ∆ OAB.
WWW.MATHVN.COM
11
10:
A. PH N CHUNG:
Câu 1: Cho hàm s ( ) ( )
4
2
1 , 1
2
x
y m x m
−
= + + − có th (C) . m là tham s .
1. Kh o sát và v th (C) khi m=0
2. Ch ng minh r ng th hàm s (1) luôn i qua 2 i m A và B c nh. nh m bi t 2 ti p tuy n t i
A và B h p nhau góc 600
Câu 2:
3. Gi i phương trình: 4sin 2 sin 1 3sin 2 cos2
3
x x x x
π 
+ = + − 
 
4. Gi i h phương trình:
2
2
4 8
3 12
x xy y
xy y x
 − + =

+ + =
Câu 3: Tính di n tích hình ph ng gi i h n b i:
lnx x
x
y
e +
= , tr c Ox và hai ư ng th ng x=1;x=4.
Câu 4: Tính th tích hình chóp S.ABC bi t SA, SB, SC ôi m t h p v i nhau góc 600
và có dài l n
lư t là a, 2a, 3a.
Câu 5: nh m phương trình ( ) ( )( )2 3log 2 4 1 log 1 3x m m x x− + = + − − −   có nghi m duy nh t. Tìm
nghi m duy nhât ó.
B. PH N T CH N: (Thí sinh ch ư c ch n Câu 6 ho c Câu 7)
Câu 6: (Chương trình chu n)
I/ Trong Oxyz cho
2 1
:
2 1 1
x y z
d
+ −
= = và (P): x-y-1=0:
a. Vi t phương trình tham s ư ng th ng d’ là hình chi u vuông góc c a d lên (P). Tính góc gi a
d và d’.
b. G i A là giao i m c a (P) và d. Vi t phương trình các m t c u ti p xúc (P) t i A và c t d t i B
sao cho AB= 6
II/ Gi i phương trình:
3
3 2 3 2
3 1
log log log log
23
x
x x
x
  
− = +  
   
Câu 7: (Chương trình nâng cao)
I/ Trong Oxyz cho A là giao i m c a 1 : 1 2
2
x t
d y t
t
=

= +
 +
và m t ph ng (P):x-2y+z=0
a Vi t phương trình chính t c ư ng th ng ∆ qua A vuông góc v i d và h p v i (P) m t góc 300
b. Vi t phương trình m t c u có tâm I thu c d, i qua A và c t P m t ư ng tròn dài 2 2π
II/ Tìm φ ( )0;2π∈ bi t th hàm s
( )2
2 cos 3sin
1
x x
y
x
ϕ ϕ+ + +
=
−
có hai i m c c tr là A và B
sao cho AB dài nh t, ng n nh t.
WWW.MATHVN.COM
12
11:
A. PH N CHUNG:
Câu 1: Cho hàm s ( )
2
, 1
1
x
y
x
=
−
có th (C) .
1. Kh o sát và v th (C) c a hàm s (1)
2. Tìm M trên (C) bi t ti p tuy n t i M t o v i 2 ti m c n c a (C) m t tam giác có chu vi bé nh t.
Câu 2:
5. Gi i phương trình:
2
16sin 4cos4 3 cos sinx x x x+ = +
6. Gi i phương trình: ( )
3
5 5 2x x x− − − =
Câu 3: Tính th tích v t th tròn xoay sinh ra b i hình tròn ( ) ( ) ( )
2 2
: 3 1 1C x y− + − = quay quanh tr c
Oy.
Câu 4: Cho t di n ABCD có AB=a, AC= 2a , AD=2a. ư ng th ng AC h p v i AB,AD các góc 450
,
AB h p v i AD góc 600
. Tính t s th tích c a t di n và hình c u ngo i ti p t di n.
Câu 5: Cho
2 2 2
1.a b c+ + = Ch ng minh r ng:
3 3 3
3 1a b c abc+ + − ≤ .
B. PH N T CH N: (Thí sinh ch ư c ch n Câu 6 ho c Câu 7)
Câu 6: (Chương trình chu n)
a. Trong Oxyz vi t phương trình m t ph ng (P) i qua H(1;2;3) và c t Ox, Oy, Oz l n lư t t i A, B, C
sao cho H là tr c tâm ∆ABC
b. Trong Oxyz vi t phương trình m t c u tâm I ∈ Oz, i qua A(1;1;1) và c t (Oxy) m t ư ng tròn dài 2π
c. Gi i phương trình :
0 1 2 2
2 3 4 .... 120 , xx
xC C C C N−
+ + + + = ∈
Câu 7: (Chương trình nâng cao)
I/ Trong Oxyz cho A(3;0;0) B(1;-2;8) và m t ph ng (P):x-2y+2z+6=0
a Tìm M∈(P) sao cho AM BM+ nh nh t.
b. Vi t phương trình m t ph ng (Q) qua A, B và c t (P) theo giao tuy n d h p v i AB góc 900
II/ Gi i h phương trình :
2 2
3 5 5 3
4 2 5.4
log log log .log
x x y x y
y xy xy
x y x y
− −
 + =
 + =
WWW.MATHVN.COM
13
12:
A. PH N CHUNG:
Câu 1: Cho hàm s (C) ( ) ( )
3
2 16
2 2 1
3 3
x
y mx m x
−
= + − − +
1. Kh o sát và v th (C) khi m =0
2. Ch ng minh r ng (Cm) luôn ti p xúc v i 1 ư ng th ng c nh t i 1 i m c nh.
Câu 2:
3. Gi i phương trình: ( )sin3 sin 3 cos 1x x x+ = −
4. Gi i b t phương trình :
2
0.522 2
4
log log 0,25 logx
x
x
+ ≥
Câu 3: Tính tích phân:
1
4
0
1 2
x
I dx
x
=
−∫
Câu 4: Cho hình tr có chi u cao b ng bán kính áy và b ng a. L y trên các ươgn tròn áy (O) và
(O’) các i m A, B sao cho AB=2a. tính góc gi a hai ư ng th ng OA, O’B và th tích t di n O’OAB
Câu 5: Cho a,b>0 và
1 1
1
a b ab
+ =
+
. Tìm giá tr nh nh t:
2 2
a b ab
P
ab a b
+
= +
+
B. PH N T CH N: (Thí sinh ch ư c ch n Câu 6 ho c Câu 7)
Câu 6: (Chương trình chu n)
a. Trong Oxy cho ∆ ABC có tâm ư ng tròn ngo i ti p là I(2;1), A∈Oy và ư ng th ng
BC:3x y 10 0− − = . Tìm t a A,B,C bi t góc BAC b ng 450
và 0A By y> >
b. Trong Oxyz cho A(0;1;0), B(1;-2;2). Hãy vi t phương trình m t ph ng (P) qua O, B và cách A m t
kho ng b ng
2
2
c. Gi i phương trình :
4
4 1 0z + =
Câu 7: (Chương trình nâng cao)
a. Trong Oxy cho ( ) 2
: 2P y x= có hai tiêu i m là F . ư ng th ng d quay quanh F c t (P) t i M,N.
Ch ng minh r ng
1 1
MF NF
+ không i.
b. Trong Oxyz vi t phương trình tham s ư ng th ng qua M(1;-2;2). d ⊥ OM và d h p v i Oy m t góc
450
c. Tìm h s c a
6
x trong khai tri n thành a th c c a bi u th c: ( ) ( )1 2
1 1
nn
P x x x
+
= + + + . Bi t h s c a
10
x b ng 10.
WWW.MATHVN.COM
14
PH L C II: Cách gi i nhanh bài toán b ng máy tính b túi.Phép chia theo sơ
Horner.
Trong các kì thi quan tr ng có môn toán, máy tính b túi ư c phép s d ng và tr thành công c không
th thi u i v i thí sinh. Tuy nhiên ít ai có th t n d ng ư c t i a các ch c năng c a máy tính trong
gi i toán. Nay tôi xin gi i thi u m t s phương pháp tìm nghi m b ng ch c năng SOLVE c a máy tính. Bài
vi t ư c vi t v i máy fx-570ES và tôi cũng khuyên các em t p làm quen s d ng máy này trong quá
trình gi i toán.
VD1. Tìm nghi m c nh: ( ) ( )3 2
2 3 1 6 4 0 1x a x ax− + + − =
Gi i:
So n phương trình (1) vào máy tính. ( )3 2
2 3 1 6 4 0x A x Ax− + + − = . D u = so n b ng cách nh n: ALPHA
+ CALC
Nh n ti p: Shift + SOLVE
Sau ó, máy h i: A=? ta cho ng u nhiên A=2 r i nh n phím =
Ti p n, d a vào “linh c m” mách b o, ta oán x=-3, nh n ti p phím =
Máy hi n nghi m x=0.5. Ta ghi nghi m này ra gi y. có th ây s là nghi m c nh c n tìm??!!
Nh n ti p Shift + SOLVE v i A=2
L n này ta th v i x=10
Máy hi n x=2 .
Thay A=-3;4;5.. và làm tương t ta ch th y máy báo x=2
V y ta k t lu n x=2 là nghi m c nh.
ây chính là cách tìm nghi m c nh trong bài t p trang 35
VD2. Tìm m sao cho: ( ) ( ) ( )3 2 2
3 1 2 4 1 4 1y x m x m m x m m= − + + + + − + c t Ox t i 3 i m phân bi t
có hoành >1
Gi i:
So n phương trình ( ) ( ) ( )3 2 2
3 1 2 4 1 4 1 0x A x A A x A A− + + + + − + = vào máy và nh n Shift + SOLVE.
Máy h i giá tr c a A. Ta cho a=3
Tai l i ti p t c oán nghi m x=-5
Máy hi n x=1.732281591 . Ta không quan tâm n nghi m này vì ây là nghi m “x u”. M c ích c a
ta là tìm nghi m h u t phân tích thành nhân t . Nh n ti p Shift + SOLVE.
L n này ta cho A=9 và x=10
Máy hi n x=10. Ta ghi nh n nghi m này
V i A=9 cho x=-5 ta nh n ư c k t qu x=2
Th tương t v i A b ng 1 vài giá tr và th x=2, x=10 vào ta u nh n ư c thông báo x=2. V y x=2
là nghi m c nh c a phương trình.
VD3. Gi i phương trình: ( )sin 2 cos2 cos 3sin 2 1x x x x+ − + =
Gi i:
Lúc này “lí trí” mách b o ta r ng. C n phân tích phương trình v phương trình tích. Hơn n a, ph i có
nghi m “ p” m i có th phân tích ư c. Ta dùng Shift + SOLVE tìm nghi m này.
Nh p phương trình trên vào máy
Nh n Shift + SOLVE.
Ta l n lư t th x b ng các góc c bi t như: ; ; ...
3 6 2
π π π
± ± ±
Khi th n các nghi m là à
2 6
v
π π
thì máy hi n r t nhanh. ki m tra ta nn n: sin( _ ALPHA _X_)
WWW.MATHVN.COM
15
Máy hi n =1 và =
1
2
. Và n u coi sin(x) là bi n thì có th phân tích phương trình qua 2 nhân t là
( )sin 1x − hay ( )2sin 1x − . Ta ch n phân tích theo hư ng ( )sin 1x − .
( )1 3sin 3 1 cos sin 2 cos2 0x x x x⇔ − + − + + =
( )2
3(sin 1) 1 1 2sin sin 2 cos 0x x x x⇔ − + + − + − =
( ) 2
3 sin 1 2(1 sin ) sin 2 cos 0x x x x⇔ − + − + − =
( )( )sin 1 1 2sin 2sin cos cos 0x x x x x⇔ − − + − =
( )( ) ( ) ( )( )sin 1 1 2sin cos 2sin 1 0 sin 1 1 2sin cos 0x x x x x x x− − + − = ⇔ − − + =
n ây, ta ã hoàn thành ư c ý ưa phương trình u tiên v phương trình tích. Vi c gi i phương
trình u gi ây ã tr nên d dàng.
GI I CÁC BÀI TOÁN HÌNH H C GI I TÍCH B NG MÁY TÍNH B TÚI FX – 570ES
Câu 1: Trong Oxyz cho: 1
2 2
: 1
1
x t
d y t
z
= +

= − +
 =
; 2
1
: 1
3
x
d y t
z t
=

= +
 = −
a) Tính kho ng cách gi a d1 và d2.
b)Vi t phương trình m t ph ng (P) ch a d1 và song song v i d2.
Gi i:
s d ng ch c năng vectơ c a máy ta nh n: MODE + 8 (vector)
Ch n vectơ A máy h i ta ch n h vectơ nào (Vct A(m) m?)
Ch n 1:3
Nh p t a vecto ch phương c a d1. (2;1;0) Nh n ti p Shift + STO + B copy các thông s c a vextơ
A vào vectơ B.
S a t a c a vectơ B thành (0;1;-1)
Ta có ( )1 2(2; 1;0) ; 1;1;3M d N d− ∈ ∈ ( )1;2;3MN⇒ − (Bư c này ghi ra gi y)
Nh n Shift+5(vector) Nh n 1 (Dim) 3(Vct C) sau ó nh p thông s c a vector ( )1;2;3MN −
a) Theo công th c: ( )1 2
1 2
;
1 2
; .
;
d d
d d MN
d
d d
 
 
=
 
 
tương ng v i:
; .
;
A B C
A B
 
 
 
 
là các vec tơ ư c lưu trong máy
tính.
tính tích có hư ng c a hai vectơ &A B ta nh n: ON Shift+5 3(vct A) x Shift+5 4 =
tính dài vector ta dùng ch c năng ABS(. b ng cách nh n phím Shift+hyp
tính tích vô hư ng &A B c a ta nh n ON Shift+5 3(vct A) Shift+5 7:●(dot) Shift+5 4(vct
B) =
V y nên tính dài c n tìm ta so n vào màn hình máy tính như sau:
(Abs((VctAxVctB)●VctC))÷(Abs(VctAxVctB))
K t qu máy hi n:
11
3
.
b)Vi t phương trình m t ph ng (P) ch a d1 và song song v i d2:
WWW.MATHVN.COM
16
Vi c u tiên c n làm ó là ta ph i tìm 1 vectơ pháp tuy n c a m t ph ng ( )α . g i vector pháp tuy n
c n tìm là a ta th y: ( )1
1 2
2
;
a d
d A d B
a d
 ⊥
= =
⊥
Nên a c n tìm là 1 2;d d 
  . tìm a b ng máy tính ta làm như sau:
ON Shift+5 3(vct A) x Shift+5 4 =
Màn hình so n th o hi n như sau:
VctAxVctB nh n phím = xem k t qu
Máy hi n: Vct Ans (-1;2;2)
V y ( )1;2;2a = − . Mp( )α i qua M(2;-1;0)
Nên ( ) ( ) ( ) ( ): 2 2 1 2 0 2 2 3 0x y z x y zα − − + + + = ⇔ − + + + =
Thí sinh ch c n gi các bư c làm vào bài làm, công vi c còn l i hãy cho máy tính. Ta th y hoàn thành 1
bài hình h c gi i tích trong thi th t nh nhàng.
Các b n có th th làm các bài toán có l i gi i trong sách giáo khoa hình h c 12 hay trong các sách tham
kh o b ng chi c máy tính c a mình. S có nhi u b t ng ang ch các b n khám phá!
SƠ HORNER VÀ NG D NG:
Chia a th c ( ) 1
0 1 ....n n
nP x a x a x a−
= + + + cho ( )x c− ta có:
( ) ( )( )1 2
0 1 1....n n
n nP x x c b x b x b x b− −
−= − + + + +
Trong ó ( )0;1;2;3;...;ib i n= nh b i sơ Horner:
a0 a1 a2 a3 …
c b0 b1 =cb0+ a1 b2 =cb1+ a2 b3 =cb2+ a3 bi =cbi-1+ ai
Áp d ng:
VD1. Tính thương và s dư trong phép chia:
( ) 4 3 2
2 8 6P x x x x x= + − − + cho x+2
Gi i:
Ta có sơ Horner:
2 1 -8 -1 6
-2 2 -3 -2 3 0
V y ( ) ( )( )3 2
2 2 3 2 3 0P x x x x x= + − − + +
n ây, chúng ta ã hi u ph n nào công d ng c a sơ horner. Trong bài toán liên quan n tham
s , vi c tìm ư c nghi m c nh và phân tích thành tích s làm công vi c gi i toán nh nhàng r t
nhi u. Nghi m c nh ã có máy tính, còn vi c chia a th c: Hãy sơ Horner làm cho b n.
Ta quay l i v i ví d u ph n ph l c:
VD2. Phân tích thành tích: ( ) ( )3 2
2 3 1 6 4 0 1x a x ax− + + − =
Gi i:
( )3 2
2 3 1 6 4 0x a x ax− + + − = Ta ã có ư c nghi m c nh x=2. v y nên
2 -3(a+1) 6a -4
2 2 -(3a-1) 2 0
V y (1) ( ) ( )3
2 2 3 1 2 0x x a x ⇔ − − − + = 
ây chính là m t ph n trong bài làm Bài3 trang 35.
VD3. nh m phương trình: ( ) ( ) ( )3 2
3 4 3 7 3 0mx m x m x m A− − + − − + =
có 3 nghi m dương phân bi t.
WWW.MATHVN.COM
17
Gi i:
Ta d dàng nh n ra: a+b+c+d=0 ⇒ phương trình (A) có 1 nghi m x=1
Sơ Horner:
m -3m-4 3m+7 -m+3
1 m -2(m-2) m-3 0
Nên ( ) ( ) ( )2
1 2 2 3 0A x mx m x m ⇔ − − − + − = 
(A) Có 3 nghi m dương phân bi t ( ) ( )2
2 2 3 0g x mx m x m⇔ = − − + − = có hai nghi m dương phân
bi t u khác 1
( ) ( )
( ) ( )
2
0
' 2 3 0
2
0
3
0
1 2 2 3 0
m
m m m
m
S
m
m
P
m
g m m m

≠

∆ = − − − >
 −
⇔ = >

−
= >

= − − + − ≠
( ) ( );0 3;4m⇔ ∈ −∞ ∪
VD4. nh m phương trình có 3 nghi m phân bi t:
( ) ( )3
1 1 0 1x m x− − − =
Gi i:
( ) 3
1 1x mx m⇔ − + −
Dùng máy tính ta “mò” ư c nghi m: x=1
Sơ Horner:
1 0 -m m-1
1 1 1 1-m 0
V y (1) ( )( )2
1 1 0x x x m⇔ − + + − =
(1) Có 3 nghi m phân bi t:
2
( ) 1 0g x x x m= + + − = có hai nghi m phân bi t khác 1
( )
3
4 3 0 3
34
1 1 1 1 0 4
3
m m
m
g m
m
∆ = − > > 
⇔ ⇔ ⇔ < ≠ 
= + + − ≠  ≠
Sơ Horner ng d ng r t nhi u trong gi i toán, nh t là d ng toán liên quan n kh o sát hàm s .
Các b n nên t p s d ng sơ này m t cách thu n th c. Bài t p áp d ng tôi s nêu lên 2 bài d ng
chia a th c nh m giúp các b n hoàn thi n kĩ năng.
BÀI T P:
Bài 1. N u x=-m là m t nghi m c a phương trình
3 2 2 3
4 6 0x mx m x m− + + = . Hãy tìm ghi m còn
l i.
Bài 2. Cho bi u th c: ( ) 5 4 3 2
2 3 7 11 9Q x x x x x= + − − + +
a. Tính giá tr bi u th c t i x=3
b. Tìm thương c a phép chia (Q) cho x-3
G i ý: Dư s c a phép chia (Q) cho x-3 là giá tr c a Q(3).
WWW.MATHVN.COM
18
B N QUY N THU C V NHÀ XU T B N I H C SƯ PH M THÀNH PH
H CHÍ MINH, KHI IN HAY TRÍCH D N PH I CÓ TÊN TÁC GI HO C NHÀ
XU T B N
WWW.MATHVN.COM

Weitere ähnliche Inhalte

Was ist angesagt?

Toan pt.de134.2011
Toan pt.de134.2011Toan pt.de134.2011
Toan pt.de134.2011BẢO Hí
 
Các đề thi đại học từ năm 2002 đén năm 2012" LuyenjthiBMT"
Các đề thi đại học từ năm 2002 đén năm 2012" LuyenjthiBMT"Các đề thi đại học từ năm 2002 đén năm 2012" LuyenjthiBMT"
Các đề thi đại học từ năm 2002 đén năm 2012" LuyenjthiBMT"luyenthibmt
 
Ap dung bat dang thuc de giai phuong trinh he pt
Ap dung bat dang thuc de giai phuong trinh he ptAp dung bat dang thuc de giai phuong trinh he pt
Ap dung bat dang thuc de giai phuong trinh he pthaohoctap
 
Toan pt.de030.2012
Toan pt.de030.2012Toan pt.de030.2012
Toan pt.de030.2012BẢO Hí
 
So phuc thanhtung
So phuc thanhtungSo phuc thanhtung
So phuc thanhtungHuynh ICT
 
152 bai toan on thi vao 10
152 bai toan on thi vao 10152 bai toan on thi vao 10
152 bai toan on thi vao 10Tam Vu Minh
 
Chuyên đề các dạng toán về lũy thừa số hữu tỉ
Chuyên đề các dạng toán về lũy thừa số hữu tỉChuyên đề các dạng toán về lũy thừa số hữu tỉ
Chuyên đề các dạng toán về lũy thừa số hữu tỉKim Liên Cao
 
Bai tap-toan-nang-cao-lop-7
Bai tap-toan-nang-cao-lop-7Bai tap-toan-nang-cao-lop-7
Bai tap-toan-nang-cao-lop-7Kim Liên Cao
 
Ve do thi ham tri tuyet doi
Ve do thi ham tri tuyet doiVe do thi ham tri tuyet doi
Ve do thi ham tri tuyet doiQuoc Nguyen
 
Chuyen de boi duong toan cua thay nguyentatthu
Chuyen de boi duong toan cua thay nguyentatthuChuyen de boi duong toan cua thay nguyentatthu
Chuyen de boi duong toan cua thay nguyentatthuTam Vu Minh
 
Cac chuyen _de_boi_duong_hoc_sinh_gioi_toan_lop_7_
Cac chuyen _de_boi_duong_hoc_sinh_gioi_toan_lop_7_Cac chuyen _de_boi_duong_hoc_sinh_gioi_toan_lop_7_
Cac chuyen _de_boi_duong_hoc_sinh_gioi_toan_lop_7_ntmtam80
 
204 de thi vao lop 10 chuyen chon
204 de thi vao lop 10 chuyen chon204 de thi vao lop 10 chuyen chon
204 de thi vao lop 10 chuyen chonTam Vu Minh
 
Cm bat dang thuc bang pp tiep tuyen
Cm bat dang thuc bang pp tiep tuyenCm bat dang thuc bang pp tiep tuyen
Cm bat dang thuc bang pp tiep tuyenVui Lên Bạn Nhé
 
Chuyen de-pt-bpt-va-hpt-dai-so
Chuyen de-pt-bpt-va-hpt-dai-soChuyen de-pt-bpt-va-hpt-dai-so
Chuyen de-pt-bpt-va-hpt-dai-soNguyen Van Tai
 
05 phuong trinh logarith p2
05 phuong trinh logarith p205 phuong trinh logarith p2
05 phuong trinh logarith p2Huynh ICT
 
04 phuong trinh mu p2
04 phuong trinh mu p204 phuong trinh mu p2
04 phuong trinh mu p2Huynh ICT
 

Was ist angesagt? (20)

Toan pt.de134.2011
Toan pt.de134.2011Toan pt.de134.2011
Toan pt.de134.2011
 
Các đề thi đại học từ năm 2002 đén năm 2012" LuyenjthiBMT"
Các đề thi đại học từ năm 2002 đén năm 2012" LuyenjthiBMT"Các đề thi đại học từ năm 2002 đén năm 2012" LuyenjthiBMT"
Các đề thi đại học từ năm 2002 đén năm 2012" LuyenjthiBMT"
 
Ap dung bat dang thuc de giai phuong trinh he pt
Ap dung bat dang thuc de giai phuong trinh he ptAp dung bat dang thuc de giai phuong trinh he pt
Ap dung bat dang thuc de giai phuong trinh he pt
 
Toan pt.de030.2012
Toan pt.de030.2012Toan pt.de030.2012
Toan pt.de030.2012
 
So phuc thanhtung
So phuc thanhtungSo phuc thanhtung
So phuc thanhtung
 
Basic số phức cực hay
Basic số phức cực hayBasic số phức cực hay
Basic số phức cực hay
 
152 bai toan on thi vao 10
152 bai toan on thi vao 10152 bai toan on thi vao 10
152 bai toan on thi vao 10
 
Chuyên đề các dạng toán về lũy thừa số hữu tỉ
Chuyên đề các dạng toán về lũy thừa số hữu tỉChuyên đề các dạng toán về lũy thừa số hữu tỉ
Chuyên đề các dạng toán về lũy thừa số hữu tỉ
 
Luong giac
Luong giacLuong giac
Luong giac
 
Bai tap-toan-nang-cao-lop-7
Bai tap-toan-nang-cao-lop-7Bai tap-toan-nang-cao-lop-7
Bai tap-toan-nang-cao-lop-7
 
Ve do thi ham tri tuyet doi
Ve do thi ham tri tuyet doiVe do thi ham tri tuyet doi
Ve do thi ham tri tuyet doi
 
Chuyen de boi duong toan cua thay nguyentatthu
Chuyen de boi duong toan cua thay nguyentatthuChuyen de boi duong toan cua thay nguyentatthu
Chuyen de boi duong toan cua thay nguyentatthu
 
Cac chuyen _de_boi_duong_hoc_sinh_gioi_toan_lop_7_
Cac chuyen _de_boi_duong_hoc_sinh_gioi_toan_lop_7_Cac chuyen _de_boi_duong_hoc_sinh_gioi_toan_lop_7_
Cac chuyen _de_boi_duong_hoc_sinh_gioi_toan_lop_7_
 
204 de thi vao lop 10 chuyen chon
204 de thi vao lop 10 chuyen chon204 de thi vao lop 10 chuyen chon
204 de thi vao lop 10 chuyen chon
 
Cm bat dang thuc bang pp tiep tuyen
Cm bat dang thuc bang pp tiep tuyenCm bat dang thuc bang pp tiep tuyen
Cm bat dang thuc bang pp tiep tuyen
 
10 de-thi-hsg-toan-10-co-dap-an
10 de-thi-hsg-toan-10-co-dap-an10 de-thi-hsg-toan-10-co-dap-an
10 de-thi-hsg-toan-10-co-dap-an
 
Chuyen de-pt-bpt-va-hpt-dai-so
Chuyen de-pt-bpt-va-hpt-dai-soChuyen de-pt-bpt-va-hpt-dai-so
Chuyen de-pt-bpt-va-hpt-dai-so
 
Bien doi dai_so
Bien doi dai_soBien doi dai_so
Bien doi dai_so
 
05 phuong trinh logarith p2
05 phuong trinh logarith p205 phuong trinh logarith p2
05 phuong trinh logarith p2
 
04 phuong trinh mu p2
04 phuong trinh mu p204 phuong trinh mu p2
04 phuong trinh mu p2
 

Ähnlich wie Phương Pháp Giải Nhanh Toán Đại Học - Tôi Là Quản Trị

Tổng quát về tích phân
Tổng quát về tích phân Tổng quát về tích phân
Tổng quát về tích phân Hoàng Hải Huy
 
Tich phan %28 nguyen duy khoi%29
Tich phan %28 nguyen duy khoi%29Tich phan %28 nguyen duy khoi%29
Tich phan %28 nguyen duy khoi%29trongphuckhtn
 
Tich phan (nguyen duy khoi)
Tich phan (nguyen duy khoi)Tich phan (nguyen duy khoi)
Tich phan (nguyen duy khoi)roggerbob
 
Những điều cần biết luyện thi quốc gia kỹ thuật giải nhanh hệ phương trình tá...
Những điều cần biết luyện thi quốc gia kỹ thuật giải nhanh hệ phương trình tá...Những điều cần biết luyện thi quốc gia kỹ thuật giải nhanh hệ phương trình tá...
Những điều cần biết luyện thi quốc gia kỹ thuật giải nhanh hệ phương trình tá...https://www.facebook.com/garmentspace
 
Cong thuc 2013 (pc2013072414 ovr's conflicted copy 2013 11-08)
Cong thuc 2013 (pc2013072414 ovr's conflicted copy 2013 11-08)Cong thuc 2013 (pc2013072414 ovr's conflicted copy 2013 11-08)
Cong thuc 2013 (pc2013072414 ovr's conflicted copy 2013 11-08)trongphuc1
 
Gtxstk dhnn1
Gtxstk dhnn1Gtxstk dhnn1
Gtxstk dhnn1Phi Phi
 
Cđ dãy số viết theo quy luật
Cđ dãy số viết theo quy luậtCđ dãy số viết theo quy luật
Cđ dãy số viết theo quy luậtCảnh
 
ĐẠI SỐ 9 - Phương trình bậc nhất, bậc hai, bậc ba và các bài toán thi vào lớp 10
ĐẠI SỐ 9 - Phương trình bậc nhất, bậc hai, bậc ba và các bài toán thi vào lớp 10ĐẠI SỐ 9 - Phương trình bậc nhất, bậc hai, bậc ba và các bài toán thi vào lớp 10
ĐẠI SỐ 9 - Phương trình bậc nhất, bậc hai, bậc ba và các bài toán thi vào lớp 10Bồi dưỡng Toán lớp 6
 
17_skkn_toan_9_ren_ki_nang_giai_bai_toan_bang_cach_lap_phuong_trinh_va_he_phu...
17_skkn_toan_9_ren_ki_nang_giai_bai_toan_bang_cach_lap_phuong_trinh_va_he_phu...17_skkn_toan_9_ren_ki_nang_giai_bai_toan_bang_cach_lap_phuong_trinh_va_he_phu...
17_skkn_toan_9_ren_ki_nang_giai_bai_toan_bang_cach_lap_phuong_trinh_va_he_phu...https://dichvuvietluanvan.com/
 
skkn toan 9 ren ki nang giai bai toan bang cach lap phuong trinh va he phuong...
skkn toan 9 ren ki nang giai bai toan bang cach lap phuong trinh va he phuong...skkn toan 9 ren ki nang giai bai toan bang cach lap phuong trinh va he phuong...
skkn toan 9 ren ki nang giai bai toan bang cach lap phuong trinh va he phuong...https://dichvuvietluanvan.com/
 
Chuyen de tich phan cuc hay
Chuyen de tich phan cuc hayChuyen de tich phan cuc hay
Chuyen de tich phan cuc hayOanh MJ
 
Cẩm nang kết cấu xây dựng
Cẩm nang kết cấu xây dựngCẩm nang kết cấu xây dựng
Cẩm nang kết cấu xây dựngAn Nam Education
 
Hàm số mũ
Hàm số mũHàm số mũ
Hàm số mũdiemthic3
 
Cđ giải hpt không mẫu mực
Cđ giải hpt không mẫu mựcCđ giải hpt không mẫu mực
Cđ giải hpt không mẫu mựcCảnh
 

Ähnlich wie Phương Pháp Giải Nhanh Toán Đại Học - Tôi Là Quản Trị (20)

Tổng quát về tích phân
Tổng quát về tích phân Tổng quát về tích phân
Tổng quát về tích phân
 
Tich phan %28 nguyen duy khoi%29
Tich phan %28 nguyen duy khoi%29Tich phan %28 nguyen duy khoi%29
Tich phan %28 nguyen duy khoi%29
 
Tich phan (nguyen duy khoi)
Tich phan (nguyen duy khoi)Tich phan (nguyen duy khoi)
Tich phan (nguyen duy khoi)
 
Những điều cần biết luyện thi quốc gia kỹ thuật giải nhanh hệ phương trình tá...
Những điều cần biết luyện thi quốc gia kỹ thuật giải nhanh hệ phương trình tá...Những điều cần biết luyện thi quốc gia kỹ thuật giải nhanh hệ phương trình tá...
Những điều cần biết luyện thi quốc gia kỹ thuật giải nhanh hệ phương trình tá...
 
Cong thuc 2013 (pc2013072414 ovr's conflicted copy 2013 11-08)
Cong thuc 2013 (pc2013072414 ovr's conflicted copy 2013 11-08)Cong thuc 2013 (pc2013072414 ovr's conflicted copy 2013 11-08)
Cong thuc 2013 (pc2013072414 ovr's conflicted copy 2013 11-08)
 
Phương pháp chặn
Phương pháp chặnPhương pháp chặn
Phương pháp chặn
 
Gtxstk dhnn1
Gtxstk dhnn1Gtxstk dhnn1
Gtxstk dhnn1
 
Giao trinh xac suat thong ke hn1
Giao trinh xac suat thong ke   hn1Giao trinh xac suat thong ke   hn1
Giao trinh xac suat thong ke hn1
 
Cđ dãy số viết theo quy luật
Cđ dãy số viết theo quy luậtCđ dãy số viết theo quy luật
Cđ dãy số viết theo quy luật
 
ĐẠI SỐ 9 - Phương trình bậc nhất, bậc hai, bậc ba và các bài toán thi vào lớp 10
ĐẠI SỐ 9 - Phương trình bậc nhất, bậc hai, bậc ba và các bài toán thi vào lớp 10ĐẠI SỐ 9 - Phương trình bậc nhất, bậc hai, bậc ba và các bài toán thi vào lớp 10
ĐẠI SỐ 9 - Phương trình bậc nhất, bậc hai, bậc ba và các bài toán thi vào lớp 10
 
Pt bpt-bac-cao
Pt bpt-bac-caoPt bpt-bac-cao
Pt bpt-bac-cao
 
17_skkn_toan_9_ren_ki_nang_giai_bai_toan_bang_cach_lap_phuong_trinh_va_he_phu...
17_skkn_toan_9_ren_ki_nang_giai_bai_toan_bang_cach_lap_phuong_trinh_va_he_phu...17_skkn_toan_9_ren_ki_nang_giai_bai_toan_bang_cach_lap_phuong_trinh_va_he_phu...
17_skkn_toan_9_ren_ki_nang_giai_bai_toan_bang_cach_lap_phuong_trinh_va_he_phu...
 
skkn toan 9 ren ki nang giai bai toan bang cach lap phuong trinh va he phuong...
skkn toan 9 ren ki nang giai bai toan bang cach lap phuong trinh va he phuong...skkn toan 9 ren ki nang giai bai toan bang cach lap phuong trinh va he phuong...
skkn toan 9 ren ki nang giai bai toan bang cach lap phuong trinh va he phuong...
 
Chuyen de tich phan cuc hay
Chuyen de tich phan cuc hayChuyen de tich phan cuc hay
Chuyen de tich phan cuc hay
 
Cẩm nang kết cấu xây dựng
Cẩm nang kết cấu xây dựngCẩm nang kết cấu xây dựng
Cẩm nang kết cấu xây dựng
 
Giaoan hoahoc10
Giaoan hoahoc10Giaoan hoahoc10
Giaoan hoahoc10
 
Hàm số mũ
Hàm số mũHàm số mũ
Hàm số mũ
 
Cđ giải hpt không mẫu mực
Cđ giải hpt không mẫu mựcCđ giải hpt không mẫu mực
Cđ giải hpt không mẫu mực
 
Emailing buoi 2 thuat toan
Emailing buoi 2   thuat toanEmailing buoi 2   thuat toan
Emailing buoi 2 thuat toan
 
Dai so 8t4149
Dai so 8t4149Dai so 8t4149
Dai so 8t4149
 

Mehr von Hải Finiks Huỳnh

Giao trinh-xu-ly-du-lieu-nghien-cuu-voi-spss-for-windows
 Giao trinh-xu-ly-du-lieu-nghien-cuu-voi-spss-for-windows Giao trinh-xu-ly-du-lieu-nghien-cuu-voi-spss-for-windows
Giao trinh-xu-ly-du-lieu-nghien-cuu-voi-spss-for-windowsHải Finiks Huỳnh
 
Bao cao tot nghiep ve marketing truc tiep bang internet
Bao cao tot nghiep ve marketing truc tiep bang internetBao cao tot nghiep ve marketing truc tiep bang internet
Bao cao tot nghiep ve marketing truc tiep bang internetHải Finiks Huỳnh
 
Giải chi tiết đề thi đh lí 2007-2014
Giải chi tiết đề thi đh lí 2007-2014Giải chi tiết đề thi đh lí 2007-2014
Giải chi tiết đề thi đh lí 2007-2014Hải Finiks Huỳnh
 
Niên giám 2012 đại học công nghiệp tphcm chuyên ngành kinh tế IUH
Niên giám 2012 đại học công nghiệp tphcm chuyên ngành kinh tế IUHNiên giám 2012 đại học công nghiệp tphcm chuyên ngành kinh tế IUH
Niên giám 2012 đại học công nghiệp tphcm chuyên ngành kinh tế IUHHải Finiks Huỳnh
 
Hướng dẫn cách trích dẫn và ghi tài liệu tham khảo cho luận văn, luận án tại ...
Hướng dẫn cách trích dẫn và ghi tài liệu tham khảo cho luận văn, luận án tại ...Hướng dẫn cách trích dẫn và ghi tài liệu tham khảo cho luận văn, luận án tại ...
Hướng dẫn cách trích dẫn và ghi tài liệu tham khảo cho luận văn, luận án tại ...Hải Finiks Huỳnh
 
Giai thich ngu phap tieng anh mai-lan-huong
Giai thich ngu phap tieng anh mai-lan-huongGiai thich ngu phap tieng anh mai-lan-huong
Giai thich ngu phap tieng anh mai-lan-huongHải Finiks Huỳnh
 
Tieu luan marketing ngan hang techcombank
Tieu luan marketing ngan hang techcombankTieu luan marketing ngan hang techcombank
Tieu luan marketing ngan hang techcombankHải Finiks Huỳnh
 
Tieu luan qttc cty-kinh do-iuh-2010-2012
Tieu luan qttc cty-kinh do-iuh-2010-2012Tieu luan qttc cty-kinh do-iuh-2010-2012
Tieu luan qttc cty-kinh do-iuh-2010-2012Hải Finiks Huỳnh
 
Noi dung LTDH toan 2014 - hai phung huynh
Noi dung LTDH toan 2014 - hai phung huynhNoi dung LTDH toan 2014 - hai phung huynh
Noi dung LTDH toan 2014 - hai phung huynhHải Finiks Huỳnh
 
[toilaquantri] lam chu_tu_duy_thay_doi_van_menh
[toilaquantri] lam chu_tu_duy_thay_doi_van_menh[toilaquantri] lam chu_tu_duy_thay_doi_van_menh
[toilaquantri] lam chu_tu_duy_thay_doi_van_menhHải Finiks Huỳnh
 

Mehr von Hải Finiks Huỳnh (20)

Giao trinh-xu-ly-du-lieu-nghien-cuu-voi-spss-for-windows
 Giao trinh-xu-ly-du-lieu-nghien-cuu-voi-spss-for-windows Giao trinh-xu-ly-du-lieu-nghien-cuu-voi-spss-for-windows
Giao trinh-xu-ly-du-lieu-nghien-cuu-voi-spss-for-windows
 
BÍ KIẾP VIẾT CV HOÀN HẢO
BÍ KIẾP VIẾT CV HOÀN HẢOBÍ KIẾP VIẾT CV HOÀN HẢO
BÍ KIẾP VIẾT CV HOÀN HẢO
 
Slide báo cáo tốt nghiệp
Slide báo cáo tốt nghiệpSlide báo cáo tốt nghiệp
Slide báo cáo tốt nghiệp
 
Bao cao tot nghiep ve marketing truc tiep bang internet
Bao cao tot nghiep ve marketing truc tiep bang internetBao cao tot nghiep ve marketing truc tiep bang internet
Bao cao tot nghiep ve marketing truc tiep bang internet
 
Giải chi tiết đề thi đh lí 2007-2014
Giải chi tiết đề thi đh lí 2007-2014Giải chi tiết đề thi đh lí 2007-2014
Giải chi tiết đề thi đh lí 2007-2014
 
Qc tin chi_moi_iuh_2014
Qc tin chi_moi_iuh_2014Qc tin chi_moi_iuh_2014
Qc tin chi_moi_iuh_2014
 
Niên giám 2012 đại học công nghiệp tphcm chuyên ngành kinh tế IUH
Niên giám 2012 đại học công nghiệp tphcm chuyên ngành kinh tế IUHNiên giám 2012 đại học công nghiệp tphcm chuyên ngành kinh tế IUH
Niên giám 2012 đại học công nghiệp tphcm chuyên ngành kinh tế IUH
 
100 bai nghe tieng anh can ban
100 bai nghe tieng anh can ban100 bai nghe tieng anh can ban
100 bai nghe tieng anh can ban
 
100 bai nghe anh van can ban
100 bai nghe anh van can ban100 bai nghe anh van can ban
100 bai nghe anh van can ban
 
Si(hệ đo lường quốc tế)
Si(hệ đo lường quốc tế)Si(hệ đo lường quốc tế)
Si(hệ đo lường quốc tế)
 
Hướng dẫn cách trích dẫn và ghi tài liệu tham khảo cho luận văn, luận án tại ...
Hướng dẫn cách trích dẫn và ghi tài liệu tham khảo cho luận văn, luận án tại ...Hướng dẫn cách trích dẫn và ghi tài liệu tham khảo cho luận văn, luận án tại ...
Hướng dẫn cách trích dẫn và ghi tài liệu tham khảo cho luận văn, luận án tại ...
 
Huong dan thuc hanh kinh te luong
Huong dan thuc hanh kinh te luongHuong dan thuc hanh kinh te luong
Huong dan thuc hanh kinh te luong
 
Giai thich ngu phap tieng anh mai-lan-huong
Giai thich ngu phap tieng anh mai-lan-huongGiai thich ngu phap tieng anh mai-lan-huong
Giai thich ngu phap tieng anh mai-lan-huong
 
Tieu luan marketing ngan hang techcombank
Tieu luan marketing ngan hang techcombankTieu luan marketing ngan hang techcombank
Tieu luan marketing ngan hang techcombank
 
10 vạn câu hỏi vì sao
10 vạn câu hỏi vì sao10 vạn câu hỏi vì sao
10 vạn câu hỏi vì sao
 
Tieu luan qttc cty-kinh do-iuh-2010-2012
Tieu luan qttc cty-kinh do-iuh-2010-2012Tieu luan qttc cty-kinh do-iuh-2010-2012
Tieu luan qttc cty-kinh do-iuh-2010-2012
 
Noi dung LTDH toan 2014 - hai phung huynh
Noi dung LTDH toan 2014 - hai phung huynhNoi dung LTDH toan 2014 - hai phung huynh
Noi dung LTDH toan 2014 - hai phung huynh
 
Everyday english, second edition
Everyday english, second editionEveryday english, second edition
Everyday english, second edition
 
[toilaquantri] lam chu_tu_duy_thay_doi_van_menh
[toilaquantri] lam chu_tu_duy_thay_doi_van_menh[toilaquantri] lam chu_tu_duy_thay_doi_van_menh
[toilaquantri] lam chu_tu_duy_thay_doi_van_menh
 
Xu huong cac nhom nganh o vn
Xu huong cac nhom nganh o vnXu huong cac nhom nganh o vn
Xu huong cac nhom nganh o vn
 

Phương Pháp Giải Nhanh Toán Đại Học - Tôi Là Quản Trị

  • 1. Hoàng Vi t Quỳnh Toaën hoåc phöí thöng Các phương pháp gi i nhanh thi i h c WWW.MATHVN.COM Liên hệ: Ketnoitrithuc2013@gmail.com http://toilaquantri.com - Chia sẻ nhiều tài liệu LTĐH bổ ích https://www.facebook.com/phung.huynh.93.1102
  • 2. 1 Các phương pháp gi i toán i s và gi i tích L i nói u: Sau 12 năm h c t p, gi ây ch còn m t kì thi duy nh t ang ch i các em ó là kì thi i h c. ây s là kì thi khó khăn nh t trong su t 12 năm các em ng i trên gh nhà trư ng. Kì thi i h c chính là m t bư c ngo t l n trong cu c i c a m i h c sinh vì th m i h c sinh c n ph i chu n b ki n th c th t toàn di n vì n i dung c a thi mang tính liên t c. Có l trong các môn, môn toán v n luôn chi m v trí quan tr ng và là v t c n l n nh t trên bư c ư ng ti n t i gi ng ư ng i h c. Vì th tôi xin m o mu i góp chút ki n th c ã thu lư m ư c trong quá trình h c t p vi t lên quy n sách này. Hy v ng ây s là tài li u b ích cho các em h c t p. Quy n sách ư c chia thành sáu ơn v bài h c và hai ph l c. M i bài u là nh ng ph n quan tr ng, xu t hi n thư ng xuyên trong thi i h c. m i bài u có nh ng c i m sau: • Ph n tóm t t ki n th c ã h c ư c trình bày ng n g n và t ng quát nh m khơi l i ph n ki n th c ã quên c a các em. • H th ng các bài làm ư c ch n l c kĩ lư ng, có tính i n hình và khai thác t i a các góc c nh c a v n nêu ra, ng th i phương pháp gi i ng n g n, tr c quan cùng nhi u kinh ngh m gi i giúp các em có th hi u ư c n i dung bài gi i và cách áp d ng cho các d ng thi s g p sau này. ng th i, các ví d u ư c trình bày t cơ b n n nâng cao. ây là nh ng bài trích ra t thi d tr c a các năm trư c và tham kh o t nh ng tài li u c a các th y cô có nhi u năm kinh nghi m trong quá trình luy n thi nên m b o v m c và gi i h n ki n th c. L i gi i trong các ví d ch là tư ng trưng nh m m c ích nêu lên phương pháp gi i, các em và các th y cô khi tham kh o cu n t i li u này có th tìm ra và trình bày cách gi i và cách trình bày h p lí hơn. Các em nên t p gi i các d ng bài trên m t cách thu n th c và c l p. sau khi gi i xong m i xem ph n l i gi i. ó là i u mà tác gi kì v ng nhi u nh t. • Lí gi i các phương pháp, ưa ra thu t toán gi i chung, ưa ra b n ch t l i gi i, ó là ph n l i bình, lưu ý cu i m i bài t p. Ph n ph l c là 12 thi tiêu bi u theo c u trúc thi m i nh t do B GD& T công b . Các thi có m c khó r t cao, òi h i ngư i làm ph i tư duy r t nhi u. V i m c khó ó, tôi mong r ng khi các em gi i thu n th c các bài trong b thi này các em s có t tin và ki n th c t i m cao khi làm bài môn toán. Ph l c 2 là m t s m o dùng máy tính oán nghi m c nh, ph c v cho quá trình gi i các bài t p v phương trình tích như lư ng giác, h phương trình, phương trình, cách gi i nhanh bài toán hình h c b ng máy tính… ng th i gi i thi u thêm phương pháp chia Horner giúp các em làm nhanh bài toán có chia a th c, phân tích thành tích… V i d nh là s gi i thi u quy n sách cho các em trong tháng cu i cùng trư c khi thi i h c nên sách ã gi n lư c m t s ph n không c n thi t và các ki n th c bên l , ch gi i thi u nh ng tr ng tâm c a thi nên bài t p có th còn ít. Tôi cũng có l i khuyên cho các thì sinh là hãy tìm thêm các thi trên m ng internet vì ây là kho ki n th c vô t n. M c dù r t c g ng nhưng cu n sách r t có th còn nhi u thi u sót do th i gain biên so n ng n ng th i kinh nghi m và s hi u bi t còn h n ch . R t mong ư c s góp ý c a b n c. M i góp ý xin liên h v i tác gi qua a ch sau: Hoàng Vi t Quỳnh Khu 6a – Th tr n L c Th ng – B o Lâm – Lâm ng Email: vquynh2971991@yahoo.com.vn Blog: http://vn.myblog.yahoo.com/vquynh-qflower Tel: 063-3960344 - 01676897717 WWW.MATHVN.COM
  • 3. 2 Bài I: ng d ng phương trình ư ng th ng gi i phương trình căn th c. VD1. Nh c l i ki n th c v ư ng th ng. 1) Phương trình t ng quát: ư ng th ng i qua M(x0;y0) và có vetơ pháp tuy n n (A;B) thì ư ng th ng ó có phương trình: (d): A(x-x0)+B(y-y0)=0 (d): Ax+By+C=0 VD1. ư ng th ng qua M(1;2) nh n n (2;1) làm vectơ pháp tuy n. (d): 2(x-1)+1(y-2)=0 (d): 2x+y-4=0 2) Phương trình tham s : ư ng th ng i qua M(x0;y0) và có vectơ ch phương a (a1;a2) (d):    += += tayy taxx 20 10 VD2. ư ng th ng qua M(3;4) nh n a (2;3) làm vtcp có phương trình: (d):    += += ty tx 34 23 VD3. Cho (d): x+y=4. Vi t phương trình tham s c a (d). Gi i: Vectơ pháp tuy n : n (1,1) Vectơ ch phương : a (1,-1) i m i qua M(2;2) (d) :    −= += ty tx 2 2 VD2. ng d ng VD1. Gi i phương trình : 101238 33 =−++ xx Gi i: t: 83 +x =1+3t và 3 12 x− =3-t k( -1/3 ≤t≤1/3) x3 +8=(1+3t)2 (*) và 12-x3 = (3-t)2 (**) L y (*)+(**) ta có 20=10t2 +10 t2 =1 t=1 ho c t=-1(lo i) x3 =8 x=2 Tip: Có ph i b n ang t h i: thu t toán nào ã giúp ta nhìn th y ư c cách t n t ??? WWW.MATHVN.COM
  • 4. 3 Không ph i ng u nhiên mà tôi l i trình bày l i v n ư ng th ng, m t v n tư ng ch ng như ch ng liên quan gì n i s . Nhưng gi ây ta m i nh n ra ư c “ ư ng th ng” chính là “tuy t chiêu” gi i phương trình d ng căn th c. M u ch t ó là: B1: 101238 33 =−++ YX xx T ó ta có phương trình ư ng th ng : X+3Y=10 B2: ta vi t l i phương trình: X+3Y=10 theo tham s t    = = t-3Y 3t+1X Lúc này phương trình ã quy v 1 n t và vi c gi i phương trình trên là không khó. (Vì ây là ki n th c “l p nhí”) hi u rõ hơn v phương pháp này các b n hãy cùng tôi n v i VD2. VD2. Gi i phương trình : X x 3+ + Y x3 2+ =1 Gi i: G i (d): X=1+t và Y=0+t (1) t     =+ −=+ tx tx 3 2 13 (t≤1)     =+ +−=+ 3 2 2 213 tx ttx L y phương trình 2 tr pt1 ta có: -1=t3 -t2 +2t-1 t3 -t2 +2t=0 • T=0 x=-2 Lưu ý: Trong khi gi i thi, các b n nên trình bày t bư c(1) tr i nh m m b o tính ng n g n cho bài toán. Bư c g i phương trình ư ng th ng ch nên làm ngoài gi y nháp. • Trong bài trên ta có th t     =+ =+ vx ux 3 2 3 và quy v gi i h phương trình. Các b n có th xem cách này như m t bài t p. các b n hãy làm và so sánh s ưu vi t gi a 2 phương pháp. • Trong bài trên ta h n ch phương pháp lũy th a vì n u mu n kh 2 căn th c khác b c trên, ta ph i ^6 phương trình. Ta s g p khó khăn và s i m t v i 1 phương trình “kinh kh ng” và ta ph i gi i “x t khói” m i có th ra nghi m. VD3. Gi i h phương trình : ( ) ( )    =+++ =−+ 2411 13 yx xyyx ( thi H năm 2005) Gi i: t:     −=+ +=+ ty tx 21 21 (-2≤t≤2)     +−=+ ++=+ 441 441 2 2 tty ttx     +−= ++= 34 34 2 2 tty ttx Phương trình(1) tr thành: 2t2 +6- )43)(43( 22 tttt −+++ =3 WWW.MATHVN.COM
  • 5. 4 910 24 +− tt =2t2 +3 ho c t=0 x=y=3 VD4. nh m phương trình sau có nghi m: Gi i: phương trình có nghi m: mxf =)( Min f(x)≤m ≤Max f(x) t     −=− +=+ txm tmx 33 312 (-1/3≤t≤3)     +−=− ++=+ 2 2 693 9612 ttxm ttmx c ng v v i v => 5m=10+10t2 2t2 +2=m f(t)=m V i f(t)= 2t2 +2 mi n xác nh: D=[-1/3;3] F’(t)=4t =>f’(t)=0 t=0 t -∞ -1/3 0 3 +∞ F’(t) - 0 + F(t) 20/9 20 2 M có nghi m 2≤m≤20 VD3. Bài t p t luy n 1) Gi i h phương trình: 2) Gi i h phương trình: 3) Gi i h phương trình: 2 1 1 1 3 2 4 x y x x y  + + − + =  + = ( thi d b 1A – 2005) 4) Gi i phương trình: 1 sin( ) 1 cos( ) 1x x− + + = ( thi d b 2A – 2004) WWW.MATHVN.COM
  • 6. 5 Bài II: Các cách gi i phương trình và b t phương trình vô t . 1)Lũy Th a Phương pháp lũy th a là phương pháp t ng quát nh t gi i phương trình có căn. Khi g p các phương trình có d ng căn ph c t p nhưng khi chúng ta bi t “m o lũy th a” thì có th gi i bài toán m t cách d dàng. ây là m t phương pháp cơ b n, các b n ph i th c t p nhu n nhuy n vì phương trình trong thi i h c có lúc r t d nhưng ta l i không ý. các b n hãy theo dõi các ví d sau. Nhưng trư c h t hãy lưu ý v n sau: • t i u ki n • Lũy th a ch n thì hai v không âm • Các d ng cơ b n: BA =    = ≥ 2 0 BA B BA <    ≤≤ ≥ 2 0 0 BA B BA >           > ≥    ≥ < 2 0 0 0 BA B A B VD1. Gi i:        =−+−+ ≥− ≥− ≥ 10)5(25 010 05 0 xxxx x x x     −=− ≤≤ xxx x 552 50 2    +−=− ≤≤ 22 1025)5(4 50 xxxx x    =+− ≤≤ 056 50 2 xx x x=1 ∨ x=5 VD2. 132 −<+− xxx Gi i: 2 x = 3−x + 1−x     −++−++< ≥ )1)(3(2134 1 xxxxx x     −>−+ ≥ 132 1 2 xxx x    +−>−+ ≥ 1232 1 22 xxxx x    > ≥ 1 1 x x x=1 WWW.MATHVN.COM
  • 7. 6 VD3. Gi i: k: 2x+1>0 x>1/2 Bpt (4x2 -4x+1)(x2 -x+2)≥36 t t = (x2 -x) bpt tr thành: (4t+1)(t+2)≥36 4t2 +9t-34≥0 t≤-17/4 ho c t≥2 x2 -x≤-17/4 ho c x2 -x≥2 x≤1 ho c x≥2 VD4. Gi i b t phương trình : Gi i:          ≥−− >− =+− 02 0 0 2 2 2 xx xx xx 10 =∨=⇔ xx Lưu ý: b t phương trình trên các b n không nên lũy th a tính toán vì quá trình lũy th a và nhân phân ph i r t m t th i gian. Hơn n a, khi quy v m t phương trình h qu , chúng ta gi i r t d sai vì khi giao các t p nghi m s không có giá tr nào th a mãn. Trong bài trên tôi s d ng cách ánh giá theo ki u như sau: A B ≥0         ≥ > = 0 0 0 A B B ó chính là m u ch t c a bài toán VD5. Gi i phương trình : Gi i:                − =− ≥− ≥      − − 2 2 4 53 8 053 0 4 53 2 x x x x x=3 WWW.MATHVN.COM
  • 8. 7 Lưu ý: Trong phương trình trên các b n ph i “ ý” và “nhanh” m t chút vì n u như ta nguyên phương trình cho lũy th a thì ó là m t i u “không còn gì d i b ng” ta s i m t v i chuy n lũy th a 2 l n => m t phương trình b c 4. Phương trình này ta không th b m máy tính. Nhưng n u gi i tay thì ph i gi i “x t khói” m i ra trong khi th i gian không ch i ai. ng th i chúng ta không c n gi i i u ki n v i vì giám kh o ch quan tâm n bài làm và k t qu . Chúng ta hãy ch vi t “cái sư n” c a i u ki n. sau khi gi i ra nghi m ch vi c th vào i u ki n là xong. 2) Phương pháp t n ph : CÁCH GI I: ( ) ( ) ( ) 0)();( 0)();( 0)();( = ≤ ≥ n n n xuxuf xuxuf xuxuf t= n xu )( Phương trình h u t ho c h phương trình BÀI T P ÁP D NG: VD1. Gi i: t t= => t>0 ; t2 +2= x2 + x 3t=2(t2 -1) t=-0.5 (lo i) ho c t=2 x2 +x=6 x=2 ho c x=3 VD2. Gi i: T= 1−x    =+ ≥ xt t 1 0 2 Phương trình tr thành: t2 +1-(t+1)=2 t2 -t-2=0 t=2 ho c t=-1 x=5 VD3. Gi i: => WWW.MATHVN.COM
  • 9. 8 pt tr thành: t2 +t+2=8 t=2 ∨ t=-3 TH1: t=2 TH2: t=-3 LO I II: ( )nn xvxuf )()( + { ≥0; ≤0; =0 } Phương pháp chung:     = = vxv uxu m n )( )( => ưa v h phương trình. VD1. 085632323 =−−+− xx ( tuy n sinh i h c 2009) Gi i:     ≥=− =− )0(56 233 vvx ux     =−+ =+ 0832 3 8 3 5 23 vu vu       − = =+ 3 28 3 8 3 5 23 u v vu        − = =      − + 3 28 3 8 3 28 3 5 2 3 u v u u      − = =+−+ 3 28 0)202615)(2( 2 u v uuu    = −= 4 2 v u x=-2 LO I III: H PHƯƠNG TRÌNH A TH C Nh ng h phương trình này ta r t thư ng hay g p trong thi i h c. l p 10, ta thư ng g p nh ng phương trình có tên là h i x ng, ng c p… Nh ng h này ã có cách gi i “ăn li n”. nhưng trong thi i h c, ta không h tìm th y nh ng d ng ó. Nhưng t t c các h trên u quy v m t m i ó là “Phân tích thành nhân t ”. WWW.MATHVN.COM
  • 10. 9 VD1. Gi i h phương trình: ( ) ( )3 1 1 1 2 1 2 x y x y y x  − = −   = + ( H A 2003) Gi i: K: xy≠0 Ta có ( ) ( ) 1 1 1 0 1 x y x y xyxy =   ⇔ − + = ⇔   = −   TH1: ( )( )23 3 1 1 5 1 1 0 22 1 2 1 1 5 2 x y x yx y x y x y x x xy x x x x y   = = == =  − + ⇔ ⇔ ⇔ = =   − + − == + = +    − − = = TH2: 3 3 4 1 1 1 22 1 1 2 0 y xy yx x y x x x x x  = −= − = −   ⇔ ⇔   = +  − = + + + = Mà 2 2 4 2 1 1 3 2 0, 2 2 2 x x x x x VN     + + = − + + + > ∀ ⇒        V y nghi m c a h là ( ) ( ) 1 5 1 5 1 5 1 5 ; 1;1 , ; , ; 1 1 1 1 x y    − + − + − − − − =            VD2. Gi i h phương trình: ( ) ( ) ( ) 2 2 x 1 y(y x) 4y 1 x, y R . (x 1)(y x 2) y 2  + + + = ∈ + + − = (D b A2006) Gi i: ( ) ( ) ( )2 1 1 4 0 *x y x y⇔ + + + − = t: 2 1 0; 4u x v x y= + > = + − H ( ) ( ) ( ) 0 3 2 4 u yv u v y − = ⇔  + = Thay (4) vào (3) ta có: ( ) ( ) ( )3 2 . 0 1 2 0u u v v u v v⇔ + + = ⇔ + + =   2 2 1 0v v⇔ + + = 2 ( 1) 0 1 3v v x y⇔ + = ⇔ = − ⇔ + = V y (*) ( ) 2 2 1 21 0 1 3 0 2 53 x yx y x x x yx y = ⇒ = − + − =  ⇔ ⇔ + − − = ⇔  = ⇒ == −  VD3. Gi i h phương trình ( ) ( ) 3 3 2 2 x 8x y 2y x, y R . x 3 3(y 1) *  − = + ∈ − = + (D b 2A 2006) Gi i: H ( ) ( ) ( ) ( ) ( ) 3 33 3 2 2 2 2 3 6 4 2 12 4 3 6 3 6 2 x y x yx y x y x y x y  − = + − = +  ⇔ ⇔  − = − =   L y (2) thay vào (1) ta có ( ) ( )( )3 3 2 2 3 2 2 3 3 4 12 0x y x y x y x y x x y⇔ − = − + ⇔ − + = ( )2 2 12 0x x xy y⇔ + − = D th y x=0 thì y=0. Th vào (*) ta th y không th a mãn. V y ây không ph i là nghi m c a phương trình: WWW.MATHVN.COM
  • 11. 10 ( )( )2 2 2 2 2 2 3 4 012 0 3 6 3 6 x y x yx xy y x y x y  − + = + − =  ⇒ ⇔  − = − =   TH1: 2 2 2 3 0 3 1 3 1 33 6 6 6 x y x y y x y xx y y − = = = ⇒ =   ⇔ ⇔   = − ⇒ = −− = =   TH2: 2 2 2 78 4 78 4 4 13 13 3 6 13 6 78 4 78 13 13 y x x y x y x y y y x  − = ⇒ == − = −  ⇔ ⇔  − = =  = − ⇒ =  V y nghi m c a phương trình là: ( ) ( ) ( ) 78 4 78 78 4 78 ; 1;3 , 1; 3 , ; , ; 13 13 13 13 x y    − − = − −            VD4. Gi i h phương trình ( )( ) ( ) ( )( ) ( ) 2 2 2 2 13 1 25 2 x y x y x y x y  − + =  + − = (D b 2005) Gi i: Nhân c 2 v c a (1) cho 25. Nhân c 2 v c a (2) cho 13. Sau ó l y (1)-(2). (1)-(2) ( ) ( )( ) ( ) ( ) ( )22 2 2 2 2 13( ) 25 0 13 25 0x y x y x y x y x y x y x y ⇔ + − − − + = ⇔ − + − + =   ( )( ) ( )( )2 2 2 2 12 26 12 0 2 12 26 12 0x y x xy y x y x xy y⇔ − − + − = ⇔ − − − + − = D th y x=y không th a mãn h . ( )( ) ( )( ) ( ) ( ) 2 2 2 2 2 3 2 3 25 3 2 . 25 2 3 2 2 3 0 9 3 2 3 2 325 25 325 1 . 25 24 2 x y y y x y y x x y x y x y x yx y x y x y x y x y y y  = = − ⇔−   = = = −  − − =    = ⇒ ⇔ ⇔  =+ − =   + − =  =   = ⇔   =   L i bình: Làm sao ta có th phân tích nhanh ( )2 2 12 26 12x xy y− + − thành nhân t ( )( )3 2 2 3x y x y− − ?? Lúc này, công c c a chúng ta chính là máy tính b túi! Các b n hãy làm như sau: Coi như ta không th y n y. v y nên ta có phương trình b c 2 theo x:( )2 12 26 12 0x x− + − = Ch c h n các b n u bi t gi i phương trình b c 2 này b ng máy CASIO. Ta b m ư c nghi m: 3 2 2 3 x x= ∨ = . Lúc này ta g i l i n y b ng cách thêm y vào sau các nghi m tìm ư c. 3 2 2 3 x y x y= ∨ = . Quy ng b m u vì m u là h ng s . ta có nhân t c n phân tích. Lưu ý là ( )2 2 12 26 12 0x xy y− + − = ⇔ ( )( )3 2 2 3 0x y x y− − = . N u gi i b t phương trình, b n nên chú ý n d u khi phân tích (Trư ng h p này là d u - : ( ) ( )( )2 2 12 26 12 2 3 2 2 3 0x xy y x y x y− + − = − − − = ) Khi g p d ng phương trình a th c có h ng s phía v ph i (ho c có th ưa c 2 phương trình v d ng có h ng s v ph i), Ta nhân c 2 v c a phương trình trên cho s v ph i c a phương trình dư i và nhân c 2 v c a phương trình dư i cho s phương trình trên. Sau ó tr v theo WWW.MATHVN.COM
  • 12. 11 v . M c ích c a phương pháp này là quy h v phương trình tích sau ó ti n hành phân tích. H u h t các lo i phương trình a th c u gi i ư c theo cách này! Bài t p t luy n Bài 1. 4 3 2 2 3 2 1 1 x x y x y x y x xy  − + =  − + = Bài 2. ( ) ( ) 2 2 4 1 1 2 x y x y x x y y y  + + + =  + + + + = Bài 3. ( ) ( ) 2 2 22 2 3 7 x xy y x y x xy y x y  − + = −  + + = − Bài 4. ( ) ( ) 3 2 3 2 log 2 3 5 3 log 2 3 5 3 x y x x x y y y y x  + − − =  + − − = Bài 5. ( ) ( ) 2 2 1 3 0 5 1 0 x x y x y x  + + − =   + − + =  Bài 6. 9 9 25 25 16 16 1x y x y x y  + =  + = + Bài 7. 4 3 2 2 2 2 2 9 2 6 6 x x y x y x x xy x  + + = +  + = + Bài 8. 2 2 2 1 7 1 13 xy x y x y xy y + + =  + + = Bài 9. ( ) 3 4 1 8 1 x y x x y  + − = −  − = Bài 10. 2 2 2 2 2 3 2 3 y y x x x y  + =   + =  Bài 11. 3 1 1 2 1 x y x y y x  − = −   = + WWW.MATHVN.COM
  • 13. 2 Bài III: Phương trình lư ng giác. M t s công th c lư ng giác c n nh : 1. 2 2 2 2 2 2 1 1 sin x cos x 1;1 tan ;1 cot . cos sin x x x x + = + = + = 2. sin cos 1 tanx ;cot x ;tan cos sin cot x x x x x x = = = . 3. Công th c c ng: sin( ) sin cos cos cos( ) cos cos sin sin a b a b asinb a b a b a b ± = ± ± = 4. Công th c nhân ôi: sin2x = 2sinxcosx 5. cos2x = cos2 x – sin2 x = 2 cos2 x – 1 = 1 - 2 sin2 x 6. Công th c h b c: 2 21 cos2 1 cos2 cos ;sin 2 2 x x x x + − = = 7. Công th c nhân ba: Sin3x = 3sinx – 4sin3 x; cos3x = 4cos3 x – 3cosx. 8. Công th c bi u di n theo tanx: 2 2 2 2 2tan 1 tan 2tan sin 2 ;cos2 ;tan 2 1 tan 1 tan 1 tan x x x x x x x x x − = = = + + − 9. Công th c bi n i tích thành t ng ( ) ( ) ( ) 1 cos cos cos( ) cos( ) 2 1 sin sin cos( ) cos( ) 2 1 sin cos sin( ) sin( ) 2 a b a b a b a b a b a b a b a b a b = − + + = − − + = − + + 10.Công th c bi n i t ng thành tích sin sin 2sin cos 2 2 sin sin 2cos sin 2 2 cos cos 2cos cos 2 2 cos cos 2sin sin 2 2 x y x y x y x y x y x y x y x y x y x y x y x y + − + = + − − = + − + = + − − = − WWW.MATHVN.COM
  • 14. 3 Cách gi i các phương trình lư ng giác trong thi i h c: Lưu ý trư c khi gi i : Các phương trình lư ng giác trong thi i h c nhìn qua m t h c sinh thư ng r t khó khăn ph c t p nhưng chúng u quy v nh ng phương trình ơn gi n. thi i h c các năm u xoay quanh bi n i v d ng phương trình tích, t n ph . Năm 2009, thi có bi n i hơn ó là phương trình cu i bi n i v d ng công th c c ng. Nhìn chung phương pháp gi i d ng toán này là các em h c thu c các công th c trên ây và rèn luy n kĩ năng phân tích a th c thành nhân t … GI I M T S THI TIÊU BI U: 1. Gi i phương trình: 2sin 2 4sin 1 0 6 x x π  − + + =    (1) Gi i: (1) 3sin 2 cos2 4sin 1 0x x x− + + = ( ) 2 2sin 3 cos2 2 2sin 0x x x+ − = ( )2sin 3 cos sin 2 0x x x− + = sinx 0 1 3 cos sin 1 cos cos 2 6 x k x x x x π π = ⇔ =    − = − ⇔ + =    5 2 6 7 2 6 x k x k x k π π π π π   =   = +   −  = +  2. Tìm nghi m trên kho ng (0; π ) c a phương trình : Gi i: Tìm nghi m ( )0,∈ π Ta có 2 2x 3 4sin 3 cos2x 1 2cos x 2 4 π  − = + −    (1) (1) ( ) 3 2 1 cosx 3 cos2x 1 1 cos 2x 2 π  ⇔ − − = + + −    (1) 2 2cosx 3 cos2x 2 sin2x⇔ − − = − (1) 2cosx 3 cos2x sin2x⇔ − = − . Chia hai v cho 2: (1) ⇔ − = − 3 1 cosx cos2x sin2x 2 2 ( )cos 2x cos x 6 π  ⇔ + = π −    ( ) ( ) π π π ⇔ = + = − + π 5 2 7 x k a hay x h2 b 18 3 6 2 2 3 4sin 3 cos2 1 2cos ( ) 2 4 x x x π − = + − WWW.MATHVN.COM
  • 15. 4 Do ( )x 0,∈ π nên h nghi m (a) ch ch n k=0, k=1, h nghi m (b) ch ch n h = 1. Do ó ta có ba nghi m x thu c ( )0,π là 1 2 3 5 17 5 x ,x ,x 18 18 6 π π π = = = 3. . Gi i phương trình : 3 2 2 cos ( ) 3cos sin 0 4 x x x π − − − = (2) Gi i: (2) 3 2 cos x 3cosx sinx 0 4 π   ⇔ − − − =      ( )⇔ + − − = ⇔ + + + − − = 3 3 3 2 2 cosx sinx 3cosx sinx 0 cos x sin x 3cos xsinx 3cosxsin x 3cosx sinx 0 = ⇔  − = 3 cosx 0 sin x sinx 0 ≠  + + + − − − − = 2 3 2 3 cosx 0 hay 1 3tgx 3tg x tg x 3 3tg x tgx tg x 0 ⇔ =2 sin x 1 =haytgx 1 x k 2 π ⇔ = + π hay π = + πx k 4 4. . Gi i phương trình : 2 2 cos2 1 ( ) 3 2 cos x tg x tg x x π − + − = ( d b kh i B 2005) Gi i: (2) 2 2 2 2sin x cotgx 3tg x cos x − ⇔ − − = π ⇔ − − = ⇔ = − ⇔ = − ⇔ = − + π ∈2 31 tg x 0 tg x 1 tgx 1 x k ,k Z tgx 4 PHƯƠNG PHÁP T N PH TRONG PHƯƠNG TRÌNH LƯ NG GIÁC: A. t t=sinx Cos2 x= 1 – sin2 x = 1-t2 t∈[-1;1] Tan2 x = 2 2 sin cos x x = 2 2 1 t t− Cos2x = 2 1 2sin x− = 1-2t2 Sin3x = 3 3 3sin 4sin 3 4x x t t− = − B. t t = cosx 2 2 2 sin 1 cos 1x x t= − = − 2 cos2 2 1x t= + 2 2 2 2 2 sin 1 tan cos x t x x t − = = 3 3 cos3 4cos 3cos 4 3x x x t t= − = − C. t t= tanx WWW.MATHVN.COM
  • 16. 5 1 cot x t = 2 2 1 cos 1 x t = + 2 2 2 sin 1 t x t = + 2 2 1 cos 2 1 t x t − = + 2 1 sin2x=2t 1 t     +  2 2 t an2 1 t x t = + sin cos tan sin cos tan a x b x a x b at b c x d x c x d ct d + + + = = + + + D. t t=sinx ± cosx t∈ 2; 2 −  sinxcosx 2 1 2 t − = ± sin2x= ( )2 1t± + ( )( ) 2 3 3 3 2 2 1 3 sin cos sin cos sin cos sin cos 1 2 2 t t x x x x x x x x t  − − + = + + − = − =    NGUYÊN T C CHUNG GI I PHƯƠNG TRÌNH LƯ NG GIÁC Bi n i: t t Phân tích thành tích Nguyên t c : Lũy th a H b c Tích T ng T ng Tích Bi n i không ư c thì i bi n. GI I M T S THI TIÊU BI U: Bài 1. 2cos2 1 cot 1 sin sin 2 1 tan 2 x x x x x − = + − + Gi i: t t=tanx, pt tr thành: ( ) 2 2 2 2 2 1 11 1 2 1 0; 1 1 1 2 1 t t t t t t t t t t  −   + − = + − ≠ ≠ − + + + 3 2 2 3 2 1 0t t t⇔ − + − = 1t⇔ = tan 1 4 x x k π π⇔ = ⇔ = + Bài 2. cos3 cos 2 cos 1 0x x x+ − − = Gi i: t t=cosx, pt tr thành: 3 2 4 3 2 1 1 0t t t t⇔ − + − − − = WWW.MATHVN.COM
  • 17. 6 cos 11 21 cos cos 32 xt xt π = ±= ±  ⇔ ⇔−  ==   2 2 3 x k x k π π π = ⇔  = ± +  Bài 3. Gi i phương trình: 1 sin 1 cos 1x x− + − = ( thi d b 2 A – 2004) (1) Gi i: (1) 1 sin cos 2 (1 sin )(1 cos ) 0x x x x− − + − − = t t=sinx +cosx ⇔ 2 1 sin 2 t xcosx − = Pt tr thành: 2 1 1 2 1 0 2 t t t − − + + − = 2 2 2 2 1 4 2 2 4 ( 1) 0 1t t t t t t⇔ − + = + − − ⇔ − = ⇔ = Sinx+cosx =1 2 sin 1 4 x π  + =    sin sin 4 4 x π π    + =        x kπ= Bài 4. ( ) 2 2cos sin 6tan 1 sin 2 1 sin x x x x x + + − = + Gi i: t t=sinx [ 1;1]t ∈ − pt tr thành: ( ) 2 2 2 2 1 6 1 2 6 1 0 1 1 t t t t t t t t − + + − = ⇔ − − = + − 2 1 6 1 sin 52 22 1 6 sin sin 3 1 arccos 2 3 x k t x x k xt x k π π π π α π  = + =  = ⇔ ⇔ ⇔ = + −  ==  − = +  Bài 5. 6 6 1 sin cos cos8 4 x x x+ = (1) Gi i: (1) 23 1 3 1 cos4 1 1 sin 2 cos8 1 cos8 4 4 4 2 4 x x x x −  − = ⇔ − =    t t=cos4x [ 1;1]t ∈ − pt tr thành: ( )2 2 4 3 1 1 16 42 4 1 2 1 3 34 2 4 2 4 4 16 42 k xt x k t t k x k xt π ππ π π π π π   = += = + −  − = − ⇔ ⇔ ⇔     −  = + = +=   WWW.MATHVN.COM
  • 18. 7 Bài t p t luy n 1 1 sin 2x sin x 2cot g2x 2sin x sin 2x + − − = 2 x3 cos2 42 x cos 42 x5 sin =      π −−      π − 2 2cos x 2 3sinx cosx 1 3(sinx 3 cosx)+ + = + gxcottgx xsin x2cos xcos x2sin −=+ ( )( ) 1 2cos 1 sin sin2 cos2 2 x x x x− + − = ( )( )2sin 1 2cos 1 1x x+ − = ( )3 3 sin cos 2 1 sin cosx x x x+ = − 2sin cos cos 1 2 x x x− = 4 4 3 sin cos cos .sin 3 0 4 4 2 x x x x π π    + + − − − =        Cho phương trình: 2sin cos 1 sin 2cos 3 x x a x x + + = − + (2) ( d b kh i a 2002) 1. gi i phương trình khi a= 1 3 2. tìm a phương trình (2) có nghi m. 2 tan cos cos sin 1 tan tan 2 x x x x x x   + − = +    ( )2 4 4 2 sin 2 sin3 tan 1 cos x x x x − + = WWW.MATHVN.COM
  • 19. 8 Bài IV: Tích Phân Lưu ý trư c khi gi i thi: Tích phân là bài toán r t thư ng xu t hi n trong thi i h c. K t năm 2002, khi b t u ti n hành thi “Ba chung” các d ng toán tích phân và ng d ng luôn xu t hi n và là câu 1 i m. Bài t p ph n này không quá khó nhưng v n ph i òi h i kĩ năng phán oán, phân tích , và n m rõ ư c các cách làm bài toán tích phân cơ b n như i bi n s và tính theo tích phân t ng ph n… các em cùng theo dõi các ví d dư i ây. NGUYÊN T C CHUNG GI I BÀI TOÁN TÍCH PHÂN: G m có 2 phương pháp chính: A. I BI N: • i bi n lo i 1: ( )( ) ( ). 'f u x u x dx t t=u(x) Chú ý: Các bi u th c có quan h o hàm GI I CÁC VÍ D : VD 1. Tính tích phân: 2 2 0 sin 2 3 cos x I x π = +∫ Gi i: t 2 3 cost x= + ( )2cos sindt x x dx⇒ = − 2sin 2dt xdx⇒ = − X 0 2 π t 4 3 4 3 4 4 ln ln 3 3 dt I t I t − = = ⇒ =∫ VD2. Tính tích phân: 6 2 dx I 2x 1 4x 1 = + + + ∫ ( DB 1A – 2006) Gi i: t t= 2 1 4 1 4 1 2 x t x tdt dx+ ⇒ = + ⇒ = X 2 6 t 3 5 ( ) ( ) ( ) 5 5 5 2 2 3 3 3 51 1 1 3 1 ln 1 ln 31 1 2 121 1 t dt dt dt t t tt t + −   = − = + + = − + + + + ∫ ∫ ∫ VD3. Tính tích phân: 4 2 0 cos 1 tan dx I x x π = + ∫ Gi i: WWW.MATHVN.COM
  • 20. 9 t t= 2 2 1 tan 1 tan 2 cos dx x t x tdt x + ⇒ = + ⇒ = X 0 4 π t 1 2 2 2 1 1 2 2 2 2 2 2 2 1 tdt I dt t t = = = = −∫ ∫ VD 4. Tính tích phân: e 1 3 2ln x I dx. x 1 2ln x − = +∫ Gi i: t t= 2 1 2ln 1 2ln dx x t x tdt x + ⇒ = + ⇒ = X e 1 t 2 1 ( ) ( ) 22 2 2 1 1 3 1 10 2 11 4 3 t I tdt t dt t − − − = = − =∫ ∫ 1. i bi n lo i 2: B c t l n hơn b c m u: chia a th c B c t nh hơn b c m u: Xét quan h o hàm ⇒ i bi n M u có nghi m⇒ Tách phân th c Hàm h u t (m u vô nghi m): ( )( ) 2 2 du u x a+ ∫ t u(x)=atant Hàm căn th c: ( )( ) 22 a u x+ ⇒ t u(x)=atant ( )( ) 22 u xa − ⇒ t u(x)=asint (ho c u(x)=asint) VD 5. Tính tích phân: I= 3 2 0 9 dx x +∫ Gi i: t x=3tan(t) ( )2 3 tan 1dx t dt⇒ = + X 0 3 t 0 4 π WWW.MATHVN.COM
  • 21. 10 ( ) ( ) 24 2 0 3 tan 1 1 4 3 129 tan 1 0 t dt I t t π π π+ = = = +∫ VD 6. Tính tích phân: ( ) 5 2 2 1 9 1 dx I x = − − ∫ Gi i: t x-1= 3sint 3cosdx tdt⇒ = X 1 5 2 t 0 6 π 6 6 6 2 2 0 0 0 3cos cos cos 6 cos 69 9sin 1 sin 0 tdt tdt tdt I t tt t π π π π π = = = = = − − ∫ ∫ ∫ VD 7. Tính tích phân: 3 2 2 1 3 dx I x x = + ∫ Gi i: t x= 3 tant ( )2 3 tan 1dx x dx⇒ = + X 1 3 t 6 π 3 π ( )2 3 32 222 2 2 26 6 1 3 tan 1 1 1 coscos 3 3 sinsin 13tan 3tan 3 cos cos dtt tdttI dx ttt t t π π π π + − = = = + ∫ ∫ ∫ ( )3 2 6 sin1 1 6 2 33 3 sin 3sin 9 6 d t I t t π π π π − = − = − =∫ WWW.MATHVN.COM
  • 22. 11 B. PHƯƠNG PHÁP TÍNH TÍCH PHÂN T NG PH N: Công th c: b b a a b udv uv vdu a = −∫ ∫ (1) Cách l y ph n các tích phân: Kí hi u P(x) là a th c. Khi g p hai d ng nguyên hàm sau ây, ta thư ng dùng phương pháp tích phân t ng ph n: D ng 1: ( )lnP x xdx∫ ta t u= ln x (Do lnx không có nguyên hàm) D ng 2: ( ). sin( ) cos( ) ax b e P x ax b dx ax b +     +   +  ∫ ta t u=P(x) V i cách y khi l y công th c 1 ta s ư c bài toán d n t i nguyên hàm ng d ng v i b c c a P(x) th p hơn… GI I CÁC VÍ D : VD 1. Tính tích phân: 2 0 I (x 1)sin2xdx. π = +∫ ( d b kh i D 2005) Gi i: t: ( ) 2 0 1 1 1 cos2 cos2 121 2 2 4sin2 cos2 02 u x du dx x I x xdx dv xdx v x π π π = + ⇒ = − + ⇒ = + = + − = ⇒ = ∫ VD 2. Tính tích phân: 2 1 I (x 2)lnx dx.= −∫ ( d b kh i D 2006) Gi i: t: ( ) 2 1 ln 2 2 2 du dx u x x dv x dx x v x  ==  ⇒  = −  = −  22 1 2 5 2 ln 2 ln 4 12 2 4 x x I x x dx     ⇒ = − − − = − +       ∫ VD 3. Tính tích phân: 2 4 0 sin xdx π ∫ Gi i: t t= 2 2x t x tdt dx⇒ = ⇒ = X 0 2 4 π t 0 2 π WWW.MATHVN.COM
  • 23. 12 2 0 2 sinB t tdt π = ∫ Tính 2 0 sinI t tdt π = ∫ t: sin cos u t du dt dv tdt v t = =  ⇒  = = −  2 0 cos cos cos 0cos0 sin 12 2 2 2 0 0 I t t tdt t π π π π π = − + = − + + =∫ B=2I=2 VD 4. Tính tích phân: A= 2 0 cosx e xdx π ∫ Gi i: t: sin cos x x u e du e dx dv xdx v x  = = ⇒  = − = −  2 2 2 02 0 0 0 cos cos cos cos0 cos 1 cos2 2 0 x x x x A e x e xdx e e e xdx e xdx π π π ππ π = − + = − + + = +∫ ∫ ∫ (1) Tính 2 0 cosx K e xdx π = ∫ t: cos sin x x u e du e dx dv xdx v x  = = ⇒  = =  2 2 0 sin sin2 0 x x K e x e xdx e A π ππ = − = −∫ Thay vào (1): 2 2 2 1 1 2 1 2 e A e A A e A π π π + = + − ⇒ = + ⇒ = VD 5. Tính tích phân: A= 2 0 sin cosx x xdx π ∫ Gi i: t: 22 sin cossin cos du dxu x v x xdxdv x xdx ==  ⇒  ==  ∫ Tính: 2 sin cosv x xdx= ∫ t : cos sint x dt xdx= ⇒ = − WWW.MATHVN.COM
  • 24. 13 V= 3 3 2 cos 3 3 t x t dt C C − − = + = − +∫ Ch n C=0 3 cos 3 x v⇒ = − V y 3 3 0 cos 1 1 cos 03 3 3 3 x A x xdx K π π π = − + = +∫ (1) Tính ( )3 2 0 0 cos 1 sin cosK xdx x xdx π π = = −∫ ∫ t t=sin(x) cosdt xdx⇒ = X 0 π t 0 0 ( ) 0 2 0 1 0K t dt= − =∫ Thay vào (1): 1 3 3 3 A K π π = + = VD 6. Tính tích phân: 2 3 sin 1 cos x x D dx x π π + = +∫ Gi i: 2 2 3 sin 2cos 2 x x D x π π + = ∫ t: ( ) 2 sin 1 cos 1 tan 2cos 2 2 u x x du x dx dv dx x x v = +  = +  ⇒ = =   V y: ( ) ( ) 2 3 3 32 sin tan 1 cos tan 1 2 2 2 3 2 3 3 x x D x x x dx K π π π π π π    = + − + = + − + −        ∫ (3) V i: ( ) 2 2 2 2 3 3 3 1 cos tan 2cos tan sin 2 2 2 x x x K x dx dx xdx π π π π π π = + = =∫ ∫ ∫ 12 cos 2 3 x π π = − = Thay vào (3) ta có: D= ( )9 2 3 18 π+ L i bình: tích phân t ng ph n ta có cách nh t u như sau: nh t “log” – nhì “ a” ( a th c) – tam “Lư ng” (Lư ng giác) – T “mũ”. Trong phép tính tích phân t ng ph n, g p phép nào ng trư c trong 4 phép trên, hãy t u b ng phép ó! WWW.MATHVN.COM
  • 25. 14 Bài t p t luy n Tính tích phân: 3 2 0 sin .I x tgxdx π = ∫ Tính tích phân: 7 3 0 2 1 x I dx x + = + ∫ Tính tích phân: 2 0 ln e I x xdx= ∫ Tính tích phân: 4 sin 0 ( cos )x I tgx e x dx π = +∫ Tính tích phân: 0 cos sinI x xdx π = ∫ Tính tích phân: 3 2 2 6 tan cot 2I x x dx π π = + −∫ Tính tích phân: ( ) 2 2 2 1 cos2I x dx π π− = +∫ Tính tích phân: 3 6 sin 4 sin3 tan cot 2 x x I dx x x π π = +∫ Tính tích phân: 10 5 dx I x 2 x 1 = − − ∫ Tính tích phân: e 1 3 2ln x I dx. x 1 2ln x − = +∫ Tính tích phân: 2 0 sin 1 sin x x I x π = +∫ Tính tích phân: 36 0 sin sin cos2 x x I x π + = ∫ Tính di n tích hình ph ng gi i h n b i parabol ( ) 2 P : y x x 3= − + và ư ng th ng d : y 2x 1.= + Tính di n tích hình ph ng gi i h n b i các ư ng: ( ) ( ) ( ) 2 2 27 1 ; 2 ; 3 27 x C y x C y C y x = = = WWW.MATHVN.COM
  • 26. 15 Bài V:Các bài toán liên quan n ng d ng c a o hàm và th hàm s . Lưu ý trư c khi gi i thi: Các bài toán d ng này là câu chi m 1 i m, thư ng n m câu th 2 sau ph n kh o sát hàm s trong thi i h c. Mu n gi i ư c d ng toán này ta c n n m v ng các lí thuy t v s tăng, gi m hàm s , các v n v c c tr , s tương giao gi a hai th ( i u ki n ti p xúc c a hai ư ng cong)… Các ví d dư i ây s trình bày m t cách có h th ng các v n nêu trên và cách gi i ơn gi n và d hi u nh t. Các b n tham kh o các ví d sau ây: I: S TĂNG GI M C A HÀM S : Nh c l i ki n th c: Cho hàm s ( )y f x= có o hàm trên mi n I ( ) 0;f x x I≥ ∀ ∈ Hàm s tăng ( ) 0;f x x I≤ ∀ ∈ Hàm s gi m VD 1. Cho hàm s : ( ) ( )3 2 21 2 3 y f x x mx m m x= = − + + − Tìm m hàm s : a. Tăng trên R b. Gi m trên (0;2) c. Tăng trên ( )4;+∞ d. Gi m trên o n có dài b ng 2 e. Tăng trên 2 kho ng ( );4−∞ và ( )2;+∞ Gi i: TX : D R= 2 2 ' 2 2 ' 2y x mx m m m= − + + − ⇒ ∆ = − + a. Ycbt ' 0 2 0 2m m∆ ≤ ⇔ − + ≤ ⇔ ≥ b. Ycbt ( ) ( ) 2 2 ' 0 0 2 0 1 ' 2 0 3 2 0 y m m m y m m  ≤  + − ≤  ⇔ ⇔ ≤  ≤ − + ≤  Vì c. Ycbt TH1: ' 0 2 0 2m m∆ ≤ ⇔ − + ≤ ⇔ ≥ x -∞ 0 2 +∞ F’(x) + - + F(x) WWW.MATHVN.COM
  • 27. 16 TH2: ( ) 2 2' 0 ' 4 0 9 14 0 4 4 2 m y m m mS   <∆ >    ≥ ⇔ + + ≥    < − <  V y ycbt ( ); 7 2 m m  ∈ −∞ −  ≥ d. Ycbt 1 2 2 2 2 2 2 2 2 1 1x x m m m a ∆ ⇔ − = ⇔ = ⇔ − + = ⇔ − + = ⇔ = Chú ý: X1= 'b a − + ∆ ; x2= 'b a − − ∆ 1 2x x⇒ − = 2 a ∆ e. Ycbt ( ) ( ) 2 2 ' 0 2 ' 0 2 0 2' 4 0 9 14 0 2 1' 2 0 3 2 0 4 24 2 2 m m my m m my m m S m ∆ ≤ ≥ ∆ >  − + >  ≥≥  ⇔ ⇔ ⇔+ + ≥   − ≤ ≤  − ≥ − + ≥    − < <− < <  VD 2. Cho hàm s ( ) 2 2 2 21 3 3 m y x mx m m x − = + + − + tìm m hàm s : a. Gi m trên mi n xác nh. b. Tăng trên (0;2) c. Gi m trên ( )6;+∞ d. Tăng trên o n có dài b ng 2 e. Gi m trên 2 kho ng ( );0−∞ và ( )6;+∞ Gi i: MX : D=R 2 2 ' 2y x mx m m= − + + − ' m∆ = a. Gi m trên mi n xác nh. ' 0 0m⇔ ∆ ≤ ⇔ ≤ b. Tăng trên (0;2) ( ) ( ) 2 2 ' 0 0 0 1 ' 2 0 5 4 0 y m m m y m m  ≥ − + ≥  ⇔ ⇔ ⇔ =  ≥ − + + ≥  c. Gi m trên ( )6;+∞ TH1: ' 0 0m∆ ≤ ⇒ ≤ (Rõ ràng vì gi m trên D cũng có nghĩa là gi m trên ( )6;+∞ ) WWW.MATHVN.COM
  • 28. 17 TH2: ( ) 2 0' 0 ' 6 0 13 36 0 6 6 2 m y m m mS   >∆ >    ≤ ⇔ − + − ≤    < <  V y YCBT [ ] 0 4 0;4 m m m ≤ ⇔ ⇔ ≤ ∈ d. Tăng trên o n có dài b ng 2 1 2 2 ' 2 2 2 2 1x x m m a ∆ ⇔ − = ⇔ = ⇔ = ⇔ = e. Gi m trên 2 kho ng ( );0−∞ và ( )6;+∞ TH1: (Gi m trên D): ' 0 0m∆ ≤ ⇔ ≤ TH2: ( ) ( ) ' 0 ' 0 0 1 4' 6 0 0 6 2 y my S ∆ ≥  ≤ ⇔ ≤ ≤ ≤   < <  Tóm l i: ycbt 0 1 4 m m ≤  ≤ ≤ II: C C TR C A HÀM S Nh c l i ki n th c: X X0 Y’ + 0 - Y C c i X X0 Y’ - 0 + Y C c Ti u Bài 1: Cho (Cm) ( )3 2 2 31 2 1 3 y x mx m x m m= − + − + − . Tìm m : a. Tìm m C có i m c c i n m trên Oy b. Hàm s t C và CT t i i m có hoành <1 WWW.MATHVN.COM
  • 29. 18 c. Hàm s t C và CT t i i m có hoành >-1 d. Hàm s t C và CT t i i m có hoành n m trong [-2;3] e. Hàm s t C và CT t i i m có hoành dương f. Hàm s t C và CT t i i m có hoành trái d u nhau g. Hàm s t C và CT t i x1;x2 sao cho ( )3 3 1 2x x+ nh nh t Gi i: MX : D=R 2 2 ' 2 2 1y x mx m= − + − 2 ' 1m∆ = − + ' 0∆ > : X −∞ X1 X2 +∞ Y’ + 0 - 0 + Y C CT a. Ycbt Hàm s t c c i t i x=0 ( ) 2' 0 0 2 1 0 2 200 2 y m mS m  =  − = ⇔ ⇔ ⇔ =  ><   b. Ycbt : ( ) 2 2 1 1 0' 0 0 ' 1 0 2 2 0 1 1 1 1 2 m m m y m m m mS m   <  − + >∆ >   <  ⇔ > ⇔ − > ⇔   >  <  < < ⇒ 1 0m− < < c. Ycbt Hàm s t C và CT t i i m có hoành >-1 ( ) 2 2 1 1' 0 0 ' 1 0 2 2 0 1 1 1 1 2 m m m y m m m mS m   <   <∆ >   >  ⇔ − > ⇔ + > ⇔ ⇔   < −  > −  > − > − 0 1m< < d. Hàm s t C và CT t i i m có hoành n m trong [-2;3] Ycbt ( ) ( ) ( ) ( ) 2 2 ' 0 1 ' 2 0 2 4 3 0 1 1' 3 0 2 6 8 0 2 3 2 3 2 m y m m m my m m m S m ∆ >  < − ≥ + + ≥ ∀ ⇔ ⇔ ⇔ − < < ≥ − + ≥ ∀   − ≤ ≤ − ≤ ≤ e. Hàm s t C và CT t i i m có hoành dương WWW.MATHVN.COM
  • 30. 19 Ycbt ( ) 2 1 1 21 1' 0 22' 0 0 2 1 0 1 22 0 0 2 2 0 m m m y m m m mS m − < <    − < <∆ >  ≤ −  ⇔ ≥ ⇔ − ≥ ⇔ ⇔ ≤ <     > ≥ <    > f. Hàm s t C và CT t i i m có hoành trái d u nhau ( ) 2 ' 0 0 2 2 2 1 0 2 2' 0 1 y m m m  < − ⇔ ⇔ − < ⇔ < < ∆ > ⇒ < g. Hàm s t C và CT t i x1;x2 sao cho ( )3 3 1 2x x+ nh nh t Ycbt ( ) ( ) 3 1 2 1 2 1 2 ' 0 3 minP x x x x x x ∆ > ⇔  = + − + → (1) V i 2 1 2 1 2 2 1 2 x x m x x m  = −  + = V y ta có (1) ( ) ( ) 2 3 2 1 0 2 3 2 1 .2 min m P m m m − + > ⇔  = − − → 3 1 1 4 6 min m P m m − < < ⇔  = + → 2 2 2 ' 12 6 ' 0 2 2 m P m P m  = ⇒ = − + ⇒ = ⇔  = −  B ng bi n thiên: X −∞ -1 2 2 − 2 2 1 +∞ Y’ - 0 + 0 - Y -2 2 2 - 2 2 2 min 2 2P = − khi 2 2 m − = L i bình: Có l các b n ang th c m c: “T i sao l i có nh ng l i gi i ng n g n và d dàng như v y?” Bí quy t n m bi u th c y’ và d u c a nó. Lúc này, t t c yêu c u bài toán (ycbt) liên quan n c c tr u n m n dư i nh ng d u + - c a y’. Và tr c quan hơn n a, ta th y ư c hư ng i c a mình qua b ng bi n thiên. Tôi s minh h a kĩ câu d c a ví d trên ây: Ycbt : Hàm s t C và CT t i i m có hoành n m trong [-2;3] - có c c i và c c ti u y’=0 có hai nghi m ' 0⇒ ∆ > - V b ng bi n thiên: WWW.MATHVN.COM
  • 31. 20 X −∞ -2 X1 2 S X2 3 +∞ Y’ + 0 - 0 + Y C CT T ó ta có ( ) ( ) ' 2 0 ' 3 0 y y  − ≥  ≥ . V y là i u ki n th 2 ã ư c bi u hi n r t rõ ràng trên b ng bi n thiên. ây th c ra là xét quan h v d u c a h s a: ( )af α nhưng ây khi ta ã bi t rõ d u c a a thì ch c n t d u ó vào trư c ( )f α là ư c. ây cũng có th là bư c rút g n th i gian mà các em nên làm, tránh khai tri n m t th i gian. - 2 S là t ng hai nghi m X1;X2 c a phương trình y’=0 hay b ng 2 b a − . Rõ ràng n u X1;X2 n m trong [-2;3] thì 2 S cũng ph i n m trong o n này. Vì 2 b a − là giá tr có th rút ra d dàng t phương trình g c nên ta ch n giá tr trung bình này làm i u ki n. Nút th t th 3 ư c g b . - L i khuyên ó là: khi g p nh ng d ng toán như trên h c sinh hãy v b ng bi n thiên như trên ra gi y nháp sau ó tùy theo câu h i mà i n các thông s thích h p vào b ng. t ó m i hư ng gi i u ư c phơi bày! Tôi có tham kh o qua m t vài tài li u c a các th y cô giáo thì th y ph n l n các sách u trình bày l i gi i m t cách máy móc, không tr c quan, nhi u lúc có th coi là lu n qu n. . Ví d : tìm m hàm s y=f(x) tăng trên (1;+ ∞), các th y cô trình bày trong sách cũng như trên l p theo phương pháp Min- Max, xét nhi u trư ng h p… Nh ng cách gi i ó không ph i là sai tuy nhiên i u ó ôi khi làm khó các em h c sinh trong quá trình tư duy tìm trư ng h p, nh t là các em h c sinh trung bình. Phương pháp xét d u trình bày trên ây v a ng n g n rõ ràng l i không b sót trư ng h p. bài toán ư c ơn gi n hóa. Cách gi i trên cũng áp d ng ư c cho hàm s 2 2 ' ' ' ax bx c y a x b x c + + = + + vì d ng o hàm ( ) 2 22 2 ' ' ' ' ' ' ' ' ' a b a c b c x x a b a c b c y a x b x c + + = + + . Trong trư ng h p này, tùy bi u th c m u có nghi m hay không ta t thêm trư ng h p. Vì m u th c ≥0 nên khi xét d u ta ch c n xét d u t s tương t như các ví d trình bày trên. D ng hàm s này ã không còn thông d ng ( ch gi i thi u sơ lư c trong sách giáo khoa) nên xu hư ng ra ch xoay quanh 3 hàm là: b c 3, trùng phương và ' ' ax b y a x b + = + . Bài 2: Cho (Cm): ( )3 2 3 3 1 4y x mx m x= − + − + nh m : a. C(m) có hai i m c c tr A;B sao cho AB th ng hàng v i C(1;-1) b. C(m) có hai i m c c tr A;B sao cho AB = 2 5 c. C(m) có hai i m c c tr A;B sao cho AB cách u : 2y∆ = Gi i: WWW.MATHVN.COM
  • 32. 21 MX : D=R T a 2 i m c c tr th a h : ' 0 ( ) y y f x =  = V y: 2 ' 2 1 0y x x m= − − + = ( )3 2 3 3 1 4y x mx m x= − + − + ( )( )2 0 2 1y x x m cx d ax b ax b⇒ = − − + + + + = + ( )2 2 1 ( 1) 2 5y x x m x mx m⇔ = − − + − − − + ( ) ( ) 2 2 1 0 1 2 5 2 x x m y mx m  + − + = ⇔  = − − + C(m) có hai c c tr (1) ph i có 2 nghi m phân bi t ' 0⇒ ∆ ≥ 0m⇒ > a. C(m) có hai i m c c tr A;B sao cho AB th ng hàng v i C(1;-1) (2) ⇒ phương trình ư ng th ng qua hai i m c c tr là 2 5y mx m= − − + Vì AB th ng hàng v i C(1;-1) ⇒ C ∈ AB nên: -1=-2m.1-m+5 2m⇔ = V y v i m=2 AB th ng hàng v i C(1;-1) b. C(m) có hai i m c c tr A;B sao cho AB = 2 5 ( ) 2 1 2 ' 1 2x x m a ∆ ⇒ − = = ( ) ( )2 1 2 12 2 4y y m x x m m⇒ − = − − = − ( ) ( ) 2 2 2 1 2 1 2 5AB x x y y⇒ = − + − = 2 1 16 4 20 5 4 m m m m = ⇒ + = ⇔  = −  So sánh k ⇒ 1m = c. C(m) có hai i m c c tr A;B sao cho AB cách u : 2y∆ = Ycbt ( ) ( ); ;d A d B⇔ ∆ = ∆ v i : 2y∆ = ( ) 1 2 1 2 1 2 1 2 1 2 2 2 2 2 2 2 4 y y y y y y y y y y − = − = ⇔ − = − ⇔ ⇔ − = − − + = ( ) ( ) ( )1 2 1 22 5 2 5 4 2 2 10 4mx m mx m m x x m⇔ − − + + − − + = ⇔ − + − + = 2 .2 2 10 4 1m m m⇔ − − + = ⇔ = Bài 3: Cho (Cm): ( )3 2 3 3 1y x x m x= + − − nh m : a. C(m) có hai i m c c tr A;B sao cho OAB∆ vuông t i O b. C(m) có hai i m c c tr A;B n m khác phía v i tr c Ox c. C(m) có hai i m c c tr A;B cùng phía v i tr c Oy d. C(m) có hai i m c c tr A;B n m cách u ư ng th ng y=5 e. Có ư ng th ng i qua hai i m c c tr cách g c t a m t kho ng b ng 1 f. Có ư ng th ng i qua hai i m c c tr ti p xúc v i ư ng tròn ( ) ( ) 2 2 1 1 4x y− + − = g. Có ư ng th ng i qua hai i m c c tr t o v i hai tr c t a m t tam giác cân h. Có ư ng th ng i qua hai i m c c tr t o v i hai tr c t a m t tam giác có di n tích =8 Gi i: WWW.MATHVN.COM
  • 33. 22 CT CD x y 1x 1x 5y = MX : D=R T a 2 i m c c tr th a h : ' 0 ( ) y y f x =  = ( ) ( )( ) 2 3 2 2 ' 2 1 0 3 3 3 1 2 1 1 2 1 y x x m y x x m x x x m x mx m  = + − + = ⇔   = + − − = + − + + − + − ( ) ( ) 2 2 1 0 1 2 1 x x m y mx m  + − + = ⇔  = − + − ∆ C(m) có hai c c tr (1) ph i có 2 nghi m phân bi t ' 0⇒ ∆ ≥ 0m⇒ > (*) a. C(m) có hai i m c c tr A;B sao cho OAB∆ vuông t i O Ycbt OA OB⇔ ⊥ .OAOB⇔ v i ( ) ( ) ; ; A A B B OA x y OB x y  =  = ( )( )1 2 1 2 1 2 1 20 2 1 2 1 0x x y y x x mx m mx m⇔ + = ⇔ + − + − − + − = ( )( ) ( ) 22 2 1 2 1 2 1 24 2 2 1 0x x m x x m m x x m⇔ + + − + + + − = ⇔ ( ) ( ) ( ) ( ) 22 2 1 4 1 2 2 . 2 1 0m m m m m m− + + − + + − + − + − = 3 2 4 9 7 2 0m m m⇔ − + − + = ( )( )2 vì 7 4 5 2 1 0 VN m m m ∆=− ⇔ − + − − = 1m⇔ = (th a i u ki n(*)) b. C(m) có hai i m c c tr A;B n m khác phía v i tr c Ox Ycbt 1 2. 0y y⇔ < ( )( )1 22 1 2 1 0mx m mx m⇔ − + − − + − < ( )( ) ( ) 22 2 1 2 1 24 2 2 1 0m x x m m x x m⇔ + − + + + − < ( ) ( ) ( ) 22 2 4 1 2 2 2 1 0m m m m m⇔ − + − − + + − < ( )( ) 23 2 0 4 9 6 1 0 4 1 1 0m m m m m ≥ ⇔ − + − + < ⇔ − + − < 1 4 1 m m  > ⇔   ≠ c. C(m) có hai i m c c tr A;B cùng phía v i tr c Oy Ycbt 1 2 0x x⇔ > ( 1x cùng d u v i 2x ) 1 0 1m m⇔ − + > ⇔ < d. C(m) có hai i m c c tr A;B n m cách u ư ng th ng y=5 Ycbt : y=5 c t (Cm) t i trung i m AB. M là trung i m AB có t a 1 2 ; 2 1 2 x x mx m +  − + −    ( )1;3 1M m⇒ − − 5 3 1 2Ycbt m m⇔ = − ⇔ = So sánh v i i u ki n (*) ta th y m=2 là k t qu c n tìm. e. Có ư ng th ng i qua hai i m c c tr cách g c t a m t kho ng b ng 1 : 2 1 : 2 1 0y mx m mx y m∆ = − + − ⇔ ∆ + − + = WWW.MATHVN.COM
  • 34. 23 Ycbt ( ); 1d O⇔ ∆ = ( ) 2 2 2 .0 0 1 1 2 1 m m m + − + ⇔ = + ( ) ( ) 2 2 2 2 1 2 1 3 2 0m m m m⇔ − + = + ⇔ + = 0 2 3 m m = ⇔ −  =  So sánh v i i u ki n m>0 ta nh n th y không có giá tr m th a mãn yêu c u bài toán. f. Có ư ng th ng i qua hai i m c c tr ti p xúc v i ư ng tròn ( ) ( ) 2 2 1 1 4x y− + − = Ycbt ( );d I R⇔ ∆ = v i tâm I(1;1) và R=2 : 2 1 0mx y m∆ + + − = ( ) 2 2 .1 1 1 2 2 1 m m m + − + ⇒ = + ( ) 2 2 2 2 16 4 15 4 0m m m m⇔ + = + ⇔ − + = 0 4 15 m m = ⇔  =  So sánh v i (*) ta nh n 4 15 m = g. Có ư ng th ng i qua hai i m c c tr t o v i hai tr c t a m t tam giác cân G i M là giao i m c a ∆ và Ox: 2 1 0 1 ;0 0 2 mx m m M y m − + − = −  ⇒ ⇒  =   G i N là giao i m c a ∆ và Oy: ( ) 2 .0 1 0; 1 0 y m m N m x = − + − ⇒ ⇒ − = Ycbt 1 1 1 1 . 1 0 2 2 M N m x y m m m m  − ⇔ = ⇔ = − ⇔ − − =     1 1 2 1 2 m m m   =  ⇔ =   −  =  D th y v i m=1, ∆ i qua g c t a , v i m= 1 2 − không th a (*) nên lo i. V y ta ch n 1 2 m = h. Có ư ng th ng i qua hai i m c c tr t o v i hai tr c t a m t tam giác có di n tích =8 Ycbt: 1 1 1 . 2 8 2 OMN M NS OM ON x y∆⇔ = ⇔ = ( ) 2 11 1 1 . 1 4 2 4 2 mm m m m −− ⇔ = − ⇔ = ( ) 2 2 2 2 1 1 2 2 2 1 2 m m m m m m m m VN  =  − + = ⇔   = ⇔    −  − + =  So sánh (*) v y có hai giá tr m th a mãn: m=2 và m=0.5 WWW.MATHVN.COM
  • 35. 24 III: S TƯƠNG GIAO GI A HAI TH Nh c l i ki n th c: Cho: ( ) ( )1 2: ; :C y f x C y g x= = S giao i m c a C1 và C2 là s nghi m c a phương trình hoành giao i m: ( ) ( )f x g x= c bi t khi C1 ti p xúc C2: ( ) ( ) ( ) ( )' ' f x g x f x g x =  = Lưu ý: Không ư c s d ng i u ki n nghi m kép làm d ng toán ti p xúc c a hai th . hi u rõ hơn, ta hãy n v i các ví d sau: Bài 1: Cho hàm s ( ) ( ) 2 3 2 : 2 1 m mx m C y m x − − = ≠ − − và ( ): 1d y x= − nh m (d) c t (Cm) t i hai i m phân bi t: a)Có hoành l n hơn -1 b)Có hoành nh hơn 2 c) Có hoành n ng trong kho ng [ ]2;3− d)Có hoành dương e)Có hoành trái d u. Gi i: Phương trình hoành giao i m gi a (Cm) và d: ( ) ( )22 3 2 1 : 2 1 3 3 0 1 mx m x g x x m x m x − − = − ⇔ − + + + = − x −∞ 1x 2 S 2x +∞ ( )g x + 0 - 0 + (d) c t (Cm) t i hai i m phân bi t g(x)=0 có hai nghi m phân bi t ( ) 2 ' 0 1 1 0 2 m m g m  > ∆ > ⇔  < −  ≠ ⇔ ≠ − (*) a)Có hoành l n hơn -1 Ycbt: ( )1 0 1 2 g S − >  ⇔  − <  ( ) 6 1 2 1 3 3 0 5 1 1 2 m m m m m − + + + + > >  ⇔  + > −  > − So sánh v i (*) ta k t lu n: 6 1 5 2 m m − < <  > b)Có hoành nh hơn 2 ( ) ( ) 2 0 4 4 1 3 3 0 3 0 3 1 11 22 2 g m m m m S m mm > − + + + > − + < <   ⇔ ⇔ ⇔    < <+ <<     WWW.MATHVN.COM
  • 36. 25 So sánh v i (*) ta k t lu n: 2 2 1 m m < − − < < − c) Có hoành n ng trong kho ng [ ]2;3− Ycbt: ( ) ( ) ( ) ( ) 11 2 0 4 4 1 3 3 0 7 3 0 9 6 1 3 3 0 2 3 22 1 3 2 3 2 mg m m g m m m mS m   ≥ − − ≥ + + + + ≥     ≥ ⇔ − + + + ≥ ⇔ ≤     − ≤ ≤− ≤ + ≤ − ≤ ≤  So sánh i u ki n (*) ta suy ra: 11 1 7 m − ≤ ≤ − d)Có hoành dương Ycbt: ( ) 0 3 3 0 1 1 0 10 2 g o m m S m m > + > ⇔ > − ⇔ ⇔  + ≥ ⇔ ≥ −≤   So sánh v i (*) ta suy ra: m>2 e)Có hoành trái d u. Ycbt: ( )0 0 3 3 0 1g m m< ⇔ + < ⇔ < − So sánh i u ki n (*) ( ) ( ); 2 2; 1m⇒ ∈ −∞ − ∨ − − Bài 2: Cho hàm s ( ) 1 : 1 x C y x + = − và ( ): 1d y mx= + Tìm m d c t (C): a)T i 2 i m phân bi t n m trên 2 nhánh c a th . b)T i 2 i m phân bi t n m trên cùng 1 nhánh c a th Gi i: Phương trình hoành giao i m c a (C) và d: ( ) 1 1 1 1 x mx x x + = + ≠ − ( ) ( )2 2 0 1g x mx mx⇔ = − − = a)T i 2 i m phân bi t n m trên 2 nhánh c a th . (Hình 1) Ycbt: phương trình (1) có hai nghi m phân bi t th a 1 21x x< < x −∞ 1x 1 tiem can dung 2x +∞ ( )g x Cùng d u m 0 Trái d u m 0 Cùng d u m ( ) ( ). 1 0 2 0 2 0 0m g m m m m m⇔ < ⇔ − − < ⇔ − < ⇔ > WWW.MATHVN.COM
  • 37. 26 ình1H ình 2H ình3H Lưu ý: Trư ng h p này không c n ph i xét bi t th c ∆ vì khi d c t C v 2 phía c a ti m c n ng x=1 thì m c nhiên phương trình ã có 2 nghi m, không c n thi t ph i xét ∆ b)T i 2 i m phân bi t n m trên cùng 1 nhánh c a th (Hình 2) Phương trình (1) có hai nghi m phân bi t th a: 1 2 1 2 1 1 x x x x < <  < < ( ) 2 0 8 0 . 1 0 2 0 m m m g m ∆ >  + < ⇔ ⇔  > − >  0 0 8 m m m <  ⇔ >  < − 8m⇔ < − Bài 3: Vi t phương trình ư ng th ng c t th : ( ) 3 : 3 2C y x x= − + t i 3 i m phân bi t A,B,C sao cho xA=2 và BC= 2 2 . Gi i: (hình 3) 2 4A Ax y= ⇒ = Phương trình ư ng th ng qua A(2;4) là ( ): ( ) : 2 4A Ay k x x y y k x∆ = − + ⇒ ∆ = − + L p phương trình hoành giao i m c a (C) và ∆ : ( ) ( )3 3 3 2 2 4 3 2 2x x k x x x k x− + = − + ⇔ − − = − ( )3 3 2 2 0x k x k⇔ − + + − = ( )( )2 2 2 1 0x x x k⇔ − + − + = ( ) 2 2 2 1 x g x x x k = ⇔  = + − + i u ki n có BC: Khi ó t a ( ) ( )1 1 2 2; ; ;B x y C x y th a h : ( ) ( ) 2 2 1 0 1 2 4 2 x x k y kx k  + − + =  = − + (1) 2 1 2 ' 2x x k a ∆ ⇔ − = = (2) ( )2 1 2 1 2y y k x x k k⇔ − = − = ( ) ( ) 2 2 2 1 2 1 2 2BC x x y y= − + − = 3 3 4 4 2 2 4 4 8 0 1k k k k k⇔ + = ⇔ + − = ⇔ = x y x y 2 2 x y ( ) ' 0 0 0 2 0 4 4 1 0 9 k k g k k ∆ > > >  ⇔ ⇔   ≠ + − + ≠ ≠   WWW.MATHVN.COM
  • 38. 27 V y ( ): 1 2 4y x∆ = − + Bài 3: Cho (C) ( ) 3 2 3 2y f x x x= = − + . Tìm trên ư ng th ng (d):y=-2 nh ng i m mà t ó có th v ư c n (C) : a. Ba ti p tuy n phân bi t b. Ba ti p tuy n phân bi t trong ó có 2 ti p tuy n vuông góc v i nhau Gi i: a. Ba ti p tuy n phân bi t Xét ( ; 2) : 2A a d y− ∈ = − . Phương trình ư ng th ng ∆ qua ( ; 2)A a − và có h s góc : ( ) ( )2y k x a= − − ∆ . ∆ ti p xúc v i (C) H phương trình sau có nghi m: ( ) ( ) ( ) 3 2 2 3 2 2 1 3 6 2 x x k x a x x k  − + = − −  − = Thay k t (2) vào 1 ta ư c: ( )( ) ( )3 2 2 3 2 3 6 2 3x x x x x a− + = − − − ( )3 2 2 3 1 6 4 0x a x ax⇔ − + + − = ( ) ( )3 2 2 3 1 2 0x x a x ⇔ − − − + =  ( ) ( ) ( )2 2 2 3 1 2 0 4 x g x x a x = ⇔  = − − + = T A k ư c ba ti p tuy n phân bi t n (C) phương trình (3) có 3 nghi m phân bi t phương trình (4) có 2 nghi m phân bi t khác 2 ( ) ( ) ( ) ( ) 2 2 5 0 3 1 16 0 1 *3 2 0 2.2 3 1 .2 2 0 2 g a a a g a a ∆ > − − > < − ∨ >   ⇔ ⇔ ⇔   ≠ − − + ≠   ≠ b. Ba ti p tuy n phân bi t trong ó có 2 ti p tuy n vuông góc v i nhau Khi ó phương trình (3) có 3 nghi m phân bi t: 0 1 22; ;x x x= ( v i x1;x2 là hai nghi m c a phương trình g(x)=0) và 3 ti p tuy n ng v i h s góc là: ( ) ( ) ( )2 2 0 1 1 1 1 2 2 2 2' 2 0; ' 3 6 ; ' 3 6k f k f x x x k f x x x= = = = − = = − Vì 0 0k = nên : Ycbt k1.k2=-1. ( )( ) ( ) ( )2 2 2 2 1 1 2 2 1 2 1 2 1 2 1 23 6 3 6 1 9 2 4 1 **x x x x x x x x x x x x ⇔ − − = − ⇔ − + + = −  Áp d ng nh lí Viet cho phương trình (4) ta có: 1 2 3 1a x x x − + = và 1 2 1x x = Do ó (**) 3 1 9 1 2 4 1 2 a −   ⇔ − + = −      55 27 a⇔ = (th a i u ki n (*)). V y i m c n tìm là 55 ; 2 27 A   −    . WWW.MATHVN.COM
  • 39. 28 D NG TOÁN: H Ư NG CONG TI P XÚC V I M T Ư NG C NH Phương pháp: D ng 1: Cho h ư ng cong ( )mC :y=f(x;m). ch ng minh ( )mC luôn ti p xúc v i m t ư ng (C) c nh . ◊ TH1: ( )mC :y=f(x;m). là hàm a th c. ưa : ( );y f x m= v d ng: ( ) ( ) ( ): ê 2 n y ax bm g x n nguy n= ± + + ≥ . Xét ư ng cong ( ) ( ):C y g x= và ch ng minh h : ( ) ( ) ( ) ( ) ( ) ( ) 1 ' ' n n ax bm g x g x na ax bm g x g x − ± + + =  ± + + = Có nghi m m∀ ◊ TH2: ( )mC :y=f(x;m). là hàm h u t : (D ng t ng quát) ( ∆ ) ti p xúc v i (C) h sau có nghi m ( ) ( ) ( ) ( ) ( ) 0 0 2 1 2 c ax b k x x y x d c a k x a x d  + + = − + +   − = ≠ + Gi i hê trên qua 3 bư c: B1: nhân 2 v c a phương trình (2) cho: x+d ( ) ( )3 c ax ad k x d x d + − = + + B2: (1)-(3): ( )0 0 2c b ad k x d y x d − + = − − + + ( ) ( )0 0 2 4 c k x d y ad b x d ⇔ = − − + + + + B3: Thay (4) vào (2) s có 1 phương trình theo k. gi i phương trình này và tìm m sao cho phương trình úng m∀ . Lưu ý: cách gi i trên có th áp d ng i v i hàm s ax b cx d + + D ng 2: Tìm i u ki n h ư ng cong ti p xúc v i 1 ư ng c nh: Dùng i u ki n ti p xúc. II/ M t s ví d : Bài 1: Cho ( ) ( )3 2 2 : 2 2 1 2mC y x x m x m= + + + + + . Ch ng minh r ng (Cm) luôn ti p xúc v i m t ư ng cong c nh. Gi i: Ta có: ( ) ( )3 2 2 : 2 2 1 2mC y x x m x m= + + + + + ( ) 2 3 2 2x m x x x⇔ + + + + + Xét ư ng cong ( ) 3 2 : 2C y x x x= + + + ( )mC luôn ti p xúc v i (C): h sau có nghi m: ( ) ( ) ( ) 2 3 2 3 2 2 2 2 2 1 2 3 2 1 3 2 1 x m x x x x x x x m x x x x  + + + + + = + + +  + + + + = + + WWW.MATHVN.COM
  • 40. 29 Ta có: ( ) ( ) ( ) 2 0 1 2 0 x m x m  + = ⇔  + = Rõ ràng v i m i m , h (1) luôn có nghi m x=-m Vây m∀ , (Cm) luôn ti p xúc v i 1 ư ng cong c nh: ( ) 3 2 : 2C y x x x= + + + . Bài 2: Cho ( ) ( ) ( )2 2 2 4 :m m x m m C y x m − − − + = − . Ch ng minh (Cm) luôn ti p xúc v i hai ư ng th ng c nh. Gi i: ( ) ( ) ( )2 2 2 4 :m m x m m C y x m − − − + = − ( ) 4 2y m x m ⇔ = − − − (Cm) luôn ti p xúc v i ư ng th ng ( ): y ax b∆ = + ⇔ H phương trình sau có nghi m m∀ : ( ) ( ) ( ) ( ) ( ) 2 4 2 1 4 2 m ax b x m I a x m  − − = + −   = − ◊ Nhân 2 v c a phương trình (2) cho: x-m ( ) ( ) 4 3a x m x m ⇒ = − − ◊ L y (1)-(3): ( ) ( ) ( ) 8 8 2 1 2 4m b am a m b x m x m − ⇔ − − = + ⇔ = − + + − − ◊ Thay (4) vào (2): ( ) ( ) 2 1 2 16a m b a⇔ − + + =   ( ) ( )( ) ( ) ( ) 2 22 1 2 1 2 2 16 0 *a m a b m b a⇔ − + − + + − − = H (1) có nghi m m∀ ( )*⇔ úng m∀ : ( ) ( )( ) ( ) 2 2 1 0 1 2 1 2 0 2 6 2 16 0 a a a b b b b a  − =  = ⇔ − + = ⇔  = ∨ = − + − = V y (Cm) luôn ti p xúc v i 2 ư ng th ng c nh y=x+2 và y=x-6 WWW.MATHVN.COM
  • 41. 30 Bài t p t luy n 1. Cho hàm s ( ) ( )3 2 21 1 1 2 3 3 y x m x m m x= − + + + − . nh m hàm s : a) Tăng trên R b) Gi m trên (0;1) c) Tăng trên (-∞;2) d) Gi m trên o n có dài b ng 3 e) Tăng trên 2 kho ng (-∞;0) và (2; +∞) 2. Cho hàm s ( ) ( )3 2 2 3 : 3 3 1 1mC y x mx m m x m= + + − + + + + . Tìm m : a) (Cm) có i m c c i n m trên x=5 b) Hàm s t c c i và c c ti u t i nh ng i m có hoành >1 c) Hàm s t c c i và c c ti u t i x1 và x2 sao cho: 1 2 2 1 14 5 x x x x − + = 3. Cho hàm s ( ) 3 : 3 2mC y x x= − + . a) Vi t phương trình ti p tuy n có h s góc nh nh t b) Vi t phương trình ti p tuy n i qua M(1;0) c) Tìm trên Ox nh ng i m mà t ó k ư c trên C úng: ◊ m t ti p tuy n ◊ hai ti p tuy n ◊ Ba ti p tuy n ◊ hai ti p tuy n vuông góc v i nhau d) Tìm trên ư ng th ng x=1 nh ng i m mà t ó k ư c trên C úng: ◊ m t ti p tuy n ◊ hai ti p tuy n ◊ Ba ti p tuy n e) Tìm trên (C) nh ng i m mà t ó k ư c trên C úng 1 ti p tuy n. 4. Cho hàm s ( ) 4 2 : 2 2 1mC y x mx m= − + − . Tìm m (Cm) c t Ox t i b n di m phân bi t có hoàn l p thành c p s c ng. 5. Xác nh m phương trình có nghi m duy nh t: 3 2 1 0x mx+ − = 6. Cho hàm s ( ) ( )3 2 2 3 : 3 3 1mC y x mx m x m= − + − − . Tìm m (Cm) c t Ox t i 3 i m phân bi t trong ó có úng 2 i m có hoành âm. 7. Cho hàm s ( ) ( )3 : 1 1mC y x k x= + + + . Tìm k (Ck) ti p xúc v i ư ng th ng ( ): 1y x∆ = + 8. Cho hàm s ( ) 3 2 3 : 3 4mC y x mx m= − + . Tìm m (Cm) c t ư ng th ng ( ):d y x= t i A,B,C sao cho AB=BC. 9. Cho hàm s ( ) 2 1 : 2 m x C y x + = + . Ch ng t r ng ư ng th ng y=-x+m luôn luôn c t th t i hai i m phân bi t AB. Tìm m o n AB ng n nh t. 10. Cho hàm s ( ) ( ) ( ) 2 3 1 : 1m m x m m C y x m + − + = + . Trong ó m là tham s khác 0: a) Tìm nh ng i m mà th không i qua m∀ . b) Ch ng minh r ng th c a (1) luôn ti p xúc v i 2 ư ng th ng c nh. 11. Cho hàm s ( ) ( ) ( ) ( ) ( )3 2 : 3 3 1 6 1 1 1mC y m x m x m x m= + − + − + + + . Ch ng minh r ng h th (Cm) luôn luôn i qua 3 i m c nh th ng hàng. WWW.MATHVN.COM
  • 42. 31 Bài VI: M t s d ng toán khác c n lưu ý. I/ Gi i h n: D ng toán này ã t ng xu t hi n trong thi i h c t r t lâu (năm 2002 – 2003) Tuy nhiên ã r t lâu không th y xu t hi n trong thi i h c. Tuy nhiên ta cũng nên chú ý n d ng toán này. âu tôi xin trình bày phương pháp t ng quát làm bài d ng này là “ G i s h ng v ng b ng h s b t nh”. Bài 1. Tìm 33 2 21 5 7 lim 1x x x x→ − − + − Gi i: Ta có: ( ) 3 33 2 3 2 2 2 21 1 5 7 5 2 7 2 lim lim 1 1 1 1x x x x x x x x x→ →  − − + − − + − = −   − − −  ( )( ) 3 3 21 1 2 3 5 2 1 lim lim 1 1 5 2x x x x x x x→ → − − − = − − − + = ( ) ( )( ) ( ) 2 1 3 1 3 lim 2 81 5 2x x x x x→ − + + − = + − + ( ) ( ) 3 2 2 21 1 2 32 2 23 7 2 1 lim lim 1 1 7 2 7 4 x x x x x x x x → → + − − = −   − + + + +    = ( ) ( )21 32 23 1 1 lim 3 127 2 7 4 x x x → = + + + + Thay (2),(3) vào (1) có: 3 1 11 8 12 24 A − = − = Lưu ý: Trong l i gi i ta ã thêm s 2 vào t th c f(x). Có l b n ang t h i: ● T i sao ph i thêm s 2 ? ● Làm cách nào nh n ra s 2 ? S 2 là h ng t ã b xóa! Mu n làm d ng bài này, ta ph i khôi ph c nó. Mu n khôi ph c s 2 này ta làm như sau: B1: c R∀ ∈ luôn có: ( ) 33 2 2 2 5 7 1 1 x c x c f x x x  − − + − = −   − −  B2: Trong các s c ó. Ta tìm s c sao cho x2 -1 có cùng nhân t chung v i ( ) 3 1 5f x x c= − − và ( ) 3 2 2 7f x x c= + − . i u ó x y ra khi và ch khi c là nghi m c a tuy n: ( ) ( ) ( ) ( ) 1 2 1 2 1 0 2 1 0 26 1 0 2 1 0 f c f cc f c f  =  = =  ⇔ ⇔ ==  − =   =  − = ó chính là lí do t i sao 2 xu t hi n trong bài gi i. ây là vi c nên làm trong gi y nháp. Không nh t thi t trình bày trong bài làm. Qua ví d trên ta nêu lên thu t toán sau: Gi s ( ) ( ) ( ) f x F x g x = có gi i h n 0 0 WWW.MATHVN.COM
  • 43. 32 B1: Phân tích ( ) ( ) ( ) ( ) ( ) 1 2f x c f x c f x g x g x + − = + . B2: (Tìm c): G i ( )1;2;...i iα = là nghi m c a h g(x)=0 Khi ó c là nghi m c a h : ( ) ( ) ( )1 1 0 1;2;... 0 i i f c i f c α α + = = − = V i c tìm ư c thì ( ) ( ) 1 lim ix f x c g xα→ + và ( ) ( ) 2 lim ix f x c g xα→ − s ho c là d ng xác nh ho c là d ng quen thu c. Sau khi tìm c, vi c trình bày l i gi i như ã làm. BÀI T P ÁP D NG: A= 3 2 2 0 3 1 2 1 lim 1 cosx x x x→ − + + − ( d b 2002) B= 3 20 1 2 1 3 lim x x x x→ + − + II/Phương trình và b t phương trình mũ và logarit: ây là d ng toán cũng r t thư ng xuyên xu t hi n trong thi. Nhìn chung, d ng toán này không khó. T t c các phép bi n i ch xoay quanh các công th c ã nêu trong sách giáo khoa. ph n này, tôi không nêu l i các công th c trên. Xin trình bày cách gi i c a 1 s thi g n ây. Bài làm qua 2 bư c: B1: t i u ki n. (N u i u ki n quá ph c t p thì có th n bư c 2 r i th nghi m vào i u ki n) B2: Bi n i phương trình hay b t phương trình v d ng ơn gi n cùng cơ s c 2 v : • Mũ: Chia • Logarit: log log log b a b x x a = log logn n aa m x x n = • t n ph : ( )logat f x= phương trình h u t ho c phương trình mũ ( )f x t a= phương trình h u t . • Phương pháp hàm s Bài 1. ( ) 22 2 2 3 1 2 3 1 2 3 1 81.4 78.6 16.9 0 1 x x x x x x − + − + − + − + ≤ Gi i: ( ) 2 2 2 3 1 2 3 1 6 9 1 81 78 16 0 4 4 x x x x− + − +     ⇔ − + ≤        ( )2 2 2 3 1 2. 2 3 1 3 3 81 78 16 0 2 2 x x x x− + − +     ⇔ − + ≤        t 2 2 3 1 3 2 x x t − +   =     k: t>0 Phương trình tr thành: 2 16 78 81 0t t− + ≤ 3 27 ; 2 8 t   ⇔ ∈   2 2 3 1 23 3 27 1 2 3 1 3 2 2 8 x x x x − +   ⇔ ≤ ≤ ⇔ ≤ − + ≤    WWW.MATHVN.COM
  • 44. 33 2 2 2 2 3 2 02 3 1 1 2 3 0 12 3 1 3 2 3 2 0 2 2 x xx x x x x x x x x x  ≥  ≤ − + ≥ − ≥    ⇔ ⇔   − + ≤ − − ≤     ≤  ≥ 2 1 2 x x ≥ ⇔  ≤  Bài 2. Gi i b t phương trình: 1 1 1 1x x x e e x+ − + − − ≤ − Gi i: t: 1 1 1 1 u x x u v x v x  = + − ⇔ − = − = + − Phương trình tr thành: u v e e u v− = − ( ) ( )f u f v⇔ ≤ V i ( ) ; 1x f x e x x= − ≥ ( )' 1 0x f x e⇒ = + > ⇒ ( )f x tăng. Do ó u v≤ 1 1 1 1x x x x⇔ + − ≤ + − ⇔ ≤ − Bài 3. Gi i phương trình: ( )2 3log 1 logx x+ = Gi i: t 3log 3t x t x= ⇔ = Do ó: ( ) ( )2log 1 1 2 1 3 2 t t t x t x+ = ⇔ + = ⇔ + = 22 1 3 1 3 1 3 1 2 2 2 2 2 2 t tt t            + = ⇔ + = +                          ( ) ( )2f t f⇔ = 2t⇔ = (Vì ( ) 1 3 2 2 tt f x    = +         là hàm gi m) 2 9t x⇔ = ⇔ = Bài 4. Gi i b t phương trình: ( ) ( )1 2log log 4 8 1 1x x +  − ≥  Gi i: K: ( ) ( )2 11 3 5 4 8 0 2 2 2 1 3 2 xx x x −− − > ⇔ > ⇔ − > ⇔ > ( ) ( )1 21 log log 4 8 logx x x x+  ⇔ − ≥  ( ) ( )1 1 2 2 2log 4 8 log 4 8 log 2x x x x− − ⇔ − ≥ ⇔ − ≥ ( )1 2 04 4 8 2 2 8 0 3 4 2 8 xx x x x x loai x−  ≤ ⇔ − ≥ ⇔ − − ≥ ⇔ ⇔ ≥ ≥ Bài 5. Gi i h phương trình: ( ) ( ) ( )2 3 9 3 1 2 1 1 3log 9 log 3 2 x y x y  − + − =  − = ( H A 2005) WWW.MATHVN.COM
  • 45. 34 Gi i: k: 1 0 2 x y ≥  < ≤ ( ) ( )3 3 3 32 3 1 log 3log 3 log logx y x y x y⇔ + − = ⇔ = ⇔ = Thay x=y vào (1) ta có: ( )( )1 2 1 1 2 2 1 2 1x x x x x x− + − = ⇔ − + − + − − = ( )( )1 2 0 1, 2x x x x⇔ − − = ⇔ = = V y h có hai nghi m là (x;y)=(1;1) và (x;y)=(2;2) Bài 6. Gi i phương trình: ( ) ( )4 2 2 1 1 1 log 1 log 2 1 log 4 2x x x + − + = + + (D b 1A – 2007) Gi i: K: x>1 ( ) ( ) ( ) ( )4 4 4 1 1 log 1 log 2 1 log 2 2 x x x⇔ − + + − + = ( )( ) 4 1 2 1 1 log à 1 2 2 x x v x x − +  ⇔ = >  +  2 2 1 2 à 1 2 x x v x x − − ⇔ = > + 2 5 2 3 5 0 à 1 2 x x v x x⇔ − − = > ⇔ = BÀI T P ÁP D NG: 1) ( )2 2 2 4 log 6 7 2 1 1 log 4 x x x x + − ≥   + − +    2) 2 3 2 3log log log logx x x x+ ≥ 3) 2 2log 3 log 52 x x x+ = 4) ( ) ( )2 2 3 5log 15 log 45 2x x x x+ − − − = 5) ( ) ( )0.2 3 5log 2 log log 2x x x− + ≥ + 6) ( ) ( ) ( ) ( )2 2 33 log 2 4 2 log 2 16x x x x+ + + + + = 7) ( ) ( )2 2 3 1 log 3 1 2 log 1 log 2x x x + − + = + + 8) CMR: v i m i a>0, h phương trình sau có nghi m duy nh t: ( ) ( )ln 1 ln 1x y e e x y x y a  − = + − +  − = 9) Gi i h phương trình: ( ) ( ) ( )2 2 2 2 2 2log 1 log , 3 81x xy y x y xy x y R − +  + = + ∈ = ( H A 2009) WWW.MATHVN.COM
  • 46. 2 10) Tìm m phương trình sau có úng 1 nghi m: ( ) ( )5 1 2 5 1 2 x x m x+ + − = 11) 3 2 2 3 7 9.5 5 9.7x x x x + = + 12) ( ) ( ) 7 5 5 7 x x = 13) ( ) ( ) 10 5 10 3 3 84 0 x x− + − = 14) ( )3 3 16 6 4 8 2 0x x x x− − + − + − = 15) Tìm m phương trình sau có úng 1 nghi m: 2 2 sin cos 9 9x x m+ = 16) ( )2 3 log 3 1x x x− − > 17) ( )3 3 16 6 4 8 2 0x x x x− − + − + − = 18) Cho b t phương trình: ( ) ( ) ( )2 2 2log 1 log 1x ax a+ < + a) Gi i b t phương trình khi a=2 b) Tìm t t c giá tr c a a b t phương trình có nghi m 19) 2 2 3 9 6 x x x x x − − = − + 20) ( )2 2 3.25 3 10 .5 3 0x x x x− − + − + − = 21) Tìm m phương trình có 2 nghi m trái d u: ( ) ( )3 16 2 1 4 1 0x x m m m+ + − + + = 22) Tìm m phương trình có nghi m: 9 .3 2 1 0x x m m− + + = 23) ( )2 5 4 5 3 5 3x x x + − − ≤ + 24) Tìm m h có nghi m: ( ) ( )2 2 2 log log 1mx y x y x y m  + + − =  − = 25) Gi i b t phương trình: 2 0.5 15 log log 2 2 16 x   − ≤      26) Gi i b t phương trình 3 4 1 1 3 4 3 1 1 log log log log 1 3 1 x x x x   −  +  ≤    + −     WWW.MATHVN.COM
  • 47. 2 PH L C: M T S THI C N THAM KH O (Theo c u trúc thi c a B GD& T 2010) 1: A. PH N CHUNG: Câu 1: Cho hàm s (C) ( )( )2 21 1 4 y x m x= − + , m là tham s . 1. Kh o sát và v th (C) khi m =3 2. nh m bi t th hàm s (C) c t Ox t i A và B sao cho 2 ti p tuy n t i A và B vuông góc. Câu 2: 1. Gi i phương trình: 3 27 cos 2 sin 2sin 2 x x x+ = 2. Gi i phương trình: ( ) ( )4 4 4 2x x x x x+ − − = − Câu 3: Tính gi i h n: ( )2 2 sin 20 log cos lim 2 1x xx x x x→ + − + Câu 4: Cho hình nón nh S có thi t di n qua tr c SO=a là m t tam giác vuông. M t ph ng qua S và c t ư ng tròn áy t i A và B sao cho ∆ SAB u. Tình th tích hình c u ngo i ti p hình chóp SOAB. Câu 5: Cho x,y,z [ ]0;1∈ . Tìm giá tr l n nh t: ( ) ( ) ( ) 2 2 2 A x y y z z x= − + − + − B. PH N T CH N: (Thí sinh ch ư c ch n Câu 6 ho c Câu 7 Câu 6: (Chương trình chu n) a. Trong Oxy cho ∆ ABC có A(0;2), B(2;6), và : 3 1 0C d x y∈ − + = sao cho phân giác k t A song song v i d. Tìm t a C. b. Trong Oxyz vi t phương trình ư ng th ng ∆ qua A(0;1;2) c t 1 1 1 : 1 1 1 x y z d − − = = − và h p v i 2 1 2 4 2 1 1 x y z d + − − = = = − m t góc 600 c. Cho ( ) ( ) ( ) 1 1 1 01 1 ... 1 , n n n n na x a x a x a x x R − −− + − + + − + = ∀ ∈ . Tìm n bi t 2 3 1 231a a a+ + = Câu 7: (Chương trình nâng cao) a. Trong Oxy tìm ( ) 2 2 : 1 6 3 x y M E∈ + = bi t kho ng cách t M n d: x+y=0 là l n nh t b. Trong Oxyz vi t phương trình m t ph ng qua M(1;2;2) và c t Ox, Oy, Oz t i A,B,C sao cho: 2 2 2 2 1 1 1 1 OA OB OC OM + + = c. B ng cách khai tri n: ( ) 2 1 n i+ hãy ch ng minh: ( )0 2 4 2 2 2 2 2... 1 2 cos 2 n n n n n n n n C C C C π − + − + − = , ( ), 0n N n∈ > . WWW.MATHVN.COM
  • 48. 3 2: A. PH N CHUNG: Câu 1: Cho hàm s (C) 4 22 9 y x x= − + 1. Kh o sát và v th (C) 2. Tìm trên th (C) các i m A bi t ti p tuy n t i A c t (C) t i B và C sao cho AB=AC ( B,C khác A) Câu 2: 1. Gi i phương trình: ( ) ( )1 3 cos sin 3 cos cos 1x x x x− + − = 2. Gi i h phương trình: 2 23 2 2 2 3 4 5 x y x y x x y  + − − =  + + − = Câu 3: Tính tích phân: 2 1 1 ln e dx x x x+ − ∫ Câu 4: Cho lăng tr ng ABC.A’B’C’ có AB’=a; BC’=b và ∆ ABC vuông cân t i A. Tính th tích lăng tr . ( )2a b a< < Câu 5: Cho [ ], 1;2 .x y∈ Tính giá tr l n nh t và nh nh t: ( ) ( )2 2 2 2 1 1 1 1 4A x y x y x y x y     = + + + − −        B. PH N T CH N: (Thí sinh ch ư c ch n Câu 6 ho c Câu 7 Câu 6: (Chương trình chu n) a. Trong Oxy tìm ( ) 2 2 : 1 6 3 x y M E∈ + = bi t góc F1MF2 b ng 600 . b. Trong Oxyz vi t phương trình tham s ư ng th ng ∆ song song v i (P): 2x+2y-z-3=0 và c t hai ư ng th ng 1 2 1 : 2 1 1 x y z d − − = = − và 2 1 1 : 1 2 1 x y z d − + = = − t i A và B sao cho AB=3 c. Gieo ng th i 3 con xúc x c, tính xác su t tích 3 s n t xu t hi n là 1 s ch n. Câu 7: (Chương trình nâng cao) a. Trong Oxy vi t phương trình chính t c hypebol qua M(2;1) th a góc F1MF2 b ng 600 b. Trong Oxyz vi t phương trình m t ph ng h p v i (Oxy) m t góc 450 , song song v i Ox và cách Ox m t kho ng b ng 2 c. Cho z= 3 i+ . Tìm s t nhiên n>0 sao cho n z là s nguyên dương bé nh t. WWW.MATHVN.COM
  • 49. 4 3: A. PH N CHUNG: Câu 1: Cho hàm s (C) 2mx y x m + = + 1. Kh o sát và v th (C) khi m =-1 2. Tìm trên th (C) c t Ox t i A, C t Oy t i B sao cho 2 ti p tuy n t i A và B song song Câu 2: 3. Gi i phương trình: 1 cos2 cos 3sin 2 x x x+ + = 4. Gi i phương trình: ( ) ( )2 2 2 3log 12 .log 12 2x x x x+ − − − = Câu 3: Tính tích phân: ( ) 2 4 0 sin3 1 cos xdx x π + ∫ Câu 4: Tính th tích hình chóp S.ABCD có áy là hình ch nh t, chi u cao SA=a h p v i (SBC) và (SBD) các góc 450 và 300 Câu 5: nh m h sau có nghi m: 2 2 2 1 2 4 y x xy x x y m  − + =   + − = B. PH N T CH N: (Thí sinh ch ư c ch n Câu 6 ho c Câu 7) Câu 6: (Chương trình chu n) a. Vi t phương trình ư ng tròn i qua g c t a và c t Ox, Oy t i A,B sao cho AB= 4 2 . Bi t r ng tâm ư ng tròn thu c d:x+y-4=0 b. Trong Oxyz vi t phương trình m t ph ng (P) qua M(1;1;0), song song v i 3 : 4 5 3 x y z d − = = − và cách g c t a m t kho ng b ng 1. c. Tìm ,a b R∈ bi t phương trình 3 1 5 a b z z + = + − có 1 nghi m 1 5 1 2 i z i = + . Tìm nghi m còn l i. Câu 7: (Chương trình nâng cao) a. Tìm t a 3 nh ∆ ABC vuông cân t i A có tr c i x ng là x-2y+1=0; ;A Ox B Oy∉ ∈ và : 1 0C d x y∈ + − = . b. Vi t phương trình tham s c a ư ng th ng d qua M(1;2;0), song song v i (P):2x-y+z-1=0 và h p v i (Q): x+y+2z-1=0 m t góc 600 c. Trong h p ng 15 viên bi g m 4 bi , 5 bi xanh và 6 bi vàng. Tính xác su t ch n ư c 4 viên bi c 3 màu. WWW.MATHVN.COM
  • 50. 5 4: A. PH N CHUNG: Câu 1: Cho hàm s 3 2 3 x y x= − + có th (C) 1. Kh o sát và v th (C) 2. Vi t Phương trình ư ng th ng d qua g c t a O và c t (C) t i A và B (khác O) saocho 2 ti p tuy n c a (C) t i A và B vuông góc. Câu 2: 5. Gi i phương trình: tan tan sin 2 1 2sin2 4 2 2x x x x+ + + = 6. Gi i b t phương trình: 2 2 3 2 2 5 x x x x x + − ≥ − − Câu 3: Tính tích phân: 44 4 4 0 sin sin cos x dx x x π +∫ Câu 4: Tính th tích hình chóp S.ABCD có áy là hình vuông chi u cao SA. Bi t SC=2a h p v i (SAB) m t góc 300 . Câu 5: Cho a,b,c>0 và a+b+c=1. Tìm giá tr nh nh t: 2 2 2 3 3 3 3 a b c A a b c + + = + + − B. PH N T CH N: (Thí sinh ch ư c ch n Câu 6 ho c Câu 7) Câu 6: (Chương trình chu n) I/ Trong Oxyz cho A(2;3;-1), B(5;-3;2) và (P): x+y+z-3=0: a. Vi t phương trình tham s ư ng th ng d vuông góc v i (P) và c t ư ng th ng AB t i I sao cho 2 0AI BI+ = b. Tìm ( )M P∈ sao cho AM2 +2BM2 nh nh t II/ Hãy phân ph i 2010 i m lên 2 ư ng th ng song song sao cho t ng s tam giác thu ư c là l n nh t. Câu 7: (Chương trình nâng cao) I/ a Vi t phương trình ư ng tròn trong Oxy i qua A(2;1), Tâm thu c Oy và c t Ox t i B và C sao cho góc BAC b ng 600 b. Trong Oxyz cho A(0;1;2), B(1;-1;1), C(-1;3;0). Vi t phương trình tham s ư ng th ng d vuông góc v i (ABC) và c t (ABC) t i tr c tâm H c a ∆ ABC. II/ nh m bi t th hàm s ( )2 1 2 1x m x m y x m − + + − = − ti p xúc v i Ox. WWW.MATHVN.COM
  • 51. 6 5: A. PH N CHUNG: Câu 1: Cho hàm s 3 1 x y x − = + có th (C) 1. Kh o sát và v th (C) 2. Cho A(0;2). Tìm trên (C) i m M sao cho AM ng n nh t. Câu 2: 1. Gi i phương trình: 2 2 3 cos cos cos3 cos 3 4 x x x x− + = 2. Gi i h phương trình: 2 2 2 2 1 1 3 1 1 1 x y x y x y xy  + + + =    + =  + Câu 3: Tính tích phân: 4 3 2 3 4 ln 1 x x dx x+ ∫ Câu 4: Cho hình chóp S.ABCD có (SAB) ⊥ (ABC), ∆ ABC u và ∆ ABC vuông cân t i A. Tính th tích m t c u ngo i ti p hình chóp Bi t SC= 2a Câu 5: Cho a,b,>0 và 1 1 1 a b + = . Tìm giá tr nh nh t: ( )2 2 25 1 1 4 a b ab A a b a b = + + − − + B. PH N T CH N: (Thí sinh ch ư c ch n Câu 6 ho c Câu 7) Câu 6: (Chương trình chu n) I/ Trong Oxyz cho A(2;-1;2), B(3;-3;3); C(1;-2;4) và (P): 2x-3y+z+1=0: a. Vi t phương trình tham s ư ng th ng d i qua tâm ư ng tròn ngo i ti p ∆ ABC và vuông góc v i (P) b. Tìm ( )M P∈ sao cho AM2 +2BM2 +CM2 nh nh t II/ Tìm ,a b R∈ bi t 2 3 4 2009 ...Z i i i i i= − + − + + là nghi m c a phương trình 1 1 1 a b z z + = + − . Tìm nghi m còn l i. Câu 7: (Chương trình nâng cao) I/ Trong Oxyz cho 1 : 1 2 2 x t d y t t =  = +  + ; 2 1 : 1 1 1 x y z d − = = − a Tìm 1A d∈ bi t kho ng cách t A n d2 b ng 6 b. Vi t phương trình m t ph ng (P) ch a d2 và h p v i d1 m t góc 300 II/ Gi i h phương trình: 3 3 3 log log 2 2 6 log log 1 x x y x y y x  + =   + =  WWW.MATHVN.COM
  • 52. 7 6: A. PH N CHUNG: Câu 1: Cho hàm s (C) 4 2 1 4 x y mx m= − + + , m là tham s . 1. Kh o sát và v th (C) khi m =1 2. nh m bi t th hàm s (C) có 3 i m c c tr t o thành tam giác có tr c tâm là g c t a Câu 2: 1. Gi i phương trình: sin 2 cos 2 tan 6 3 4 x x x π π π      + + + = +            2. Gi i h phương trình: ( ) ( )3 3 3 3 2 3 1 1 1 1 8 log log 1 2 3 x y x y x y x y x y      + + + + + =            = Câu 3: Tính tích phân: 2 3 1 0 x xdx I e + = ∫ Câu 4: Tính th tích hình lăng tr u ABCD.A’B’C’D’ bi t AC’=a và góc gi a BD và CD’ b ng 600 . Câu 5: Cho a,b,c>0 và 1 1 1 1 a b c + + = . Tìm giá tr l n nh t: 3 3 3 3 3 3 b c c a a b A b c c a a b + + + = + + + + + B. PH N T CH N: (Thí sinh ch ư c ch n Câu 6 ho c Câu 7 Câu 6: (Chương trình chu n) a. Trong Oxy cho ∆ ABC vuông cân t i A có di n tích b ng 2, bi t 1 2 1 0A d x y∈ = − + = và 2, : 2 0B C d x y∈ + − = . Tìm t a A,B,C v i xA, xB>0. b. Trong Oxyz vi t phương m t ph ng (P) qua A(0;1;2), B(1;3;3) và h p v i ( ): 2 0Q x y z− − = m t góc nh nh t. c. Tìm s t nhiên n th a: 3 2 3 1 1 1 7 n n nC C A+ +− = Câu 7: (Chương trình nâng cao) a. Trong Oxy cho hai ư ng tròn ( ) 2 2 : 2 2 0mC x y mx my m+ − − + − = và ( ) 2 2 : 3 1 0C x y x+ − + = . nh m bi t s ti p tuy n chung c a hai ư ng tròn là m t s l . b. Trong Oxyz vi t phương trình ư ng th ng d song song v i ( ): 2 1 0P x y z+ + − = và c t 2 ư ng th ng Ox và 2 1 : 2 1 1 x y z− + ∆ = = − t i 2 i m A,B sao cho AB ng n nh t. c. Gi i phương trình: 4 2 1 0z z+ + = , z C∈ . WWW.MATHVN.COM
  • 53. 8 7: A. PH N CHUNG: Câu 1: Cho hàm s (C) 3 2 3y x ax b= − + , (1) ( ), 0a b > 1. Kh o sát và v th (C) khi a=1 b=4 2. nh a,b bi t th hàm s (C) có 2 i m c c tr A và B sao cho ∆ OAB vuông cân. Câu 2: 3. Gi i phương trình: 2 tan2 1 tan .tan 2 sin3 x x x x   + =    4. Gi i h phương trình: 2 2 2 2 1 1 1 2 5 2 1 2 x y xy x y x y  + = +   − =  + Câu 3: Tính gi i h n: ( )0 1 lim ln 1 sin x x e x x→ − + + Câu 4: Cho hình chóp S.ABCD chi u cao SA=2a, áy là hình thang vuông t i A và B có AB=BC=a, AD=2a. M t ph ng qua trung i m M c a SA ch a CD, c t SB t i N. Tính di n tích t giác CDMN. Câu 5: nh m b t phương trình có nghi m: ( )2 1 ln 2 1 2 x x m x m mx x + + − + − ≤ − . Tìm nghi m tương ng B. PH N T CH N: (Thí sinh ch ư c ch n Câu 6 ho c Câu 7 Câu 6: (Chương trình chu n) a. Trong Oxy cho ( ) ( ) ( )7;1 , 3; 4 , 1;4A B C− − . Vi t phương trình ư ng tròn n i ti p ∆ ABC. b. Trong Oxyz vi t phương trình m t ph ng (P) qua g c t a , song song v i 1 1 2 : 1 2 1 x y z d − + − = = − và h p v i 1 2 : 2 1 1 x y z+ − ∆ = = m t góc 600 c. Tìm h s c a 3 x trong khai tri n thành a th c c a bi u th c: ( ) 62 1x x+ − . Câu 7: (Chương trình nâng cao) a. Trong Oxy cho ư ng tròn ( ) 2 2 : 6 5 0C x y x+ − + = . Tìm M thu c tr c tung sao cho qua M k ư c hai ti p tuy n c a (C) mà góc gi a hai ti p tuy n b ng 600 b. Trong Oxyz Cho ( )2;1;0M và ư ng th ng d có phương trình 1 1 2 1 1 x y z− + = = − − . Vi t phương trình chính t c c a ư ng th ng i qua i m M, c t và vuông góc v i ư ng th ng d. c. Tìm h s c a 3 x trong khai tri n thành a th c c a bi u th c: ( ) 52 1x x+ − . WWW.MATHVN.COM
  • 54. 9 8: A. PH N CHUNG: Câu 1: Cho hàm s (C) 1 1 mx y x + = + 1. Kh o sát và v th (C) khi m =-1 2. nh m bi t ti p tuy n t i i m c nh c a h th (C) cách I(1;0) m t kho ng l n nh t Câu 2: 1. Gi i phương trình: 2 2 sin sin 2 .sin 4 cos 2x x x x+ = 2. Gi i b t phương trình : ( )2 3 2 3 2 2 7 2 2 15x x x x+ − − + − + ≤ Câu 3: Tính th tích v t th tròn xoay sinh ra b i hình ph ng t o b i ( ) 1 1 : 1 1 ,C y x x = + + − tr c Ox và 2 ư ng th ng x=1; x=2 quay quanh Ox. Câu 4: Cho hình vuông ABCD c nh a và hai ư ng th ng 1 2;d d l n lư t qua A và C và vuông góc v i m t ph ng (ABCD). L y 1 2, NM d d∈ ∈ sao cho ,AM CN cùng chi u và có t ng dài b ng 6a. Tính th tích t di n MNBD Câu 5: Gi i h phương trình: 2 2 1 1 1 ln 1 1 1 ln xy x x y y xy y y x x  + = + +   + = +  + B. PH N T CH N: (Thí sinh ch ư c ch n Câu 6 ho c Câu 7) Câu 6: (Chương trình chu n) a. Trong Oxy cho A,B là hai i m trên ( ) 2 :P y x= sao cho ∆ OAB vuông t i A. Tìm t a A,B ( )0Ay < bi t OB ng n nh t. b. Trong Oxyz vi t phương trình m t ph ng (P) qua g c t a và song song v i 1 1 2 : 2 2 1 x y z d − − − = = và cách d m t kho ng b ng 1. c. Cho a giác l i n nh, bi t r ng s tam giác có nh và c nh chung v i a giác là 70. Tìm s tam giác có nh chung và không có c nh chung v i a giác. Câu 7: (Chương trình nâng cao) a. Trong Oxy vi t phương trình chính t c elip (E) qua M(2;1) sao cho 1 2.MF MF nh nh t. b. Trong Oxyz vi t phương trình m t ph ng (P) qua g c t a và l n lư t h p v i 2 m t ph ng ( ) ( ): 1 0 và : 2 1 0Q x z R x y z+ − = + − + = các góc 300 và 600 c. Tính giá tr : ( )( )2 2008 2 3 2008 1 2 3 ... 2009 1 2 3 4 ... 2009Z i i i i i i i= + + + + − + − + + . WWW.MATHVN.COM
  • 55. 10 9: A. PH N CHUNG: Câu 1: Cho hàm s (C) ( )( )2 1y x m x x= − − + 1. Kh o sát và v th (C) khi m =3 2. nh m bi t (Cm) c t Ox t i A, c t Oy t i B sao cho hai ti p tuy n c a (Cm) t i A và B vuông góc. Câu 2: 1. Gi i phương trình: 1 sin cos tan 1 sin cos x x x x x − + = + + 2. Gi i b t phương trình : ( ) ( )2 2 2 2 log log log 0.25 7 5 2 3 2 2 xx x + + = − Câu 3: Tính di n tích hình ph ng gi i h n b i: ( ) 2 : 2 3C y x x= − − và : 1d y x= + Câu 4: Cho hình chóp S.ABCD chi u cao SA=a, áy là hình vuông c nh a. ch ng minh AI ⊥ (SBD) av2 tính th tích t di n SIBD, bi t I là trung i m SC. Câu 5: Tìm giá tr nh nh t tham s m h : 2 2 1 1 3 2 x y x y m  + =   + = có nghi m x,y>0. Tìm nghi m tương ng. B. PH N T CH N: (Thí sinh ch ư c ch n Câu 6 ho c Câu 7) Câu 6: (Chương trình chu n) a. Trong Oxy cho ∆ ABC có ư ng cao và trung tuy n k t A là 2 4 0Ah x y= + + = , 2 0Am y= − = và ư ng trung tuy n k t B là :3 11 21 0Bm x y+ + = . Tính góc C b. Trong Oxyz cho 1 2 2 1 2 : ,d : 2 1 2 1 1 x t x y z d y t z t = − − −  = = =  = + Ch ng minh r ng có vô s m t ph ng (P) ch a d2 và song song v i d1. Vi t phương trình (P) sao cho d2 là hình chi u vuông góc c a d1 lên (P) c. Tìm ,x y R∈ th a: ( ) ( ) 2 1 1 1 2 2 1x y i y xi i − = + − + + + Câu 7: (Chương trình nâng cao) a. Trong Oxy cho ( ) ( ) 2 2 2 2 : 1 , 0 x y H a b a b − = > có hai tiêu i m là 1F 2; F . ư ng th ng d qua 2; F vuông góc Ox và c t (H) t i M và N sao cho 1F MN∆ u. Tìm tâm sai c a (H) và vi t phương trình (H) n u bi t di n tích 1 4 3F MN∆ = b. Trong Oxyz cho A(-1;2;2), B(0;3;0). Hãy tìm trong (P) sao cho ∆ ABC u. c. M t ư ng th ng ti p xúc v i th hàm s 3 3 4 x y x = + và c t 2 ư ng ti m c n t i A và B. Tính di n tích ∆ OAB. WWW.MATHVN.COM
  • 56. 11 10: A. PH N CHUNG: Câu 1: Cho hàm s ( ) ( ) 4 2 1 , 1 2 x y m x m − = + + − có th (C) . m là tham s . 1. Kh o sát và v th (C) khi m=0 2. Ch ng minh r ng th hàm s (1) luôn i qua 2 i m A và B c nh. nh m bi t 2 ti p tuy n t i A và B h p nhau góc 600 Câu 2: 3. Gi i phương trình: 4sin 2 sin 1 3sin 2 cos2 3 x x x x π  + = + −    4. Gi i h phương trình: 2 2 4 8 3 12 x xy y xy y x  − + =  + + = Câu 3: Tính di n tích hình ph ng gi i h n b i: lnx x x y e + = , tr c Ox và hai ư ng th ng x=1;x=4. Câu 4: Tính th tích hình chóp S.ABC bi t SA, SB, SC ôi m t h p v i nhau góc 600 và có dài l n lư t là a, 2a, 3a. Câu 5: nh m phương trình ( ) ( )( )2 3log 2 4 1 log 1 3x m m x x− + = + − − −   có nghi m duy nh t. Tìm nghi m duy nhât ó. B. PH N T CH N: (Thí sinh ch ư c ch n Câu 6 ho c Câu 7) Câu 6: (Chương trình chu n) I/ Trong Oxyz cho 2 1 : 2 1 1 x y z d + − = = và (P): x-y-1=0: a. Vi t phương trình tham s ư ng th ng d’ là hình chi u vuông góc c a d lên (P). Tính góc gi a d và d’. b. G i A là giao i m c a (P) và d. Vi t phương trình các m t c u ti p xúc (P) t i A và c t d t i B sao cho AB= 6 II/ Gi i phương trình: 3 3 2 3 2 3 1 log log log log 23 x x x x    − = +       Câu 7: (Chương trình nâng cao) I/ Trong Oxyz cho A là giao i m c a 1 : 1 2 2 x t d y t t =  = +  + và m t ph ng (P):x-2y+z=0 a Vi t phương trình chính t c ư ng th ng ∆ qua A vuông góc v i d và h p v i (P) m t góc 300 b. Vi t phương trình m t c u có tâm I thu c d, i qua A và c t P m t ư ng tròn dài 2 2π II/ Tìm φ ( )0;2π∈ bi t th hàm s ( )2 2 cos 3sin 1 x x y x ϕ ϕ+ + + = − có hai i m c c tr là A và B sao cho AB dài nh t, ng n nh t. WWW.MATHVN.COM
  • 57. 12 11: A. PH N CHUNG: Câu 1: Cho hàm s ( ) 2 , 1 1 x y x = − có th (C) . 1. Kh o sát và v th (C) c a hàm s (1) 2. Tìm M trên (C) bi t ti p tuy n t i M t o v i 2 ti m c n c a (C) m t tam giác có chu vi bé nh t. Câu 2: 5. Gi i phương trình: 2 16sin 4cos4 3 cos sinx x x x+ = + 6. Gi i phương trình: ( ) 3 5 5 2x x x− − − = Câu 3: Tính th tích v t th tròn xoay sinh ra b i hình tròn ( ) ( ) ( ) 2 2 : 3 1 1C x y− + − = quay quanh tr c Oy. Câu 4: Cho t di n ABCD có AB=a, AC= 2a , AD=2a. ư ng th ng AC h p v i AB,AD các góc 450 , AB h p v i AD góc 600 . Tính t s th tích c a t di n và hình c u ngo i ti p t di n. Câu 5: Cho 2 2 2 1.a b c+ + = Ch ng minh r ng: 3 3 3 3 1a b c abc+ + − ≤ . B. PH N T CH N: (Thí sinh ch ư c ch n Câu 6 ho c Câu 7) Câu 6: (Chương trình chu n) a. Trong Oxyz vi t phương trình m t ph ng (P) i qua H(1;2;3) và c t Ox, Oy, Oz l n lư t t i A, B, C sao cho H là tr c tâm ∆ABC b. Trong Oxyz vi t phương trình m t c u tâm I ∈ Oz, i qua A(1;1;1) và c t (Oxy) m t ư ng tròn dài 2π c. Gi i phương trình : 0 1 2 2 2 3 4 .... 120 , xx xC C C C N− + + + + = ∈ Câu 7: (Chương trình nâng cao) I/ Trong Oxyz cho A(3;0;0) B(1;-2;8) và m t ph ng (P):x-2y+2z+6=0 a Tìm M∈(P) sao cho AM BM+ nh nh t. b. Vi t phương trình m t ph ng (Q) qua A, B và c t (P) theo giao tuy n d h p v i AB góc 900 II/ Gi i h phương trình : 2 2 3 5 5 3 4 2 5.4 log log log .log x x y x y y xy xy x y x y − −  + =  + = WWW.MATHVN.COM
  • 58. 13 12: A. PH N CHUNG: Câu 1: Cho hàm s (C) ( ) ( ) 3 2 16 2 2 1 3 3 x y mx m x − = + − − + 1. Kh o sát và v th (C) khi m =0 2. Ch ng minh r ng (Cm) luôn ti p xúc v i 1 ư ng th ng c nh t i 1 i m c nh. Câu 2: 3. Gi i phương trình: ( )sin3 sin 3 cos 1x x x+ = − 4. Gi i b t phương trình : 2 0.522 2 4 log log 0,25 logx x x + ≥ Câu 3: Tính tích phân: 1 4 0 1 2 x I dx x = −∫ Câu 4: Cho hình tr có chi u cao b ng bán kính áy và b ng a. L y trên các ươgn tròn áy (O) và (O’) các i m A, B sao cho AB=2a. tính góc gi a hai ư ng th ng OA, O’B và th tích t di n O’OAB Câu 5: Cho a,b>0 và 1 1 1 a b ab + = + . Tìm giá tr nh nh t: 2 2 a b ab P ab a b + = + + B. PH N T CH N: (Thí sinh ch ư c ch n Câu 6 ho c Câu 7) Câu 6: (Chương trình chu n) a. Trong Oxy cho ∆ ABC có tâm ư ng tròn ngo i ti p là I(2;1), A∈Oy và ư ng th ng BC:3x y 10 0− − = . Tìm t a A,B,C bi t góc BAC b ng 450 và 0A By y> > b. Trong Oxyz cho A(0;1;0), B(1;-2;2). Hãy vi t phương trình m t ph ng (P) qua O, B và cách A m t kho ng b ng 2 2 c. Gi i phương trình : 4 4 1 0z + = Câu 7: (Chương trình nâng cao) a. Trong Oxy cho ( ) 2 : 2P y x= có hai tiêu i m là F . ư ng th ng d quay quanh F c t (P) t i M,N. Ch ng minh r ng 1 1 MF NF + không i. b. Trong Oxyz vi t phương trình tham s ư ng th ng qua M(1;-2;2). d ⊥ OM và d h p v i Oy m t góc 450 c. Tìm h s c a 6 x trong khai tri n thành a th c c a bi u th c: ( ) ( )1 2 1 1 nn P x x x + = + + + . Bi t h s c a 10 x b ng 10. WWW.MATHVN.COM
  • 59. 14 PH L C II: Cách gi i nhanh bài toán b ng máy tính b túi.Phép chia theo sơ Horner. Trong các kì thi quan tr ng có môn toán, máy tính b túi ư c phép s d ng và tr thành công c không th thi u i v i thí sinh. Tuy nhiên ít ai có th t n d ng ư c t i a các ch c năng c a máy tính trong gi i toán. Nay tôi xin gi i thi u m t s phương pháp tìm nghi m b ng ch c năng SOLVE c a máy tính. Bài vi t ư c vi t v i máy fx-570ES và tôi cũng khuyên các em t p làm quen s d ng máy này trong quá trình gi i toán. VD1. Tìm nghi m c nh: ( ) ( )3 2 2 3 1 6 4 0 1x a x ax− + + − = Gi i: So n phương trình (1) vào máy tính. ( )3 2 2 3 1 6 4 0x A x Ax− + + − = . D u = so n b ng cách nh n: ALPHA + CALC Nh n ti p: Shift + SOLVE Sau ó, máy h i: A=? ta cho ng u nhiên A=2 r i nh n phím = Ti p n, d a vào “linh c m” mách b o, ta oán x=-3, nh n ti p phím = Máy hi n nghi m x=0.5. Ta ghi nghi m này ra gi y. có th ây s là nghi m c nh c n tìm??!! Nh n ti p Shift + SOLVE v i A=2 L n này ta th v i x=10 Máy hi n x=2 . Thay A=-3;4;5.. và làm tương t ta ch th y máy báo x=2 V y ta k t lu n x=2 là nghi m c nh. ây chính là cách tìm nghi m c nh trong bài t p trang 35 VD2. Tìm m sao cho: ( ) ( ) ( )3 2 2 3 1 2 4 1 4 1y x m x m m x m m= − + + + + − + c t Ox t i 3 i m phân bi t có hoành >1 Gi i: So n phương trình ( ) ( ) ( )3 2 2 3 1 2 4 1 4 1 0x A x A A x A A− + + + + − + = vào máy và nh n Shift + SOLVE. Máy h i giá tr c a A. Ta cho a=3 Tai l i ti p t c oán nghi m x=-5 Máy hi n x=1.732281591 . Ta không quan tâm n nghi m này vì ây là nghi m “x u”. M c ích c a ta là tìm nghi m h u t phân tích thành nhân t . Nh n ti p Shift + SOLVE. L n này ta cho A=9 và x=10 Máy hi n x=10. Ta ghi nh n nghi m này V i A=9 cho x=-5 ta nh n ư c k t qu x=2 Th tương t v i A b ng 1 vài giá tr và th x=2, x=10 vào ta u nh n ư c thông báo x=2. V y x=2 là nghi m c nh c a phương trình. VD3. Gi i phương trình: ( )sin 2 cos2 cos 3sin 2 1x x x x+ − + = Gi i: Lúc này “lí trí” mách b o ta r ng. C n phân tích phương trình v phương trình tích. Hơn n a, ph i có nghi m “ p” m i có th phân tích ư c. Ta dùng Shift + SOLVE tìm nghi m này. Nh p phương trình trên vào máy Nh n Shift + SOLVE. Ta l n lư t th x b ng các góc c bi t như: ; ; ... 3 6 2 π π π ± ± ± Khi th n các nghi m là à 2 6 v π π thì máy hi n r t nhanh. ki m tra ta nn n: sin( _ ALPHA _X_) WWW.MATHVN.COM
  • 60. 15 Máy hi n =1 và = 1 2 . Và n u coi sin(x) là bi n thì có th phân tích phương trình qua 2 nhân t là ( )sin 1x − hay ( )2sin 1x − . Ta ch n phân tích theo hư ng ( )sin 1x − . ( )1 3sin 3 1 cos sin 2 cos2 0x x x x⇔ − + − + + = ( )2 3(sin 1) 1 1 2sin sin 2 cos 0x x x x⇔ − + + − + − = ( ) 2 3 sin 1 2(1 sin ) sin 2 cos 0x x x x⇔ − + − + − = ( )( )sin 1 1 2sin 2sin cos cos 0x x x x x⇔ − − + − = ( )( ) ( ) ( )( )sin 1 1 2sin cos 2sin 1 0 sin 1 1 2sin cos 0x x x x x x x− − + − = ⇔ − − + = n ây, ta ã hoàn thành ư c ý ưa phương trình u tiên v phương trình tích. Vi c gi i phương trình u gi ây ã tr nên d dàng. GI I CÁC BÀI TOÁN HÌNH H C GI I TÍCH B NG MÁY TÍNH B TÚI FX – 570ES Câu 1: Trong Oxyz cho: 1 2 2 : 1 1 x t d y t z = +  = − +  = ; 2 1 : 1 3 x d y t z t =  = +  = − a) Tính kho ng cách gi a d1 và d2. b)Vi t phương trình m t ph ng (P) ch a d1 và song song v i d2. Gi i: s d ng ch c năng vectơ c a máy ta nh n: MODE + 8 (vector) Ch n vectơ A máy h i ta ch n h vectơ nào (Vct A(m) m?) Ch n 1:3 Nh p t a vecto ch phương c a d1. (2;1;0) Nh n ti p Shift + STO + B copy các thông s c a vextơ A vào vectơ B. S a t a c a vectơ B thành (0;1;-1) Ta có ( )1 2(2; 1;0) ; 1;1;3M d N d− ∈ ∈ ( )1;2;3MN⇒ − (Bư c này ghi ra gi y) Nh n Shift+5(vector) Nh n 1 (Dim) 3(Vct C) sau ó nh p thông s c a vector ( )1;2;3MN − a) Theo công th c: ( )1 2 1 2 ; 1 2 ; . ; d d d d MN d d d     =     tương ng v i: ; . ; A B C A B         là các vec tơ ư c lưu trong máy tính. tính tích có hư ng c a hai vectơ &A B ta nh n: ON Shift+5 3(vct A) x Shift+5 4 = tính dài vector ta dùng ch c năng ABS(. b ng cách nh n phím Shift+hyp tính tích vô hư ng &A B c a ta nh n ON Shift+5 3(vct A) Shift+5 7:●(dot) Shift+5 4(vct B) = V y nên tính dài c n tìm ta so n vào màn hình máy tính như sau: (Abs((VctAxVctB)●VctC))÷(Abs(VctAxVctB)) K t qu máy hi n: 11 3 . b)Vi t phương trình m t ph ng (P) ch a d1 và song song v i d2: WWW.MATHVN.COM
  • 61. 16 Vi c u tiên c n làm ó là ta ph i tìm 1 vectơ pháp tuy n c a m t ph ng ( )α . g i vector pháp tuy n c n tìm là a ta th y: ( )1 1 2 2 ; a d d A d B a d  ⊥ = = ⊥ Nên a c n tìm là 1 2;d d    . tìm a b ng máy tính ta làm như sau: ON Shift+5 3(vct A) x Shift+5 4 = Màn hình so n th o hi n như sau: VctAxVctB nh n phím = xem k t qu Máy hi n: Vct Ans (-1;2;2) V y ( )1;2;2a = − . Mp( )α i qua M(2;-1;0) Nên ( ) ( ) ( ) ( ): 2 2 1 2 0 2 2 3 0x y z x y zα − − + + + = ⇔ − + + + = Thí sinh ch c n gi các bư c làm vào bài làm, công vi c còn l i hãy cho máy tính. Ta th y hoàn thành 1 bài hình h c gi i tích trong thi th t nh nhàng. Các b n có th th làm các bài toán có l i gi i trong sách giáo khoa hình h c 12 hay trong các sách tham kh o b ng chi c máy tính c a mình. S có nhi u b t ng ang ch các b n khám phá! SƠ HORNER VÀ NG D NG: Chia a th c ( ) 1 0 1 ....n n nP x a x a x a− = + + + cho ( )x c− ta có: ( ) ( )( )1 2 0 1 1....n n n nP x x c b x b x b x b− − −= − + + + + Trong ó ( )0;1;2;3;...;ib i n= nh b i sơ Horner: a0 a1 a2 a3 … c b0 b1 =cb0+ a1 b2 =cb1+ a2 b3 =cb2+ a3 bi =cbi-1+ ai Áp d ng: VD1. Tính thương và s dư trong phép chia: ( ) 4 3 2 2 8 6P x x x x x= + − − + cho x+2 Gi i: Ta có sơ Horner: 2 1 -8 -1 6 -2 2 -3 -2 3 0 V y ( ) ( )( )3 2 2 2 3 2 3 0P x x x x x= + − − + + n ây, chúng ta ã hi u ph n nào công d ng c a sơ horner. Trong bài toán liên quan n tham s , vi c tìm ư c nghi m c nh và phân tích thành tích s làm công vi c gi i toán nh nhàng r t nhi u. Nghi m c nh ã có máy tính, còn vi c chia a th c: Hãy sơ Horner làm cho b n. Ta quay l i v i ví d u ph n ph l c: VD2. Phân tích thành tích: ( ) ( )3 2 2 3 1 6 4 0 1x a x ax− + + − = Gi i: ( )3 2 2 3 1 6 4 0x a x ax− + + − = Ta ã có ư c nghi m c nh x=2. v y nên 2 -3(a+1) 6a -4 2 2 -(3a-1) 2 0 V y (1) ( ) ( )3 2 2 3 1 2 0x x a x ⇔ − − − + =  ây chính là m t ph n trong bài làm Bài3 trang 35. VD3. nh m phương trình: ( ) ( ) ( )3 2 3 4 3 7 3 0mx m x m x m A− − + − − + = có 3 nghi m dương phân bi t. WWW.MATHVN.COM
  • 62. 17 Gi i: Ta d dàng nh n ra: a+b+c+d=0 ⇒ phương trình (A) có 1 nghi m x=1 Sơ Horner: m -3m-4 3m+7 -m+3 1 m -2(m-2) m-3 0 Nên ( ) ( ) ( )2 1 2 2 3 0A x mx m x m ⇔ − − − + − =  (A) Có 3 nghi m dương phân bi t ( ) ( )2 2 2 3 0g x mx m x m⇔ = − − + − = có hai nghi m dương phân bi t u khác 1 ( ) ( ) ( ) ( ) 2 0 ' 2 3 0 2 0 3 0 1 2 2 3 0 m m m m m S m m P m g m m m  ≠  ∆ = − − − >  − ⇔ = >  − = >  = − − + − ≠ ( ) ( );0 3;4m⇔ ∈ −∞ ∪ VD4. nh m phương trình có 3 nghi m phân bi t: ( ) ( )3 1 1 0 1x m x− − − = Gi i: ( ) 3 1 1x mx m⇔ − + − Dùng máy tính ta “mò” ư c nghi m: x=1 Sơ Horner: 1 0 -m m-1 1 1 1 1-m 0 V y (1) ( )( )2 1 1 0x x x m⇔ − + + − = (1) Có 3 nghi m phân bi t: 2 ( ) 1 0g x x x m= + + − = có hai nghi m phân bi t khác 1 ( ) 3 4 3 0 3 34 1 1 1 1 0 4 3 m m m g m m ∆ = − > >  ⇔ ⇔ ⇔ < ≠  = + + − ≠  ≠ Sơ Horner ng d ng r t nhi u trong gi i toán, nh t là d ng toán liên quan n kh o sát hàm s . Các b n nên t p s d ng sơ này m t cách thu n th c. Bài t p áp d ng tôi s nêu lên 2 bài d ng chia a th c nh m giúp các b n hoàn thi n kĩ năng. BÀI T P: Bài 1. N u x=-m là m t nghi m c a phương trình 3 2 2 3 4 6 0x mx m x m− + + = . Hãy tìm ghi m còn l i. Bài 2. Cho bi u th c: ( ) 5 4 3 2 2 3 7 11 9Q x x x x x= + − − + + a. Tính giá tr bi u th c t i x=3 b. Tìm thương c a phép chia (Q) cho x-3 G i ý: Dư s c a phép chia (Q) cho x-3 là giá tr c a Q(3). WWW.MATHVN.COM
  • 63. 18 B N QUY N THU C V NHÀ XU T B N I H C SƯ PH M THÀNH PH H CHÍ MINH, KHI IN HAY TRÍCH D N PH I CÓ TÊN TÁC GI HO C NHÀ XU T B N WWW.MATHVN.COM